Anda di halaman 1dari 209

‫ﻣﻠﺰﻣﺔ ﺇﻣﺘﺤﺎﻥ ﺍﻟﱪﻭﻣﱰﻙ‬

‫‪٢٠١٦‬ﻡ‬

‫ﻟﺘﺨﺼﺺ ﺍﻟﻌﻼﺝ ﺍﻟﻄﺒﻴﻌﻰ‬


‫ﻭﻗﻞ ﺭﺏ ﺫﺩﻧﻲ ﻋﻠﻤﺎ‬ ‫ﺑﺴ ـ ـ ـ ـ ـ ــﻢ اﷲ اﻟ ـ ـ ـ ـ ـ ــﺮﺣﻤﻦ اﻟ ـ ـ ـ ـ ـ ــﺮﺣﻴﻢ‬

                  
               
                  
                 
         

‫ھﺘﺬاﻛﺮ إﯾﮫ و ﻣﻨﯿﻦ‬ ٧٠ ‫ﺳﺆال واﻟﺰﻣﻦ ﺳﺎﻋﺘﯿﻦ اﻻﺧﺘﺒﺎر ﻛﻠﻪ‬


Physical Therapy Main References
1. Muscle: Testing and Function with Posture and Pain. Physiotherapy‫ ﺳﺆال‬١٧
5th Edi on, Florence Peterson, 2005.
Orthopedics ‫ﺳﺆال‬١٧
2. Clayton`s Electro Therapy, Theory and Practice,
Angela Forster, 9th Edi on 1999. Medical knowledge ‫ ﺳﺆال‬١١
3. Tidy`s Physiotherapy, 13th Edi on. 13 Neurology & pediatrics &burn ‫ﺳﺆال‬
4. Physical Rehabilita on, Assessment and Treatment, 3rd
12 Cardiopulmonary and geriatrics ‫ﺳﺆال‬
Edi on, Susan B.Q., Sullivan, 1994.
5. Tide`s Physical Therapy, Ann Thomason, 12th Edition.
3- ‫ ذاﻛﺮ‬anatomy ‫ وﺧﺎﺻﺔ ﺟﺪا ﻛﻮﯾﺲ‬upper & lower &facial % ٥٥ ‫و اﻟﻨﺠــــــــــــــــــــــــــــــــــــــــــﺎح ﻣﻦ‬
‫اﺳﻤﮫ رھﯿﺐ ﻣﺮﺟﻊ وﻓﻲ‬
Muscles Testing
Function with Posture & Pain
‫ ﻣﻨﮫ ﺑﯿﺠﯿﺒﻮا اﻟﻤﺮﺟﻊ ھﺬا‬anatomy ‫ ﻓﯿﮭﺎ ﻣﺮﺑﻌﺎت ھﺘﻼﻗﻲ ﺑﺎﻟﻨﺺ‬muscle
‫ﺍﻟﻠﻬﻢ ﺍﻧﻚ ﻋﻔﻮ ﻛﺮﻳﻢ ﲢﺐ ﺍﻟﻌﻔﻮ ﻓﺎﻋﻔﻮ ﻋﲏ‬
&ms action
. ‫ﻛﻞ ﺣﺎﺟﮫ ﻣﻮﺟﻮده ﻓﻲ ﻣﻠﻔﺎت اﻟﺠﺮوب‬

‫ ال ذاﻛﺮوا‬electrotherapy ‫ ﺧﺎﺻﺔ ﻛﻮﯾﺲ‬us ,IR,S.W ‫ ال واﻧﻮاع‬TENS

‫ﻻ ﺗﻨﺴﻮﻧﺎ ﻣﻦ اﻟﺪﻋﺎء‬
‫ ﳏﻤﺪ ﻣﺮﻋﻰ‬.‫ﺩ‬

‫ ﺳﻴﺪ ﺻﺤﺼﺎﺡ‬.‫ﺩ‬

‫ ﺍﲪﺪ ﻓﺘﺤﻰ‬.‫ﺩ‬

‫ ﳏﻤﺪ ﻋﺼﻤﺖ‬.‫ﺩ‬
17 physiotherapy questions

Electro – EXs – P.T Topics

1-Electrotherapy has physiological effect of heat that not dependent on


which of the following:
A - type of the wave
b- size of the area to be treated
c – duration of the application
d – way application

The correct Answer is :


A : type of Wave , because that

2-In electrical stimulation intensity for muscle contraction (shortening)


depend on :

a- shape of current and electrical device used


b- Duration of ttt
c- Size of electrodes
d- Intensity and frequency of current

the correct answer is :


D – the intensiy of contraction of muscle depend on the intensity &
frequency of the current

3-For strengthening the muscle we do electrical stimulation for :


a- 0 pulse
b- 5 pulse
c-16 pulse
d-10 pulse

The correct answer is :


C- 16 pulse The more pluse give more strength
4-Muscle strength depend on all except:
a- size of muscle
b- lever arm
c- number of muscle spindle
d- type of ex

The Correct answer is :


D - Type of exercise not dependent cause for Muscle strength, but
muscle strength is also a result of the combination of three factors:
1- Physiological strength, which depends on factors such as muscle
size, the cross-sectional area of the muscle and responses to training.
2- Neurological strength, which looks at how weak or how strong the
signal is that tells the muscle to contract.
3- Mechanical strength, which refers to a muscle’s pulling force and
the way those forces can be changed using bones and joints as levers.

5-Type of laser used in PT ttt :


A. High
B. Soft
C. Mid

The correct Answer is :


C- Mid laser- Therapy lasers tend to fall into a particular category of
laser light known as 3A or 3B & are often referred to as 'mid laser'
sources.
More recently, the terms Low Level Laser Therapy (LLLT) and Low
Intensity Laser Therapy (LILT) have been adopted.
Ohshiro & Calderhead suggest that LLLT involves treatment with a dose
that causes no detectable temperature rise in the treated tissues and no
macroscopically visible change in tissue structure – essentially, the
energy can cause in increase in temperature and a change in tissue
structure, but that is not the intention with therapy laser which is
applied at levels below that needed to achieve these more overt effects
(c/f surgical laser).

6- which modality has greater effect in decrease pain in knee joint in


fatty patient :
a- ultra sonic
b- infrared
c- hot pack
d- paraffin wax

The correct Answer is :


A- Ultra-sonic waves because it`s deep than others and fatty tissue is a
good absorber for u.s waves .

7- when you treat knee with elevated layer of fat you can use :
a- IR
b- U.S
c- S.W
d- TENS

The correct answer is:


B- U.S due to present of fatty tissue .

8-Which of the following tissues absorbs the least amount of an


ultrasound beam at 1 MHz? :
a-Bone
b-Skin
c-Muscle
d-Blood

The Correct Answer is:


D-the blood Vessels is the least absorbs tissues for U.S as it described
before .

9-A 63-year-old woman presents to physical therapy with a diagnosis of


herpes zoster. The physician informs the physical therapist that the L5
dorsal root is involved and that a transcutaneous electrical
neuromuscular stimulation(TENS) unit should be used to help control the
pain. Where should the TENS unit electrodes be placed? :
a-Posterior thigh
b-Lateral hip/greater trochanter area
c-Anterior thigh
d-Anterior lateral tibia

The correct Answer is :

D – Anterior Tibia due to it`s involved in l5 nerve supply . The L5 nerve


supplies the nerves to the muscles that raise the foot and big toe.

10- modality can increase the temperature of fat?


a- ultraviolet
b- ultrasonic
c- S.W
d- FES
The correct Answer is:

B- U.S as we described above .

11-Pt treat by phototherapy what this modality


A -low laser
b -fluid therapy
c-ultrasound

the correct answer is :


A- Low Laser , because it has form of light amplifier - it provides
enhancement of particular properties of light energy.
Laser light will behave according to the basic laws of light, in that it
travels in straight lines at a constant velocity in space. It can be
transmitted, reflected, refracted and absorbed. It can be placed within
the electromagnetic spectrum according to its wavelength/frequency
which will vary according to the particular generator under
consideration.

13- In radiating energy when we use it 30 inch from the patient then
change it to 15 inch so there will be
a- radiating energy will become 4 times I
b- no changes in it
c- will be increased 2 times

the correct answer is :


A – radiating energy will become 4 times because the law of I1/d2
Where: I = Intensity, d = Distance between the source and
the point of calculation
So, to double the distance from an infrared source decreases the
intensity to (1/2)2 = 1/4 of its original value.
This is, of course, only strictly true if there is
halving the distance will quadruple the intensity.

14-T.E.N.S frequency :
a- 10 -70 HZ
b- 12 -20HZ
c- 5 - 50Hz
d- 1 - 250 Hz

the correct Answer is :


D- 1 – 250 Hz , the rate of
delivery of these pulses (the pulse frequency) will normally be variable
from about 1 or 2 pulses per second (pps) up to 200 or 250 pps

12-When using ultrasound in treating chronic bursitis of hip, the most


benefit might occur if the ultrasound frequency and dosage were:

A. 1 MHz and 1.5 Watts/cm2


B. 1 MHz and 0.5 Watts/cm2
C. 3 MHz and 1.5 Watts/cm2
D. 3 MHz and 0.5 Watts/cm2

The correct answer is :


A-1 MHz to reach in deep tissue & 1.5 Watts /cm2 which is an adequate
intensity for bursitis in hip region , so the choice is 1 MHz and 1.5
Watts/cm2

15-when you apply TENS for a patient and


you find sever pain, what would you do
A -Decrease frequency
B-Increase wave length
C-Decrease pulse width
D-Switch off device
The correct Answer is :
D – switch off device

16 -TENS stimulation
A -Presynaptic inhibition
B -Presynaptic excitation
C -Post synaptic inhibition
D -Post synaptic excitation

The correct answer is :


A- presynaptic inhibition , TENS unit reduces pain through nociceptive
inhibition at the presynaptic level in the dorsal horn, thus limiting its
central transmission. The electrical stimuli on the skin preferentially
activate low-threshold, myelinated nerve fibers. The afferent input from
these fibers inhibits propagation of nociception carried in the small un-
myelinated C fibers by blocking transmission along these fibers to the
target or T cells located in the substantia gelatinosa (laminae 2 and 3)
of the dorsal horn .

17-when treating chronic pain which type of TENS used “


a- brief
b- conventional
c- burst
d- acupuncture

The correct answer is :


D- acupuncture like a tens ,

In acute stage we use : convential or high frequency


In sub acute stage we use : burst
In chronic stage we use : low frequency (acupuncture)

18-pat has chronic brachialgia the best tense use :


a-conventional
b-low frequency
c-burst
d-intense

The correct answer is :


B – Low frequency , described as above
19- Patient with elbow pain for 5 months and just stopped playing
tennis, which type of TENS would you use:

a- Conventional TENS
b- Acupuncture like TENS
C- Brief intense TENS
d- Bust TENS

The correct answer is :

B-Acupuncture like Tens , described as above

Traditional TENS (Hi TENS) Usually use stimulation at a relatively high


frequency (90 - 130 Hz The stimulation is delivered at ‘normal’ intensity -
definitely there but not uncomfortable.

Acupuncture (Lo) TENS Use a lower frequency stimulation (2-5 Hz) with
wider (longer) pulses (200-250ms - still not at the patients’ threshold,
but quite a definite, strong sensation.

burst TENS interrupting the stimulation outflow at rate of 2 - 3


bursts/second. The stimulation intensity will need to be relatively high .

extract :
Acupuncture TENS is used for any chronic pain

20-Prevention of transmission of pain beyond the dorsal horn


a- peripheral n injury
b- gate control theory
c- spinal cord injury
d- opiate theory

The correct answer is :


B – gate control theory , described as above

21-Baby with flaccid klumps' palsy need stimulation by:


a-interrupted galvanic stimulation
b- reciprocal stimulation
c-Diadynamic

The correct Answer is :


A- interrupted galvanic stimulation , due to we use galvanic stimulation
with denervated muscles

22-Decrease edema in ankle joint by :


a) Galvanic Stimulation
b) Interferential
c) Russian current
d) Monophonic rectangular

The correct answer is :


A- Galvanic stimulation because Monophasic Rectangular is a type of
direct galvanic current or interrupted direct current or High voltage
interrupted galvanic stimulation use for decrease edema next to it
Faradic current.

23- To decrease edema


a- H.V.P.C (high voltage pulsed current)
b- b- Rectangular mono phasic

the correct answer is :


a- H.V.P.C = high voltage pulsed galvanic stim. .

24- Decrease edema in ankle joint by:


a- H.V.G.S (high voltage galvanic stimulation)
b- Interferential
c- Mono phasic rectangular
d- Russian current

the correct answer is :


A- H.V.G.S as we described before

25- All of the following are physiological effect of electrical stimulation


except:
a- Vasoconstriction of blood vessels
b- Increase blood flow
c- Increase metabolic rate

The correct answer:


A- Vasoconstriction of blood vessels isn`t physiological effect of
electrical stimulation instead of it vasodilatation, increase blood flow,
increase metabolic rate, increase temperature of the skin, increase
stimulation of the neural receptors .

26- Contraction of muscle by electrical stimulation which not occur :


a- increase demand of oxygen
b- increase metabolic rate.
c- Increase waste out.
d- vasoconstriction of blood vessels

the correct answer :


D – vasoconstriction of blood vessels , as described above .

27- Electric stimulation which is not true


a- Vasodilatation
b- Increase metabolic rate
c- Decrease muscle spasm
d- Vasoconstriction

The correct answer is:


D – Vasoconstriction of blood vessels

28- Heating with (E.S) which is not correct?


a- V.D
b- Decrease metabolic-rate
c- Decrease muscle spasm

The correct answer is :


B-decrease metabolic rate , described as above.

29-Electrical nerve stimulation frequency is:


A-23-120
B-50-150
C-10-70

The correct answer is :


B- 50 to 150 HZ not below can`t stimulate & not above to avoid fatigue
of muscle.

30-The rate of electrical nerve stimulation through the skin


A- 10-70cycle/second or Hz
B-35-120cycle/second or Hz
C-50-150cycle/second or Hz
D-15-200cycle/second or Hz

The correct answer:

C- 50-150cycle/second or HZ, described as above.

31- Which current used for stimulated Denervated Muscle?


a- TENS
b- Faradic
c- Diadynamic
d- Direct galvanic

The correct Answer is:

D- Direct galvanic is the current which can stimulate denervated


Muscle directly.

32-Your plan of care includes use of iontophoresis in the management of


calcific bursitis of the shoulder. To administer this treatment using the
acetate ion, the current characteristics and polarity should be:

A-monophasic twin peaked pulses using the positive pole


B-monophasic twin peaked pulses using the negative pole
C-continuous monophasic using the positive pole
D-continuous monophasic using the negative pole

The correct Answer is :


D – Continuous monophasic is a direct galvanic current and we using the
negative pole When we use ionophoresis it must be direct current or
continuous monophasic, and must put the chemical substance in the
same polarity of the electrode, so we choose negative pole due to the
substance is acetate ion.
33-one of these electrical stimulation methods does not stimulate
denervated muscle:
A high voltage interrupted long duration
B TENS
C galvanic
D faradic

The correct Answer is:


B-Tens current play its role in a gate control theory which needed
nerves and receptors to make its role for modulating pain.

34 -frequency that can make titanic muscle contraction


a- 50
b- 30
c- 20

The correct Answer is:


A- 50, as we described above to stimulate innervated muscle at least
should be 50 Hz.

35 - tennis elbow player has pain in elbow joint 5 month ago and there is
production of pro-inflammatory cytokines .what can we use?
A. Pulsed us
B. Continuous us
C. Pulsed with lidocain
D. Continuous with lidocain

The correct Answer is:

C- Pulsed with Lidocain.


36-U.S has these frequencies:
b- 1 or 3 MHZ
c- 1 or 5 MHZ
d- 1 or 5 HZ

The Correct Answer is:


B- 1 or 3MHZ

37-Patient has fracture upper tibia and fixated by plate and screw.
Complain from complication of knee limitation and pain. Which
modalities are contra-indicated?
a- ultra sonic
b- Faradic current
c- Short wave
d- Ice application

The correct Answer is:


C- Short wave because it`s cause deep heating for a large area not
specific like U.S, and it`ll cause pain due to increase the heat absorbed
by metals in plate and screw.

38-U.S can be mostly absorbed by skeletal muscle as it mostly contains:


a- adipose tissue
b- Connective tissue
c- Nerve endings
D- Proteion

The correct answer is :


B – Proteion, as we described above.

39-Use of continuous ultrasound at 1.5 Watts/cm2 will result in:

a- no change in nerve conduction velocity


B-increase in motor nerve conduction velocity and decrease in sensory
nerve conduction velocity
C-decrease in motor nerve conduction velocity and increase in sensory
nerve conduction velocity
D-increase in both motor and sensory nerve conduction velocity

The correct Answer is:


D- Increase in both motor & sensory nerve conduction velocity.

40- When use iontophoresis; it is a type of :


a- low frequency TENS
b- HVGS
c- Direct current
d- Russian current

The correct answer is:


C- Direct Current as described above

41-Which of the following is contraindicated to ultrasound at 1.5


watts/cm2 with a 1 MHz sound head?
A- Over a recent fracture site.
B- Over non-cemented metal implant.
C- Over a recently surgically repaired tendon.
D- Over the quadriceps muscle belly.

The correct answer is:


C-Over recently surgically repaired tendon is contraindicated, over
cemented plate contraindicated, but the other can we use U.S with it.

42-The therapist has given a patient an ultraviolet treatment. The


patient calls the therapist the next day with complaints of peeling and
itching. These signs and symptoms resolve three days later (a total of 4
days after treatment). What dosage did the patient receive?

a- Sub-erythemal dose
b- Minimal erythemal dose.
c- First-degree erythemal dose.
d- Third-degree erythemal dose.

The correct Answer is:


C-first degree Erythemal dose

43– pt has infected lumber wound use


a) Ultra Voilet
b) U/S
c) TENS
d) faradic

The Correct Answer is:


A- ultra violet used for infected wound and with chronic wound we use
HVPC

44- Patient with groin pain 2 weeks ago used :


a- cold + interferential
b- hot + interferential
c- pulsed US + interferential
d- continue US + interferential

The correct Answer is :


D-continue US+Interferential due to after 2 weeks ago it`s mean that it`s
chronic so we use Us continues waves to make a thermal effect used
for treatment chronic deep pain.

45- pt has pain due to deep ms strain we use :


a) Pulsed U/S 1MHZ
b) ,, ,, 3MHZ
c) Cont U/S 1 MHZ
d) ,, ,, 3 MHZ

The correct Answer is :

C- Cont. Us 1MHZ , as described above

46-what is best for scar tissue

A- ultra sound
B -infrared
C –shortwave

The correct Answer is :


A-ultra sound waves penetrate well more than Infrared and shortwave
for Scar Tissue.

47-After applying a hot pack to a patient you observe hyperemia in the


treatment area after a few minutes. As the therapist you should:

A-remove the hot pack immediately and assess for a burn.


B-keep the hot pack in place as this is indicative of heat dissipation
secondary to vasodilation.
C-elevate the part to minimize edema formation.
D-remove the hot pack immediately as the patient is having a potentially
dangerous histamine reaction.

The correct Answer is :


B- keep the hot pack to a patient because it`s the action of the heat to
make hyperemia due to vasdilation

48-Muscle reeducation In comparing the use of cold pack and hot pack
treatments, which of the following statements is false?

A-Cold packs penetrate more deeply than hot packs.


B-Cold increases the viscosity of fluid and heat decreases the viscosity
of blood.
C-Cold decreases spasm by decreasing sensitivity to muscle spindles
and heat decreases spasm by decreasing nerve conduction velocity.
D-Cold decreases the rate of oxygen uptake, and heat increases the rate
of oxygen uptake.

The in correct Answer is:


C- Cold decreases spasm by decreasing sensitivity to muscle spindles
and heat decreases spasm by decreasing nerve conduction velocity.

cold : decrease spasm by decrease muscle tension by increasing in the


threshold of activation of the muscle spindle Decreased muscle spasm
The muscle relaxation proba­bly results from a decrease in firing rates
of the efferent fibers in the muscle spindle

49-A patient recently diagnosed with multiple sclerosis presents to a


physical therapy clinic. The patient asks the therapist what she needs
to avoid with this condition. Which of the following should the patient
avoid?
a- Hot tubs.
b- Slightly increased intake of fluids.
c- Application of ice packs.
d- Strength training.

The correct Answer is:


A- Hot Tubs , any hot application is avoided for multiple sclerosis

50-SWD contra indication is


a- hematoma
b- arterial insuffiency
c- sinusitis

The Correct Answer is:


B-arterial insuffiency ,
Contra-indications to short-wave diathermy:
Arterial disease, Metal in the tissues, Disturbed skin sensation,
Pregnancy, Tumors, X-ray therapy, venous thrombosis or Phlebitis
, It is unsafe to apply short-wave diathermy to patients who are unable
to understand the degree of heat.

51-SWD is contraindicated in
a- Infected wounds
b- Peripheral vascular disease
c- Hematoma
The correct Answer is:
B-peripheral Vascular disease as we described above

52-The most affective distance between IRR and patient is


a-30cm
b-90cm
c-70cm

The correct Answer is:


C- 70 cm is the effective distance between the IRR & the Pt.

53- Which of the following statements is false about treatment with


infrared lamps?

A- Near infrared heats deeper than far infrared.


B- Infrared lamps heat both sides of an extremity at one time.
C-The therapist can change the intensity of the heat by changing the
angle between
D- The beam and the body part being treated

The correct Answer is:


B- Infrared lamps heat both sides of an extremity at one time because it
affect only on the side which radiated on.

54-To treat patient with deep heat, which is not form of deep heat
a- SWD
b- MWD
c- IR ( infra-red)
d- US (ultra sound)
The corrected Answer is:
C- IR is a superficial heating

55-Cryotherapy contraindication
a- peripheral artery disease
b- acute injury
c- acute inflammation

The Correct Answer is:


A-peripheral artery disease
56-Burn in the body's response to thermal insult from external agent
such as:
a- heat and cold
b- chemicals
c- electricity and radiation
d- all of the above

The correct Answer is:


D-all of the above are external agent may cause thermal insult

57-Burns caused due to constant direct currents are called


a- Galvanic burns
b- Chemical burns
c- Electrolytic burns
d- All of the above

The correct Answer is:


D- all of the above

58- Coupling media of ultrasonic which of the following of least effect


A -aquatic gel
B -glycerol
C - liquid paraffin
D – crrogel

The correct Answer is:


C-liquid paraffin is the least effect for coupling media of Ultrasonic.

59- Coupling media of ultrasonic which of the following of fastest effect


A -aquatic gel
B -glycerol
C - liquid paraffin
D – crrogel

The correct Answer is:


A-Aquatic gel is the fastest effect for coupling media

60 -After exposure to U.V.R patient of come with pigmentation due to


release
a-histamine
c- melanin
d- bradykinin
e- endorphin

The correct Answer is :


C- melanin is a responsible for a pigmentation .

61 -After exposure to U.V.R patient of come with Erthyrema due to


release
a-histamine
f- melanin
g- bradykinin
h- endorphin

The correct Answer is :


A- histamine is a responsible for Erthyema

62- Short wave (SWD) contraindications:


a- Phlebitis
b- Sprain
c- Muscle strain
d- Hematoma

The correct Answer is:


A- phlebitis as we described above.

63- for denervated ms we use at least


A-10 m.sec
B-50 m.sec
C-100 m.sec
D-300 m.sec

The correct Answer is:


C- 100 m.sec at least with denervated muscle , if he dosen`t say at least
we should choose 300 m.s or more

64-If there is tendonitis in this muscle the best method for treatment
a- US with deep friction massage
b- T N E S
c -SWD.
The correct Answer is:
a-US with deep fiction massage as we described above.

65-Therapist use electrical method for pumping action with limb


elevated and compressed to
A-decrease edema
B-increase circulation
C-increase venous return

The correct Answer is:


A- Decrease edema, because we do pumping action with compressed
limb elevated

66-In bell`s palsy we use


a- continues direct frequency current
b- T.E.N.S
c- Interrupted direct current

C-interrupted direct current , because it`s innervated muscle.

67-patient coming with industrial burn involved the dorsum muscle of


the left hand after study there is dis-continues chronoxia227/m. so we
could use:
a- continues direct current
b- faradic current
c- interrupted direct current

The correct Answer is:


A- continues direct current , due to it`s appear in discontinues chronoxia
which mean that it`s denervated muscle

Rheobase: is the smallest current that will produce a muscle


contraction if the stimulus is of infinite duration; in practice an impulse
of 1ms (o. I sec) is used.

In denervation the Rheobase may be less than that of innervated


muscle, and it often rises as re-innervation commences. These changes
are not however sufficiently predictable to be reliable guides.

The Rheobase varies considerably in different muscles and according to


the skin resistance and temperature of the part, while a rise may be due
to fibrosis of the muscle.
Chronaxie: The chronaxie is the duration of the shortest impulse that
will produce a response with a current of double the rheobase.

The chronaxie of innervated muscle is appreciably than that of


denervated muscle, the former being less and the latter more than 1ms
if the constant voltage stimulator is used. With the constant-current
stimulator the values are higher, but bear a similar relationship to each
other.

Chronaxie is not a satisfactory method of testing electrical reactions as


partial denervation is not clearly shown, the chronaxie being that of the
predominant state of the fibers; for example, the chronaxie of a muscle
with 25% of its fibers innervated would be the same as that of a
completely denervated muscle.
.

68-Applied electrical stimulation to area for long lasting period cause


a-skin irritation
b-weakness of ms
c-edema occure

The correct Answer is:


A- skin Irritation

69-Pt of bells palsy long time treated with (E.S) you


A- M.S weakness
B-no change
C-fibrosis in the affected side

The correct Answer is:


C-fibrosis in the affected side due to long use of electricity to stimulate
the muscles.

70-Aphysical therapist is performing electro -myographic testing. During


a maximal output test of the patient’s quadriceps muscle, 35% of the
motor unit potential is poly-phasic. What is the significance of this
finding?
A-It is normal in the quadriceps.
B-It is normal in the triceps brachii, not in the quadriceps
C-It is normal in the biceps brachii, not in the quadriceps
D-It is abnormal in any muscle.
The correct Answer is:
D- it`s abnormal in any muscle

71- Benefit of isotonic exercise except:


a- high mechanical energy
b- Less resistance to fatigue
c- High efficiency
d- Not delayed muscle soreness

The correct Answer is:


D- not delayed muscle soreness

72-During immersion technique in paraffin wax, the temperature will be.


A- 37-41
B- 41-43
c- 43-47.
d- 47-51

The correct Answer is:


B- 41 to 43 , the paraffin wax Temp. from 40 to 44 C

73-Paraffin wax bath’s temperature should be kept at


a. 30°- 35°C
b. 40°- 44°C
c. 50° - 53°C

The correct Answer is:


B- 40 -44 C , as we described above.

74-Apatient arrives at an outpatient clinic with an order from the


physician for whirlpool and wound care to a lower Extremity wound. The
therapist decides to test the temperature in the whirlpool at warm.
Which of the following Setting in degrees Celsius is appropriate?
A-27.5o Celsius
B-35.5o Celsius
C-49o Celsius
D-60o Celsius

The correct Answer is :


B- 35.5 C
75-Manipulation is described in the management of
A-osteoporosis
B-spinal dysfunction
C-osteomyelitis

The correct Answer Is:


B- spinal dysfunction , we contra indicated to make any manipulation
technique for osteoporosis or osteomyelitis .

76-Accessory movement is not


A-necessary to completion of joints ROM
B-Passively occur
c-under voluntary control

The correct Answer is:


C- under voluntary control ,

77-An athlete with stable fracture of neck of humerus, referred to you To


improve mobility, which form of exercise will you choose
a- Isometric
b- Resistive exercise in all directions
c- Pendulum ex

The correct Answer is:


B- Resistive Exe. Because there is no need to fear from the resistance
movement of the arm

78-An athlete with unstable fracture neck of humerus , refereed to you


to improve mobility. Which form of exercise will you chose
a- isometric
b- resistive exercise in all directions

The correct Answer is:


A-Isometric exe. To prevent refracture of the humreus.

79-Percusiion is applied from


a- 1 to 3 min
b- 2 to 4 min
c- 2 to 5 min
d- 2 to 6 min

The correct Answer is:


C- 2to 5 min. in excessive secretions 3-5 Min

80-Capsular tightness has limited your patients ability to fully extend


her knee. Treatment to restore joint motion
a- anterior glide , external rotation of tibia
b- anterior glide , internal rotation of tibia
c- posterior glide, e1xternal rotation of tibia
d- posterior glide, internal rotation of tibia

The correct Answer is:


A- Ant. Glide with external rotation of tibia will increase th extension of
the Knee .

Post Glide with internal rotation increase flex

81-Boy have 16 years old have acute knee sprain after chronic we
advice :
a- knee protection
b- knee immobilization
c- range of motion
d- hot application
The correct Answer is:
b-Knee immobilization

82-Following cast immobilization for a now healed supracondylar


fracture of the humerus, a patient’s elbow lacks mobility. To increase
elbow range of motion, joint mobilization in the maximum loose-packed
position should be performed at:
A-full extension
B-90 degrees of flexion
C-70 degrees of flexion
D-30 degrees of flexion

The correct Answer is:


C-70 degree of flexion is the maximum loose – packed position .

83-During under water exercises which will be difficult


a- moving the limb horoziantly
b- moving the limb toward bottom
c- moving the limb to the surface
The correct Answer is:
B- moving the limb toward bottom (against bouncy force of water)

84-At the beginning of the muscle activity which make increase of blood
supply to it
a- sympathetic adrenergic
b- sympathetic cholerigric
c- parasympathetic

The correct Answer is:


A-sympathetic Adrenergic

85-Which of the following is not benefit in physiotherapy:


a-increase muscle fiber size
b-increase heart rate
c-increase ligaments and tendons
d-increase the power structure of bones

The correct Answer is:


C-increase ligaments & Tendons , it can`t be increased by
physiotherapy.

86-inorrect about poor posture


A)need surgery and physiotherapy
B)spinal and frozen shoulder problems
C)pain and affect strength of muscles
The correct Answer is:
A-need surgery and physiotherapy

87-Application of intermittent traction to cervical depend on:


a- wt of patient
b- localization of pain
c- proper diagnosis and knowing problem
The correct Answer is:
C- proper diagnosis and knowing the problem

88-Which of the following is the normal end-feel perceived by an


examiner assessing wrist flexion?
a-Bone to bone
b-Soft tissue approximation
cTissue stretch
d-Empty
The correct Answer is:
C-tissue stretch

89-A physical therapist should place the knee in which of the following
positions to palpate the lateral collateral ligament ( LCL)?
a- Knee at 60o of flexion and the hip externally rotated.
b- Knee at 20o of flexion and the hip at neutral.
c Knee at 90o of flexion and hip externally rotated.
d- Knee at 0o and the hip at neutral.

The correct Answer is:


C-knee at 90 of flexion and hio externally rotated

90-Atherapist is mobilizing patient’s right shoulder. The movement


taking place at the joint capsule is not completely to end range. It is a
large amplitude movement from near the beginning of available range to
near the end of available range. What grade of mobilization according to
Maitland, is being performed?
a-Grade I
b- Grade II
c-Grade III
d-Grade IV

The correct Answer is:


B-Grade 2 Large amplitude within near the beginning range of motion to
near the end of range of motion
Maitland Joint Mobilization Grading Scale
Grade I- Small amplitude rhythmic oscillating mobilization in early range
of movement
Grade II- Large amplitude rhythmic oscillating mobilization in midrange
of movement
Grade III- Large amplitude rhythmic oscillating mobilization to point of
limitation in range of
movement Grade IV- Small amplitude rhythmic oscillating mobilization
at end range of
movement
Grade V- (Thrust Manipulation) - Small amplitude, quick thrust at end
range of movement

91-A 67-year-old woman presents to an outpatient facility with a


diagnosis of right adhesive capsulitis. The therapist plans to focus
mostly on gaining abduction range of motion. In which direction should
the therapist mobilize the shoulder to gain abduction range of motion?
A. Posteriorly
B. Anteriorly
C. Inferiorly
The correct Answer is:
C-inferiorly to gain more abduction , posteriorly to gain more flexion ,
Anteriorly to gain more Extension

92-You wish to mobilize a patient’s shoulder using an inferior glide


technique. It would be best to use this technique be propositioning the
patient’s arm in:
a- 95 degrees of abduction with lateral rotation
b- 125 degrees of abduction and internal rotation
c- 55 degrees of abduction and neutral rotation
d- 95 degrees of shoulder flexion and neutral rotation

The correct Answer is:


C-55 degree of abduction with internal rotation , glenohumeral lose pack
position , 55 degree of abduction and 30 horizntal adduction
the close packed position of the glenohumeral joint : abduction and
external rotation

93-While observing the ambulation of a 57-year-old man with an arthritic


right hip, the therapist observes a right lateral trunk lean. Why does the
patient present with this gait deviation?
a-To move weight toward the involved hip and increase joint
compression force .
b-To move weight toward the uninvolved hip and decrease joint
compression force .
cTo bring the line of gravity closer to the involved hip joint.
d-To take the line of gravity away from the involved hip joint .
The correct Answer is:
C- To bring the line of gravity closer to the involved hip joint to decrease
weight bearing on it.

94-in shoulder inferior gliding position of the arm


a-slight flexion
b-slight extension
c-30 degree adduction
d-external rotation
The correct Answer is:
C-30 degree horizontal adduction

95- An 87-year-old woman presents to an outpatient physical therapy


clinic complaining of pain in the left sacroiliac joint. The examination
reveals higher left anterior superior iliac spine (ASIS) than the right
ASIS, shorter leg length on the left side (measured in supine position),
and left side posterior superior iliac spine (PSIS) lower than the right
PSIS. In what position should you place the patient to perform the
correct sacroiliac mobilization of the left in nominate?
a- Right side lying
b- Supine
c- Prone
d-None of the above

The correct Answer is:


D- None of the above , the Sacroiliac joint disappear after 80 years old

96-What is this position called :


a-half kneeling
b-prone kneeling
c-sitting kneeling
d-half kneeling

The correct Answer is:


B-prone kneeling

97-The ability of the posture to modify external environment and


preserve alignment of one body part to another?
a- Postural control
b- Strength
c- Balance

The correct Answer is:


A-Postural Control

98-Ex’s do with optimal performance and max capacity with well being
is?
A-Fitness
B- Power
C- Co-ordination

The correct Answer is:


A-fitness

99- What is the best way to first exercise the postural (or extensor)
musculature when it is extremely weak to facilitate muscle control?
a- Isometric-ally .
b- Concentrically.
c- Eccentrically.
d- Isometric kinetically.

The correct Answer is:


A-isometrically

100-when detect target heart rate in unstable angina we don`t make


a-valsalva maneuver
b-isometric ex
c-heavy arm ex
d-isotonic ex

The correct Answer is:


B-isometric exersice.

Training or target heart rate is one of the methods to detect


exercise intensity not type of exercise .
Mode of exercise : interval which is better for cardiac patients and
continuous Types of exercise: aerobic & circuit weight training &
respiratory training

101-when detect target heart rate in unstable angina we do:


a- valsalva maneuver
b- isometric ex
c- isotonic ex
The correct Answer is:
c-isotonic exersice.
102-The therapist works in a cardiac rehabilitation setting. Which of the
following types of exercises are most likely to be harmful to a 64-year-
old man with a history of myocardial infarction?
a-Concentric
b-Eccentric
c-Aerobic
d-Isometric

The correct Answer is:


D- Isometric exersice.

104- A67 y female has acute sever RH.ARTHRITIS Physiotherapy in future


do strengthening ex which one is better
a-isometric ex
b-isotonic ex
c-iso kinetic ex
d-immobilization

The correct Answer is:


A- Isometric exe.

105-Type of contraction which entire MS acting over two joint muscle


a-active insuffiency
b-passive insuffiency
c-isometric cont

The correct Answer is:


A-Active insuffiency is the contraction muscle acting over two joint
muscle .

106-Muscle acting over two joint can`t contract on two joint at same
time
a-active insuffiency
b-passive insuffiency
c- isometric cont

The correct Answer is:


B- Passive insuffiency is the muscle which stretched over two joints at
the same time of contracting the opposite side .

107-If the proximal bone of the joint is fixed and the distal part of the
limb moves in sequence :
a. ACL training
b. Closed-chain exercise
c. Open-chain exercise

The correct Answer is:


C- Open –chain exercise

108-Type of contraction which MS lengthening to provide tension


a-eccentric cont
b-concentric cont
c-isotonic cont

The correct Answer is:


A-Eccentric contration which MS lengthening to provide tension
Concentric contraction which MS shortining to provide tension

109-The most important factor to TTT endurance :


a-intensity
b-duration of TTT
c-repetitions per set

The correct Answer is:


C-Repetition per set for endurance , intensity for strength

110- which statement incorrect about passive movement to


a-passive movement segment must hold gently
b-proximal part must supported
c-passive movement from proximal to distal to increase lymph and
venous return.

The correct Answer is:


C- passive movement from proximal to distal to increase lymph and
venous return

111-Ex`s which occur in round and consist of continuous training


a-open kinematic chain
b-close kinematics chain
c-circuit training
d-isometric
The correct Answer is:
C- circuit training

112-Therapist measure target heart rate to determine


a-frequency of exercise
b-intenisty of exercise
c-duration of exercise
d-type of exercise
The correct Answer is :
B- intensity of Exercise as we described above .

113-One of the following does not use phosphotognase of the body:


a- weight lifting
b- jogging
c- running a marathon
d- diving
The correct Answer is:
A-weight lifting is anaerobic exe.

contract
not contract
not Contract
Relaxed
contract

114- One 0f the following is not depends on oxygen of the body


a- weight lifting
b- jogging
c- running a marathon
d- diving

The correct Answer is:


A- weight lifting as we described above.

115- One 0f the following is not depends on oxygen of the body:


a- aerobic
b- anaerobic

The correct Answer is:


B-anaerobic
116- One of the following is not an aerobic exercise:
A- weight lifting
B- jogging
C- running a marathon

The correct Answer is:


A-weight lifting

117- Microwaves are strongly absorbed by


a- Water
b- Fat
c- Protein
d- Muscle tissue

The correct Answer is:


A-Water

118-Generally the ROM not depends on:


a-bony structure
b-elasticity of muscle , ligament and tendon
c-tightness of muscle, ligament and tendon
d-bulk of the soft tissue

The correct Answer is:


D-Bulk of the soft tissue.

119-How often dose the joint commission on accreditation of healthcare


organization ( JCAHO ) require that all electrical equipment in hospitals
be inspected ?
A. Every 3 months
B. Every 6 months
C. Every 12 months
D. Every 3 years

The correct Answer is:


C- every 12 months.

120-During weight bearing, a soft tissue contracture resulting in


supination of the forefoot will be compensated for by:
A. pronation of the forefoot
B. pronation of the rear foot
C. supination of the rear foot
D. pronation of the forefoot and rear foot

The correct Answer is:


B-pronation of the Rear foot

(((((((Splints&Orthosis&prosthesis Questions (((((((((

121- Which is CORRECT ABOUT patellar tendon bearing orthotic:


a- increase loading of proximal leg
b- suspect decrease 50% load
c- unload of hip joint

The Correct Answer is:


B-suspect decrease 50 % load PATELLA TENDON BEARING ANKLE
FOOT ORTHOSIS
AFO Patella Tendon Bearing Orthosis - Unloading Patella Tendon
Bearing Orthosis is a custom ankle foot orthosis with external bracing
designed to unweight the ankle or heel.
Off-loading up to 100% of forces, it utilizes a below-the-knee socket
design that

122-Patellar tendon loading-orthosis which isn’t correct?


a- Carry proximal leg load
b- Carry 50% of load
c- Used with assistive elbow cruches
d- Decrease load in hip joint

The Correct Answer is:


D- Decrease load in hip joint

123-When using a patellar-tendon-bearing prosthesis, a patient will


experience excessive knee flexion in early stance if the:
A-socket is aligned too far anteriorly
B-socket is aligned too far posteriorly
C-foot is outset excessively
D-foot is inset excessively

The Correct Answer is:


A-socket is aligned too far anteriorly
124-While evaluating a patient who has just received a new left below-
knee prosthesis, the therapist notes that the toe of the prosthesis stays
off the floor after heel strike. Which of the following is an unlikely cause
of this deviation?
A- The prosthetic foot is set too far anterior
b- The prosthetic foot is set in too much dorsiflexion
C- The heel wedge is too stiff.
d-The prosthetic foot is outset too much.

The Correct Answer is:


D- The prosthetic foot is outset too much.

125-A 17-year-old football player is referred to the outpatient physical


therapy clinic with a diagnosis of a recent third-degree medial collateral
ligament sprain of the knee. The patient wishes to return to playing
football as soon as possible. Which of the below is the best protocol?

A- Fit the patient with a brace that prevents him from actively moving
the knee into the last available 20o of extension. Prescribe general
lower extremity strengthening with the exception of side lying hip
adduction.
B-Do not fit the patient with a brace. All lower extremity strengthening
exercises are indicated.
C -Fit the patient with a brace that prevents him from actively moving
the knee into the last available 20o of extension. Avoid all open-chain
strengthening for the lower extremity.
D-Do not fit the patient with a brace. Prescribe general lower extremity
strengthening with the exception of side lying hip adduction.

The Correct Answer is:


A- fit the patient with a brace that prevents him form actively moving
the knee into the last available 20 degree of extension . prescribe
general lower exteremity strengthening with exception of side lying hip
adduction.

126-The therapist is treating a patient who has suffered a recent stroke.


There is a significant lack of dorsiflexion in the involved lower extremity
and a significant amount of medial/lateral ankle instability. The therapist
believes than an ankle foot orthosis (AFO) would be beneficial. Which of
the following is an appropriate AFO?
A-Solid AFO
B-Posterior leaf spring AFO
c-Hinged solid AFO
D-A or C

The correct Answer is:


D-A or C to prevent supination or sprain of the medial or latral side of the
foot and prevent dorsiflexion .

127-Which of the following is the most energy efficient and allows a T1


complete paraplegic the most functional mobility during locomotion?
a-Manual wheelchair
b-Electric wheelchair
c-Bilateral knee ankle orthoses and crutches
d-Bilateral ankle-foot orthoses and crutches

The correct Answer is:


A-Manual Wheelchair

128-The therapist is ambulating a 42-year-old man who has just received


an above knee prosthesis for the left leg. The therapist notices pistoning
of the prosthesis as the patient ambulates. Which of the following is the
most probable cause of this deviation?
A-The socket is too small
B-The socket is too large
C-The foot bumper is too soft.
D-The foot bumper is too hard.

The correct Answer is:


B-The socket is too large.

129-Patellar tendon bearing POP cast is indicated in the following


fracture
a- Patella
b- Tibia
c- Medial malleolus
d- Femur

The correct Answer is:


B-Tibia
130-Which of the following is used to treat a patient referred to physical
therapy with a diagnosis of Dupuytren’s contracture?
a- Knee continuous passive motion (CPM)
b- Work simulator set for squatting activities.
c-Hand splint.
d- A two-pound dumb bell

The correct Answer is:


C- Hand splint because Dupuytren`s contracture.

131-A therapist is assisting a patient with pre-gait activities who has


been fitted with hip disarticulation prosthesis. To ambulate with the
most correct gait pattern, what must be mastered first?
a-Forward weight shift on to the prosthesis
b-Swing-through of the prosthesis
c-Maintain stability while in single limb support of the prosthesis
d-Posterior pelvic tilt to advance the prosthesis

The correct Answer is:


D- posterior pelvic tilt to advance the prosthesis.

132-The therapist in an outpatient physical therapy clinic receives an


order to obtain a shoe orthotic for a patient. After evaluating the patient,
the therapist finds a stage I pressure ulcer on the first metatarsal head.
Weight bearing surfaces need to be transferred posteriorly. Which
orthotic is the most appropriate for this patient?
a- Scaphoid pad.
b- Thomas heel.
c-Metatarsal pad
dCushion heel

The correct Answer is:


C- Metatarsal pad , Metatarsal pads successfully transfer weight onto
the metatarsal shafts of this patient.

133-Which of the following is the most appropriate orthotic for a patient


with excessive foot pronation during static standing?
A-Scaphoid pad
B-Metatarsal pad
C-Metatarsal bar
D-Rocker bar
The correct Answer is:

A- Scaphoid pad ,Thomas heel and a scaphoid pad are for patients with
excessive pronation.

134-Which of the following is a false statement about below-knee


amputations?
a-Gel socket inserts should be left in the prosthesis overnight
b-The therapist should puncture any blisters that appear on the stump
c-Areas of skin irritation on the stump can be covered with a dressing,
then a nylon sock before donning the prosthesis.
d-When not in use the prosthesis should be lain on the floor

The correct Answer is:


B- The Therapist should puncture any blisters that appear on the stump

A cushion heel absorbs shock at contact


http://orthopedics.about.com/.../find.../g/Thompson-Test.htm

134-Patient has drop foot and use prosthetics he complain


from redness after walking 60 steps your response
a- tell the doctor
b- still use the prothetic
cstop use untill reacomedation occurred
d- neglect the cause

The correct Answer is:


C- stop until recommendation occurred

135-A patient developed tarsal tunnel syndrome secondary to a pre-


existing pes valgus. All of the following shoe modifications might help to
correct this problem except for a:
a-medial heel wedge.
b-heel insert.
c-scaphoid pad.
d-rocker sole.

The correct Answer :


D- rocker Sole

136- prothotic thermoplastic material made from


a- Low temperature thermoplastic material
b- Medium temperature thermoplastic material
c- High temperature thermoplastic material
d- Thermositting plastic

The correct Answer :


A-Low temperature thermoplastic material.

137- A patient presents to physical therapy with a long-standing


diagnosis of bilateral pes planus. The therapist has given the patient
custom-fit orthotics. After using the orthotics for 1 week, the patient
complains of pain along the first metatarsal. The therapist decides to
use joint mobilization techniques to decrease the patient’s pain. In
which direction should the therapist mobilize the first metatarsal?
a-Inferiorly
b-Superiorly
c-Laterally
d-A and C

The correct Answer is:


A- inferiorly
Superior glide of the proximal phalanx on the first metatarsal head.
Before performing this manipulation, the examiner performs deep-tissue
massage along the entire flexor hallucis brevis muscle. This is followed
with a series of hold-relax stretches in which the patient lightly presses
against the examiner's thumb (A). Upon releasing the pressure, the
examiner dorsiflexes the joint a few degrees, gently pushing the first
metatarsophalangeal joint into a more dorsiflexed position (B). This
process is repeated until bony resistance is noted. At this point, the
examiner performs a series of gentle mobilizations in which the distal
aspect of the first metatarsal head is driven inferiorly (C), while the
proximal phalanx is tractioned

138-Below knee amputation what you see


a- there is knee flexion
b- there is no knee flexion
c-there is edematous stump
D- there is deformed stump
The correct Answer is :
C- there is edematous stump

139-Syme's amputation also called


a- supramalleolar amputation
b- below knee amputation
c- above knee amputation

The correct Answer is :


A-supra-malleolar amputation .

140-Patient with prosthesis has excessive flxion durng gait due to:
A-rigid S.A.C.H
B-SOFT S.A.C.H
C-the socket too big

The correct Answer is :


A-Rigid S.A.C.H

141- This is

a) S.A.CH
b) Below knee cast
c) Above knee prosthesis with pelvic band

The correct Answer is :


C-Above knee prosthesis with pelvic band

142- Rigid plaster cast to


a) maintain shape of stump
b) A.F.Orthosis

The correct Answer is :


A-maintain shape of stump

143- phantom limb is


A- Painful stump
B- Feel of limb doesn’t exist

The correct Answer is :


B- feel of limb doesn`t exist called phantom sensation .

144- Phantom pain is:


A- Dull aching pain after surgery
B- Dull aching of the limb not more exist
The correct Answer is :
B-Dull aching of the limb not more exist is called phantom pain ; but Dull
aching pain after surgery called Post-operative pain.

145-patient with left above knee amputation complains of phantom limb


pain. Simply that means
a- Dull aching pain at the thigh of the diseased side
b- di-stressing pain sensation felt by patients in the limb that is no
longer there
c- Pain is described variously as cramping, squeezing, burning, sharp
and shooting

The correct Answer is :


B- di-stressing pain sensation felt by patients in the limb that is no
longer there called into phantom limb pain
a- dull aching pain at the thigh of the diseased side may called as
chronic pain.
C- pain is described variously as cramping , squeezing , burning , sharp
and shooting called into claudication pain .

146-Orthosis is
A- External device support limb
B- Field fabricate them
C- Person fabricate orthosis

The correct Answer is :


A- External device support limb called orthosis
B- field fabricate them called orthotic
c- person fabricate orthosis called othotist

147-Orthosis is
a- device that replace the absent part of the body
b- person who fabricates orthosis
c -an external device used in rehabilitation of patients with
neuromuscular and musculoskeletal disorders
d- company which fabricates the orthosis

The correct Answer is :


C- An external device used in rehabilitation of patients as we described
above .

148-prothetist:
A- Device that replace the amputated limb of the body
B - Material from which the device are fabricated
C - Person who fabricate and design the prosthesis
D- Company which fabricates the device.

The correct Answer is :


C- person who fabricate and design the prosthesis .

149-Pt has 32 years with spinal deformity & dyspnea due to :


a-Muscle weakness
b- Spinal orthosis
c- Decrease lung volume

The correct Answer is :


C-decrease lung volume is means dyspnea .

150-Above knee prothesis is long cause limb to do during gait :

a- vaulting
b- lat. bending
c- medial and lateral sweep
d- lordosis

The correct Answer is:


A-Vaulting

151-if the axis of artificial knee below the knee of non affected side
a-Hip hiking
b-Circumduction gait
c-Short step

The correct Answer is:


A-hip hiking is like vaulting as we described above

151-A physical therapist is beginning an evaluation of a patient with a


diagnosis of “knee strain”. Range of motion limitation does not follow
the normal capsular pattern of the knee. Which of the following are
possible causes of the restriction in range of motion?
a-Ligamentous adhesions
b-Internal derangement
c-Extra-articular lesions
d-All of the above

The correct Answer is:


D- all of the above

152-A child presents to physical therapy with a diagnosis of right


Sever’s disease. What joint should be the focus of the therapist’s
evaluation?
a-Right knee joint
b-Right hip joint
c-Right wrist joint
d-Right ankle joint

The correct Answer is:


D- Right ankle joint Sever's disease is a common cause of heel pain in
growing kids, especially those who are physically active

Gait Questions

154-In the terminal swing phase of gait, what muscles of the foot and
ankle are active?
a-Extensor digitorum longus
b-Gastrocnemius
c-Tibialis posterior
d-B and C

The correct Answer is:


A- extensor digitorum longus because it`s action is dorsiflexion but the
other both choices act as planter flexion , and in terminal swing phase
we put the foot in natural position or some little dorsiflexion

155-What motion takes place in the lumbar spine with right lower
extremity single limb support during the gait cycle?
a-Left lateral flexion
b-Right lateral flexion
c-Extension
d-Flexion

The correct Answer is:


B- Right lateral flexion
after search I found this review :
in swing phase of the gait there is some mechanical movement appear
in the spine; pelvis and arms .
the arms swing on the contralateral side of the leg movement.
the upper trunk make some forward motion (flexion) to increase the
cadence of gait.
the lumber vertebra moves with the pelvis rotation on the same side of
the unsupported leg .
so it could be B right latral flexion due to rotation of the pelvis toward
the left side

156-A physical therapist is ordered to provide gait training for an 18-


year-old girl who received a partial medial meniscectomy of the right
knee one day earlier.
The patient was independent in ambulation without an assistive device
before surgery and has no cognitive deficits. The patient’s weight
bearing status is currently partial weight bearing on the involved lower
extremity. Which of the following is the most appropriate assistive
device and gait pattern?

A-Crutches, three-point gait pattern


B-Standard walker, three-point gait pattern
C-Standard walker, four-point gait pattern
D-Crutches, swing-to gait pattern

The correct Answer is:


A- crutches ,three point gait pattern , The three-point gait pattern
involves the use of two crutches or a walker. This pattern is used when
the patient is permitted to bear weight through only one lower
extremity. three-point gait pattern requires good upper body strength,
good balance, and good cardiovascular endurance.

The pattern is initiated with the forward movement of one of the


assistive gait devices. Next, the involved lower extremity is advanced.
when the patient presses down on the assistive gait device and
advances the uninvolved lower extremity.

If the un-involved lower extremity is advanced to where it is


parallel to the involved lower extremity, then this would be a
"swing to" pattern.

If the un-involved lower extremity is advanced ahead of the involved


lower extremity, then this would be a "swing through" pattern.

157- A physical therapist is evaluating a patient with muscular


dystrophy. The patient seems to “waddle” when
she walks. She rolls the right hip forward when advancing the right
lower extremity and the left hip forward when advancing the left lower
extremity. Which of the following gait patterns is the patient
demonstrating?
A-Gluteus maximus gait
B-Dystrophic gait
C-Arthrogenic gait
D-Antalgic gait

The correct Answer is:


B-Dystrophic gait is waddling gait or latral trunk sway gait due to
weakness in both side of pelvis girdle.
158-A physical therapist is treating a 24-year-old woman with lumbar
muscle spasm. Part of the patient’s treatment involves receiving
instruction on correct sleeping positions. Which of the following would
be most comfortable?
A-Supine with no pillows under the head or extremities
B-Prone with a pillow under the head only.
CSidelying with a pillow between flexed knees

The correct Answer is:


C-sidelying with pillow between flexed knees , make the distance
between the lumber vertebrae more wider and more relaxed for the
muscle.

159-The therapist is treating a 52-year-old woman after right total hip


replacement. The patient complains of being self-conscious about a
limp. She carries a heavy briefcase to and from work every day. The
therapist notes a Trendelenburg gait during ambulation on level
surfaces. What advice can the therapist give the patient to minimize gait
deviation?
a-Carry the briefcase in the right hand
b-Carry the briefcase in the left hand
c-The patient should not carry a briefcase at all.
d-It does not matter in which hand the briefcase is carried.

The correct Answer is :


A- carry the briefcase in the right hand to increase the trunk lean toward
right side because the right side of the pelvis was raising as it`s muscle
weakness after surgery THS so after we know that the Hip girdle
muscles are responsible for keeping the pelvis level when walking. If
you have weakness on one side, this will lead to a drop in the pelvis on
the contralateral side of the pelvis while walking (Trendelenburg sign).

160-While gait training a patient following a cerebral vascular accident,


you observe the knee on the affected side going into recurvatum during
stance phase. The most likely cause of this deviation can be attributed
to:
A- Severe spasticity of the hamstrings or weakness of the
gastrocnemius- soleus.
B- weakness or severe spasticity of the quadriceps
C- Weakness of the gastrocnemius-soleus or spasticity of the pretibial
muscles
D- Weakness of both the gastrocnemius-soleus and pretibial muscles

The correct Answer is:


B- Weakness or severe spasticity of the quadriceps

161-flexion occur in any phase from this


a- mid stance
b- toe off
c- mid swing
d- heel strike

The correct Answer is:


C- mid swing

162-Which is not correct about hip during gait


a- max hip flexion occur in terminal swing
b- Slight hip extension before ic
c- Hip extension occur during double limb support

The correct Answer is:


A- max hip flexion occure in terminal swing .

163-C.O.G during locomotion observation can change the gait due to


a- c.o.g go upward and oblique to stance phase
b- downward during double limb support
c- side to side oscillation
d- move forward and backward

The correct Answer is:


D- move forward and backward

164-which statement correct during training with cane up and down


stairs
a- unaffected leg put first during upstairs
b- unaffected leg put down during up stairs
c- affected leg put first in down c-stairs

The correct Answer is:


A- unaffected leg put first during upstairs .
165-what is not a part of stance phase
1-deceleration
2-midstance
3-intial contact
4-preswing

The correct Answer is:


1- deceleration is a swing phase

166-pt ask to go upstairs with crutch


a) Sound limb go up 1
b) Affected limb go up 1

The correct Answer is:


a- sound limb go up first as we described above
The screw home mechanism that is present
in the last few degrees of terminal knee

167-Pt .move crutches first then Move legs to them


a- swing through
b- swing to
c- 2 gait point.

The correct Answer is:


B- swing to as we described above

168-When comparing the gait cycle of young adults to the gait cycle of
older adults, what would a therapist expect to find?
a- The younger population has a shorter step length
b- The younger population has a shorter stride length
c-The younger population has a shorter period of double support
d- The younger population has a decrease in speed of ambulation

The correct Answer is:


C- The younger population has a shorter period of double support “mid
stance “ than the older adult.

169-The therapist is ambulating a patient with an above knee


amputation. The new prosthesis causes the heel on the involved foot to
move laterally at toe-off. Which of the following is the most likely cause
of this deviation?
a-Too much internal rotation of the prosthetic knee.
b-Too much external rotation of the prosthetic knee.
c-Too much outset of the prosthetic foot.
d-None of the above would cause this deviation

The correct Answer is:


A- Too much internal rotation of the prosthetic knee lead to heel lateral
deviation.

170-A 68-years-old man is being treated by a physical therapist after a


right below-knee amputation. The patient is beginning ambulation with a
preparatory prosthesis. In the early stance phase of the involved
lower extremity, the therapist notes an increase in knee flexion. Which
of the following are possible causes of this gait deviation?
a- The heel is too stiff
b- The foot is set too far anterior in relation to the knee
c- The foot is set in too much plantar flexion
d- All of the above

The correct Answer is:


A- The heel is too stiff lead to excessive flexion and this is the rest of
reasons which may lead to the same deviation.
Socket set forward in relation to foot, Foot set in excessive dorsiflexion
and if the Prosthesis too long.

171-During evaluation of a patient, the therapist observes significant


posterior trunk lean at initial contact (heel strike). Which of the
following is the most likely muscle that the therapist needs to focus
on during the exercise session in order to minimize this gait deviation?
a- Gluteus medius
b- Gluteus Maximus
c- Quadriceps
d- Hamstrings

The correct Answer is:


B-Gluteus Maximus is a responsible for balancing and prevent posterior
trunk lean , Gluteus medius is a responsible for balancing and prevent
side trunk leaning.

172-Quadriceps function in swing phase


a- accelerated knee flexion
b- prevent excessive knee flexion

The correct Answer is:


B- prevent excessive knee flexion

PNF

1- D1 flexion diagonal prepare pt for gait


2- D2 flexion parkinsonism
3- D2 flexion rotator cuff, impingement, supra-spinatous tendinitis
4- D2extention pitcher
5- Rhythmic initiation parkinsonism
6- Rhythmic stabilization increase stability ( Ataxia )
** Rhythmic initiation progression from agonist pattern (assistive -
active free -resistive ) Uses: -in ability to initiate movement -limited ROM
** Rhythmic stabilization isometric contraction of agonist following by
isometric contraction of antagonist Uses :-increase strength and
endurance

173-A pitcher is exercising in a clinic with a sports cord mounted behind


and above his head. The pitcher simulates pitching motion using the
sports cord as resistance. Which (PNF) diagonal is the pitcher using to
strengthen the muscles involved in pitching a baseball?
A. D1 flexion
B. D1 flexion
C. D2 extension
D. D2 flexion.

The correct Answer is:


D- D2 extension.

174-To treat effectively most patients with Parkinson’s disease, the


therapist should emphasize which proprioceptive neuromuscular
facilitation (PNF) pattern for the upper extremities?
a-D2 extension
b-D2 flexion
c-D1 extension
d-D1 flexion

The correct Answer is:


B- D2 flexion
175-A therapist receives an order to evaluate a 72-year-old woman who
has suffered a recent stroke. The therapist needs to focus on pre gait
activities. Which of the proprioceptive neuromuscular facilitation(PNF)
diagonals best encourages normal gait?
a- D1flex
b- D2 flex
c- PNF is contraindicated
d- Pelvic PNF patterns only

The correct Answer is:


B- D1 flexion diagonal for preparing for Gait

176-Resistance in P.N.f for:


A. Facilitate and stimulate ms to contract
B. To increase motor contraction and motor learning
C. To make patient know direction of movement
d- all of the above

The correct Answer is:


D- all of the above is benefit of Resistive PNF .

178-All of the following in the gait cycle except


A- Swing phase
B- Mid swing
C- Initial contact
D- Heal flat

The correct Answer is:


D- Heal flat

179-As a result of a gait analysis, a therapist has determined that the


patient ambulates with excessive foot pronation. This deviation would
not occur as a result of:
a- compensated forefoot varus
b- internal tibial rotation
c- a weak tibialis posterior
d-excessive ankle dorsiflexion

The correct Answer is:


D- excessive ankle dorsiflexion isn`t cause excessive foot pronation ,
Compensated forefoot varus , internal tibial rotation , a weak tibialis
posterior make an excessive foot pronation.

180-use of can in contralateral hand will :


a- not affected gait pattern
b- reduce energy expenders in normal person
c-decrease the floor reaction force on the opposite foot
d- not affected floor reaction force on the opposite foot

The correct Answer is:


C- will decrease the floor reaction force on the opposite foot .

181-elderly person 65 year of age during his gait


a- swing phase more than 40%
b- swing phase less than 40%
c- swing phase
D2 flex U.L
D2 flex L.L

The correct Answer is:


B- swing phase less than 40 % to avoid falling and imbalance. , normal
gait in adult , 60 % stance phase to increase balance & stability and 40
% swing phase to avoid falling and imbalance

182-Pt has calf muscle tear type of massage used


A. taptman
B. vibration
C. effleurage
D. deep friction

The correct Answer is:


C- Effleurage massage

183-A physician has ordered a physical therapist to treat a patient with


chronic low back pain. The order is to “increase gluteal muscle function
by decreasing trigger points in the quadratus lumborum”. What is the
first technique that should be used by the physical therapist?
A-Isometric gluteal strengthening
B-Posture program
C-Soft tissue massage
The correct Answer is:
C- soft tissue massage for trigger point.

184-in acute of lymphedema


a-Begin with proximal part then distal
b-Begin with distal part then proximal
c- Superficial effleurage

The correct Answer is:


C- Superficial effleurage

Muscle Test Questions

185- Which method used in the measurement of shoulder abduction:


a-tape measurement
b-goniometer
c-visual analogue scale

The correct Answer is:


B-goniometer

186-During a treatment session in a rehabilitation hospital a patient


asks the physical therapist assistant to let him see the physical therapy
progress notes in his medical record. The physical therapist assistant
should.
A- contact the patient’s physician and explain the situation.
B- refuse to let the patient see the record.
C- allow the patient to see the notes.
D- get the permission of the physical therapist supervisor before giving
the patient the chart.

The correct Answer is:


D- get the permission of the physical therapist supervisor before giving
the patient the chart.

187- All of the following are objective measurements used in evaluation


except:
a- time used for performance activity
b- visual analogue scale
c- goniometric ROM
d- disability devices

The correct Answer is:


D- disability devices not objective measurements

188-A therapist is instructing a physical therapy student in writing a


SOAP note. The student has misplaced the following phrase: Patient
reports a functional goal of returning to playing baseball in 5 weeks.
Where should this phrase be placed in a SOAP note?
a-Subjective
b-Objective
c-Assessment
d-Plan

The correct Answer is:


A – Subjective

189- Which muscle bilateral weakness patient use bilateral crutches


a- Gluteus medius
b- Gluteus maximus
c- Quadriceps
a- Hamstrings

The correct Answer is:


B-Gluteus Maximus

190-To help decrease shear when transferring a patient with a spinal


cord injury from bed to chair, it would be best to use
A - a draw sheet.
B - Skin lubricant on involved skin surfaces.
c - An air mattress.
D – Sheep skin pad

The correct Answer is:


A – draw sheet

191-The therapist is assessing a patient’s strength in the right shoulder.


The patient has zero degree of active shoulder Abduction in the standing
position. In the supine position, the patient has 42 degree of active
shoulder abduction and 175 degree pain-free passive
shoulder abduction. What is the correct manual muscle testing grade for
the patient’s shoulder abduction?
a-3-/5 (fair -)
b-2+/5 (poor+)
c-2-/5 (poor-)
d-1/5 (trace)
The correct Answer is:
C- 2-/5 (poor ) , poor grade The ability to move through a partial arc of
motion in the horizontal plane is graded as poor - The grade of poor
means that the muscle is capable of completing the range of motion in
the horizontal plane.
The grade of poor+ denotes the ability to move in the horizontal plane to
completion of the range of motion against resistance or to hold the
completed position against pressure. It also means that the muscle is
capable of moving through a partial arc of motion in the antigravity
position.

192-While evaluating a patient who suffered a complete spinal cord


lesion, the therapist notes the following strength grades with manual
muscle testing: wrist extensors = 3+/5, elbow extensors = 2+/5, and
intrinsic muscles of the hand = 0/5. What is the highest possible level of
this lesion?
B. C3
C. C4
D. C7

The correct Answer is:


D- C7 level of lesion

193-A and C are equally correct A patient is referred to physical therapy


with a history of temporamandibular joint pain. The therapist notices
that the patient is having difficulty closing his mouth against minimal
resistance. With this information, which of the following muscles would
not be a target for strengthening exercise to correct
this deficit?
A-Medial pterygoid muscle
B-Temporalis
C-Masseter
D-Lateral pterygoid muscle

The correct Answer is:


D- lateral pterygoid

muscle
Lateral pterygoid muscle open jaw and protrude the mandibular and
move the mandibular from side to side .

194--A patient is in prone position with his head rotated to the left side.
The left upper extremity is placed at his side and fully internally rotated.
The left shoulder is then shrugged toward the chin. The therapist then
grasps the mid shaft of the patient’s left forearm. The patient is then
instructed to “try to reach your feet using just your left arm.” This
movement is resisted by the therapist. The test is assessing the
strength of what muscle?
A. Upper trapezius
B. Posterior deltoid
C. Latissimus dorsi
D. Triceps brachialis

The correct Answer is :


C- Latissimus dorsi action is extension of the arm
195-A patient is positioned in the supine position. The involved left
upper extremity is positioned by the therapist in 90o of shoulder flexion.
The therapist applies resistance into shoulder flexion, then extension.
No movement takes place. The therapist instructs the patient to “hold”
when resistance is applied in both directions. Which of the following
proprioceptive neuromuscular facilitation techniques is being used?
a-Repeated contractions
B- Hold-relax
C –Rhyt hmic stabilization
D-Contract-relax

The correct Answer is:


C- Rhythmic stabilization

Hold-relax (HR)-autogenic inhibition: a relaxation technique usually


performed at the point of limited range of motion in the
agonist pattern:
• An isometric contraction of the range-limiting antagonist is
performed against slowly increasing resistance.
• This is followed by a voluntaly relaxation by the patient and
then passive movement of the extremity by the clinician into
the newly gained range of the agonist pattern.

Hold-relax-active (HRA)-reciprocal inhibition: folloWing


application of the hold-relax technique, the patient performs an
active contraction into the newly gained range of the agonist pattern.

Contract-relax (C-R): a relaxation technique usually performed at the


point of limited range of motion in the agonisl:
pattern:
• An isotonic movement in rotation is performed followed by an
isometric hold of the range-limiting muscles in the antagonist
pattern against slowly increasing resistance, voluntary relaxation, and
active movement into the new range of the agonist pattern.
196-A physician prescribes isotonic exercises for the left biceps
brachialis. Which of the following exercises is in compliance with this
order?

A-Biceps curls with the patient actively and independently flexing the
left elbow using a 5-pound dumbbell as resistance
B-Rhythmic stabilization for the left elbow.
C- Elbow flexion at 90o per second with speed controlled by a work
stimulator.
d- None of the above.

The correct Answer is:


A- Biceps curls with the patient actively and independently flexing the
left elbow using a 5-pound dumbbell as resistance

197-The therapist is performing an orthopedic test on a 25-year-lld man


with the chief complaint of low back pain. The patient has a positive
Thomas test. With this information, what might the therapist need to
include in the treatment plan?
A- Stretching of the hip abductors
b- Stretching of the hip adductors
c- Stretching of the hip extensors
d-Stretching of the hip flexors

The correct answer is:


D- stretching of the hip flexors because Thomas test make for hip
flexion

198- stretching for which muscle


a)lateral rotator
b)adductor
c)hamstring
d)calf muscle
The correct Answer is:
A) External rotators (pireform muscle )

199-During an evaluation, the therapist taps on the flexor retinaculum of


the patient’s wrist, which causes tingling in the thumb. What test is
this? For what condition does it screen?
A-Phalen’s test – carpal tunnel
B-Finkelstein test – de Quervain’s disease
C-Tinel’s sign – de Quervain’s disease
D-Tinel’s sign – carpal tunnel

The correct Answer is:


D- Tinel`s sign – Carpal tunnel.

200-A 27-year-old woman is referred to a physical therapy clinic with a


diagnosis of torticollis. The right sternocleidomastoid is involved. What
is the most likely position of the patient’s cervical spine?
a-Right lateral cervical flexion and left cervical rotation
b-Right cervical rotation and right lateral cervical flexion
c-Left cervical rotation and left lateral cervical flexion
d-Left lateral cervical flexion and right cervical rotation.

The correct Answer is:


A-Right lateral cervical rotation And left cervical rotation as it`s
sternocleidomastoid action.

201- A therapist assessing radial deviation range of motion at the wrist.


The correct position of the goniometer should be as follows: the
proximal arm aligned with the forearm and the distal arm aligned with
the third metacarpal. What should be used as the axis point?
a- Lunate
b- Scaphoid
c- Capitate
d- Triquetrum

The correct Answer is :


C-cpitate in the middle of hand

202-A therapist is instructing a patient in the use of a wrist-driven


prehension orthotic. What must be done to achieve opening of the
involved hand?
A-Actively extend the wrist
B-Passively flex the wrist
C-Actively flex the wrist
D-Passively flex the wrist

The correct Answer is:


D- passively flexion the wrist

203-PT is assessing a patient’s ability to perform basic activities of daily


living. The assessment tool measures bathing, toileting, dressing,
transfers, continence and feeding. The tool does not assess the
patient’s ability to maneuver in a wheel chair The therapist is using
which of the following tests?
A. Barthel Index
C. Kenny Self-Care Evaluation
B. Katz Index of Activities of Daily Living
D. Functional Status Index

The correct Answer is:


C-The Index of Independence in Activities of Daily Living is based on an
evaluation of the functional independence or dependence of patients in
bathing, dressing, toileting, transferring, continence, and feeding.
Scoring :
A – Independent in feeding, continence, transferring, toileting,
dressing, and bathing
B – Independent in all but one of these functions
C –Independent in all but bathing and one additional function
D – Independent in all but bathing, dressing, and one additional function
E – Independent in all but bathing, dressing, toileting, and one additional
function
F – Independent in all but bathing, dressing, toileting, transferring, and
one additional function
G – Dependent in all six functions

The Kenny Self Care Evaluation is a 17-item, multidimensional measure


that assesses self-care abilities (dependence and independence) in six
categories: bed activities, transfers, locomotion, dressing, personal
hygiene and feeding.

Functional status is an individual's ability to perform normal daily


activities required to meet basic needs, fulfill usual roles, and maintain
health and well-being

The Barthel scale or Barthel ADL index is an ordinal scale used to


measure performance in activities of daily living (ADL). Each
performance item is rated on this scale with a given number of points
assigned to each level or ranking.

204-long measurement of lower limb from :


a- ASIS to medial malleolus
b- ASIS to lateral malleolus
c- greater trochanter to lateral malleolus

The correct Answer is :


A-ASIS to medial malleolus

205-The most appropriate position to strength sternocleidomastoid is


a- supine
b- sitting
c- prone
d- standing

The correct Answer is:


B – sitting is the best postion for strength sternocleidomastoid

206- which muscle close the mouth :


a- masseter
b- lateral ptyrgoid
c- buccinators

The correct Answer is:


A- masseter - Chewing muscle

207-Patient with facial palsy in LT side he cannot make horizontal Lt


lateral side by eye which ms affected
1- LT latralies
2- RT latralies
3- Rt mediales
4- 1 and 3 both right answers

The correct Answer is:


D- 1&3 both right answer because the 7th cranial nerve doesn’t affect on
muscle of the eye , so it should be there is affected 2 muscles , if one
like an 1st choice it will be the 6th cranial nerve.

208- which is not correct about oxford scale?


a- 0 no contraction
b- flicker contraction
c-4 can make full ROM against resistance

The correct Answer is:


C- 4th degree makes full ROM against resistance Manual Muscle Testing
Grading System

Grading Scale Range: 0 to 5


No visible or palpable
0
None contraction
Visible or palpable contraction
1
Trace with no motion ( a 1 )
2 Poor Full ROM gravity eliminated
3 Fair Full ROM against gravity
Full ROM against gravity,
4
Good moderate resistance
Full ROM against gravity,
5
Normal maximul resistance

The Oxford classification


0 = no contraction at all
1 = flicker of contraction only, movement of the joint does not occur
2 = movement is possible only with gravity counterbalanced
3 = movement against gravity is possible
4 = movement against resistance is possible
5 = normal functional movement is possible.

209-muscle that moves angle of mouth inferior and back


A- Platysma
B -Mentalis
c- Masseter

The correct Answer is:


A- platysma , moves angel of the mouse inferior and back

210- which of the following Closes the jaw:


a- masseter
b- levator lipii
c- mentalis
d- bicularis oris

The correct Answer is:


A-Masseter as we described above .

211- which muscle Raise the skin of the chin.:


a- buccinators
b- levator libii
c- mentalis
d – platysma

The correct Answer is:


A-mentalis we use it for shaving our faces ;)

212- which muscle Close and protrude the lips, as in whistling:


a- masseter
b- mentalis
c-orbicularis oris
d- depressor libii

The correct Answer is:


C- orbicularis oris

213-which muscle close the eyes?


A)buccinators
B)orbicularis oris
C)orbicularis occuli
D)masseter
The correct Answer is:
C- Orbicularis occuli

214- when evaluation of hip abduction of lt hip ; it was poor grade ; so :


a- can apply full ROM of abduction in side lying
b- can apply full ROM of abduction in supine
c- can apply half ROM of abduction in supine
d- can apply full ROM of abduction with

The correct Answer is:


B- can apply Full Rom of abduction in supine .

215-when you examine a muscle with no evidence of contraction,


which muscle grade would you give:
a- Poor
b- Trace
c- Good
d-zero

The correct Answer is:


D- Zero

216-which of the following Draws the eyebrow down ward and inward,
with vertical wrinkles:
a- buccinators
b- corregator
c- frontalis
d- depressor angulii

The correct Answer is:


B- Corregator for frown

217- Contraction and lastly resistive movements it’s


A- Rhythmic initiation
B- Rhythmic stabilization
C- Contract-relax
D- Hold-relax

The correct Answer is:


A-Rhythmic initiation: Progression from( agonist pattern) PASSIVE to
ACTIVE ASSISTED to ACTIVE resisted USED IN• Limited ROM due to
increase tone• Who are unable to initiate movement

218-A 35-year-old woman with a diagnosis of lumbar strain has a


physician’s prescription with a frequency and duration of 3 sessions/
week for 6 weeks. The physical therapy evaluation reveals
radiculopathy into the L5 dermatome of the right lower extremity,
increased radiculopathy with lumbar flexion, decreased radiculopathy
with lumbar extension, poor posture, and hamstring tightness bilaterally
at 60 . What is the best course of treatment?

a- Lumbar traction , hot packs, and ultrasound.


b- McKenzie style lumbar extensions, a posture program, hamstring
stretching, and a home exercise program.
c- McKenzie style lumbar extensions, a posture program, hamstring
stretching, home program, hot packs, and ultrasound.
d- Lumbar traction, hot packs, ultrasound, and hamstring stretching.

The correct Answer is:


C- McKenzie Style lumber extensions . a posture program, hamstring
stretching, and a home exercise program , hot packs , and Ultra sound .
219-A 42-year old receptionist presents to an outpatient physical
therapy clinic complaining of low back pain. The therapist decides that
postural modification needs to be part of the treatment plan .What is the
best position for the lower extremities while the patient is sitting?

a- 90o of hip flexion, 90o of knee flexion, and 10o of dorsiflexion


b- 60o of hip flexion, 90o of knee flexion, and 0o of dorsiflexion
c- 110o of hip flexion, 80o of knee flexion, and 10o of dorsiflexion
d- 90o of hip flexion, 90o of knee flexion, and 0 of dorsiflexion

The correct Answer is:


D- 90 hip flexion , 90 knee flexion .and 0 dorsiflexion .

220-from supine when there is shortening of this muscle the arm raise
above table and not down ,the MS is
a- biceps
b- brachioradialis
c- brachialis
d- pectoralis major

The correct Answer is:


D- pectoralis Major because it`s adductor muscle and the others flexors
of the elbow.

221-PT is evaluating patient who is unable to open the door using


supination ; so he expects
weakness of
a- brachioradialis
b- biceps brachii
c- anconeus
d- flexor carpi ulnaris

The correct Answer is:


B-biceps brachii make a flexion with supination
Flex Supination biceps
Flex Mid position brachialis
Flex prnation brachioradialis

222-physical therapist is evaluating elbow flexion from neutral position ;


which muscle should be evaluated ?
a- biceps brachii
b- brachialis
c- brachioradialis
D- anconeous

The correct Answer is:


B-brachialis as we described above

223-patient have above knee amputation , contracture in hip flexors


what is best position for this patient
1-Prone with pillows under hip
2 Half prone
3-Sitting with leg on pillow
4 -Side lying

The correct Answer is:


A- prone with pillows under hip to stretch the hip flexors

224-Case of above knee amputation with neglectable hip flexor


contracture refer to pt the best way to
A-passive stretch ex
B-active stretch ex
C-strength of agonist

The correct Answer is :


A- passive stretch exe, he can`t do it actively because he has already
the contracture .

225-some tests performs to detect ligament& capsule stability in


shoulder joint when positioned in abduction lateral rotation with
continued anterior propagation of humeral head this due defect in
ligament in
A superior part
B inferior part
C anterior part
The correct Answer is :
A-anterior part

226-A patient presents to an outpatient clinic with complaints of


shoulder pain. The therapist observes a painful arc between 70o and
120o of active abduction in the involved shoulder. This finding is most
indicative of what shoulder pathology?
a-Rotator cuff tear
b-Acromio - clavicular joint separation
c-Impingement
d-Labrum tear
The correct Answer is :
B- Acromio- clavicular joint movement form

227-A 67-year-old woman presents to an outpatient facility with a


diagnosis of right adhesive capsulitis. The therapist plans to focus
mostly on gaining abduction range of motion. In which direction should
the therapist mobilize the shoulder to gain abduction range of motion?
a-Posteriorly
b-Anteriorly
cInferiorly
d-Superiorly

The correct Answer is :


C- inferiorly for abduction

228-A therapist places a pen in front of a patient and asks him to pick it
up and hold it as he normally would to write. The patient picks the pen
up and holds it between the pad of the thumb and the middle and index
fingers. Why type of grasp or prehension is the patient using?
A-Palmar prehension
B-Fingertip prehension
C-Lateral prehension
D-Hook grasp

The correct Answer is:


A- Palmar Prehension: Gripping is done by two or more fingers opposed
to the pad of the thumb. Holding on to a pencil to write is an example of
palmar prehension. Palmar prehension is a combination of tip-to-tip and
lateral prehensions
Spherical Grip: Used for holding ball-shaped objects. For small objects
(say, a tennis ball) the fingers splay out and curl around enough to press
the object against the palm. In larger objects (a softball), the fingers
press the object against the fingers or thumb on the other side. For very
large objects (a basketball), this is as much friction as it is
actual grip, and may be broken easily.

Lateral Grip: You might use this grip to hold on to a tree trunk. It
involves the fingers pressing in unison against the palm, with the object
in between. This is what's commonly thought of as the power grip.

Hook Grip: This grip doesn't involve the palm or thumb at all. It requires
the fingers to be curled around the object, and the object to be pressed
against the joints between the fingers and the hand. This grip is used to
hold on to things like monkey bars or a water-ski line.

Cylindrical Grip: Much like the hook grip, but uses the palm and thumb
for added stability. This grip is used for holding things like baseball bats.

Precision grips are used when fine manipulation is desired, manipulation


that will probably be done by slight movements of the fingers and wrist.
They grip by opposing the fingers and thumb against one another in
various ways. Precision grips come in three flavors:

Tip-to-tip Prehension : Also known as the pinch grip. Accomplished by


opposing the index finger and thumb to hold on to something very small
or delicate . With the range of movement left open to the fingers and
wrist while using this prehension, the object can be moved in essentially
any direction.

Lateral Prehension: Unlike the lateral power grip, this grip is between
the thumb and the side of a finger, or between the sides of two fingers.
It is used when the wrist is doing the primary movement. Turning a key
in a lock is a use of lateral prehension.

229-selfactuallization mean
a-person frustrated because can not gain target
b-sense of blessing due to age
c- person has gain something and proued
d-no sense

The correct Answer is:


C-person has gain something and proud

230-A 17 year-old gymnast has been diagnosed with an unstable


spondylolisthesis involving L5 on S1. Appropriate management of this
problem by a physical therapist should include:
A-back extension exercises.
B- abdominal strengthening and flexion exercises.
C- joint mobilization emphasizing lumbar rotation in both supine and side
lying.
D- lumbar extension mobilization using hand pressure directed anteriorly

The correct Answer is:


B- abdominal strengthening and flexion exercise .

231-patient with posterior lumbar disc herniation by physiotherapist


apply mechanical lumbar traction put patient on
a-supine with towel under back
b-supine with flexed knee
c-prone with towel under abdomen
d-prone lying

The correct Answer is:


B-supine with flexed knee

232- Patient suffering from morning stiffness this is a sign of :


a- swelling of the body tissues
b- prolonged stiffness with inflammation.
c- Muscle inflammation.
d- Bonny projection at the joint

The correct Answer is:


B- Prolonged stiffness with inflammation.

223-Patient is suffering from morning stiffness; the most likely


developing condition is due to:
1-Bones are growing beyond their margins
2- Fasciitis (Refers to an inflammation of the fascia )
3- Accumulation of lactic acid
4- Muscle Spasm

The Correct Answer is:


B-Fasciitis (Refers to an inflammation of the fascia )

234- A 14-year old girl place excessive valgus stress to the right elbow
during a fall from a bicycle. Her forearm was in supination at the
moment the valgus stress was applied. Which of the following is most
likely involved in this type of injury?
a- Ulnar nerve
b-Extensor carpi radialis
c-Brachioradialis
d-Annular ligament

The Correct Answer is:


A - Ulnar Nerve, cubital valgus fracture means move the forearm away
from the humreus which causes fructure of medial epicondyle Fracture
causes ulnar nerve injury.

Supracondylar --> median nerve injury


*Mid humerus --> radial nerve injury
*Upper humerus --> brachial nerve injury
*The cause of cubital varus fracture of --> supracondylar Fracture

235- joint play R O M which occurs not named with


a- necessary to completion of joints ROM
b- Passively occur
c- under voluntary control

The Correct Answer is:


C- Under voluntary control

236-Streno clavicular inforced by except


A- streno clavicular lig
B-intra articular disc
C-intra calvicular lig
D- acriomo calviclar lig

The Correct Answer is:


D- Acromio claviclar lig.

237-shoulder dislocation head humerus fallen


A posterior
B inferior
C anterior

The Correct Answer is:


C- Anterior

238-volkmans ischemic disease most dependable sign for this patient is:
a-Pain
b-cyanosed color of hand
c-weak extensors
d-weak flexion

The Correct Answer is:


B-cyanosed color of hand

239-what not true about curvature of cervical region


A-cervical curvature lordosis increases preesure on disc
B- cervical curvature lordosis increases pressure on lig
C- cervical curvature lordosis increases pressure on facet jiont
D-Lumber and thoracic disorders not accompiened with cervical
curvature

The Correct Answer is:


D- lumber and thoracic disorders not accompined with cervical
curvature

240-what is not correct about bone


a-contain 90% of inorganic tissues
b-compose of 2 types sponge, compact
c-bone is hard
d-has blood supply , nerve endings

The Correct Answer is:


A- contain 90 % of inorganic tissue

241- Ankylosing means :


a- loosed packed position
b- joint stiffness
c – closed packed position
The Correct Answer is:
B- joint stiffness , ankylosing means is a stiffness of a joint due to
abnormal adhesion and rigidity of the bones of the joint

242-pt complain of groin pain with limited abduction with external


rotation you expect the diagnosis is
a-inguinal hernia
b- adductor strain
c- abductor sprain
d- external obliques weakness

The Correct Answer is:


B-Adductor strain causes limited abd with Ext. Rotation

243-in clinical examination you must do all of the following except

a-observe the patient examine both affected unaffected


b-review patient sheet
c-focus only on areas of patient`s complains
d-respect the sensitizer’s of patients

The Correct Answer is:


C- Focus only on areas of patient`s complains.

244-fixed stiffness of 1st IPJ joint of big toe called


A- Hallux varus
B- Hallux valgus
C- Hammer toe
D- Hallux rigidus

The Correct Answer is:


D- Hallux rigidus: loss of motion of first MTP joint in adults due to
degenerative arthritis osteophyte formation leads to dorsal impingement
Hallux Valgus: is lateral deviation of the great toe so as to put a valgus
deformity on the first metatarsophalangeal (MTP) joint. A deviation of
15-20° is considered abnormal.
Hallux varus : deviation of the distal portion of the great toe at the
metatarsophalangeal joint to the inner side of the foot away from the
second toe.
Hammer Toe : flexion of the PIP joint extension deformity at DIP
deformity can be rigid or flexible

245-Which nerve supplies the muscle in the picture


a. Axillary
b. Supascapular
c. Long thoracic
d. Musculocutaneous

The correct Answer is :


A- Axillary nerve supplies the teres minor and deltoid muscle, abductors
of the arm at the shoulder.

246-A patient presents to an outpatient physical therapy clinic with a


severed ulnar nerve in the upper extremity. What muscle is still active
and largely responsible for the obvious hyperextension at the
metacarpophalangeal (MCP) joints of the involved hand?
A. Dorsal interossei
B. Volar interossei
C. Extensor carpi radialis brevis
D. Extensor digitorum

The correct Answer is:


C- Exensor digitorum

247- Knee joint ROM:


a- flexion 110 – extension 0
b- flexion 130 – extension 0
c- flexion 140 – extension 0

The correct Answer is:


C-flexion 140 – extension 0

248- This angle is :


a) k angle
b) Q angle
c) C angle

The correct Answer is:


C-Q- Angle

248-Observing a patient in a standing position, the therapist notes that


an angulation deformity of the right knee causes it to be located
medially in relation to the left hip and foot. This condition is commonly
referred to as:
a-genu varum
b-genu valgum
c-pes cavus
d-none of the above.

The correct Answer is:


B- Genu Valgum commonly called "knock-knee", is a condition in which
the knees angle in and touch one another when the legs are
straightened.
Genu Varum “Bowwed Leg “is an exaggerated bending outward of the
legs from the knees down that causes the knees to be spread apart
when the feet

249-Inversion, plantar flexion, adduction A 10-year-old boy presents to


outpatient physical therapy with complaints of diffuse pain in the right
hip, thigh, and knee joint. The patient was involved in a motor vehicle
accident 3 weeks ago. He is also obese and has significant atrophy in
the right quadriceps. The right lower extremity is held by the patient in
the position of flexion, abduction and lateral rotation. Which of the
following is most likely the source of the patient’s signs and symptoms?
a-Greater trochanteric bursitis
b-Avascular necrosis
c-Slipped femoral capital epiphysis
d-Septic arthritis

The correct Answer is:


C- slipped femoral capital epiphysis is most common in the adolescent
period (boys aged 10-16 y, girls aged 12-14 y). Males have 2.4 times the
risk compared with females, Obesity is a risk factor because it places
more shear forces around the proximal growth plate in the hip at risk ,
Prodromal symptoms (eg, hip or knee pain, limp, decreased range of
motion) , Patients often hold their affected hip in passive external
rotation. , gait pattern Antalgic – Shortened stance phase on the
affected side ,Out-toeing.

250- Normal ROM of hip joint :


a- extension 35 --- flexion 125
b- extension 10 ---- flexion 125
c- extension 0 -----flexion 125

The correct Answer is :


B- extension 10 , flexion 125

251-Patient with disc herniation and decrease back motion P.T ASSIST:
A- sterno-costal angle
b-lumbosacral angle
c-cobbs angle
d-cubitus angle

The correct Answer is:


B-Lumbosacral Angle
252- While assessing the standing posture of a patient, the therapist
notes that a spinous process in the thoracic region is shifted laterally.
The therapist estimates that T2 is the involved vertebra because he or
she notes that it is at the approximate level of the:
a- Inferior angle of the scapula
b- Superior angle of the scapula
c- Spine of the scapula
d- Xiphoid process of the sternum

The correct Answer is:


B- superior Angel of the scapula T2 , Spine of the scapula T3 , Inferior
angle of the scapula T7

253-In thoracic outlet syndrome is:


a- superior to operative thoracic
b- posterior to operative thoracic
c- inferior to operative thoracic
d- anterior to operative thoracic

The correct Answer is:


B- superior to operative thoracic
Thoracic outlet syndrome is a syndrome involving compression at the
superior thoracic outlet involving compression of a neurovascular
bundle passing between the anterior scalene and middle scalene.
[2] It can affect the brachial plexus (nerves that pass into the arms from
the neck), and/or the subclavian artery or vein (blood vessels that pass
between the chest and upper extremity).

254-The thoracic outlet syndrome, is due to :


a- Upper opening chest
b- Lower opening chest
c- Lateral opening chest
d- Anterior opening che

The correct Answer is:


A- Upper opening Chest as we described above.

255-Pt walk with extended back during bearing on leg & difficulty go up
ramp this your plain is to
A- stretching hip ex
B- Strengthening hip ex
C- Strengthening knee flexor
D- Stretching knee flexor

The correct Answer is:


A-stretching hip ex

256- which muscle make hip abduction ;flexion ;internal rotation :


a- gluteus medius
b- external oblique’s
c- Sartorius
d-tensor fascia lata

The correct Answer is:


D- Tensor Fascia lata Actions: Hip - flexion, medial rotation, abduction
knee - lateral rotation, Torso - stabilization

Sartorius Actions: At the hip joint, it is a flexor, abductor and lateral


rotator. At the knee joint, it is also a flexors

Gluteus Medius Action: abduction of the hip; preventing adduction of the


hip. Medial rotation of thigh.

257- which muscle make shoulder ;flexion ;external rotation”:


a- subscapularis
b- teres major
c-teres minor
d- pectoralis major

The Correct Answer is:


C- Teres minor action is: laterally rotates the arm, stabilizes humerus
Teres Major Actions: Adducts at the shoulder and medially rotates the
arm
Subscapularis muscle: Medially rotates the arm.
Pectoralis Major: Clavicular head:
a. Flexes the arm at the shoulder (glenohumeral) joint.
Sternal head:
a. Extends the arm at the shoulder (glenohumeral) joint from a flexed
position.
Both heads working together:
a. Medially rotate the arm at the shoulder (glenohumeral) joint.
b. Adduct the arm at the shoulder (glenohumeral) joint

258-during ex there is subtalar pronation that’s mean


a-femur, tibia, pelvic internal rotation
b-femur, tibia, pelvic ext rot
C-femur ,tibia int rot with pelvic ext rot
d-femur ,tibia internal rot with pelvic int rot

The correct Answer is:


A& D the same meaning and the same correct Answer .

259-Kumar, a 31 years old motorcyclist sustained injury over his right


hip joint. X-ray revealed a posterior dislocation of the right hip joint. The
clinical attitude of the affected lower limb will be:
A. External rotation, extension & abduction
B. Internal rotation, flexion & adduction
C. Internal rotation, extension & abduction
D. External rotation, flexion & abduction

The correct Answer is:


B-Internal Rotation with Flexion and adduction is the clinical attitude of
the posterior dislocation.

260-which muscle act in Med rot, adduction and extension of shoulder


joint:
a- pectoralis minor
b- serratus anterior
c- teres minor
d- latissmus dorsi

The correct Answer is:


D- Latissmus Dorsai make a medial rotation with Extension and
Adduction of the shoulder
261-when u pushing automobile with shoulder flexion, elbow extended
the main muscle of action is:
a-serratus anterior
b-biceps
c-pectorals major
d-triceps

The correct Answer is:


A-serratus Anterior action is stabilize the shoulder and protract the
scapula that`s mean stabilize the arm in push anything.

262- To strengthen this muscle, you should:

a. Maintain dorsiflexion and walking some steps on heel


b. Walking on toes
c. Sit at edge of plinth, rest foot on ground and fully dorsiflexion the foot

The correct Answer is:


A- Maintain dorsiflexion and walking some steps on heel

263-Which three muscles attach to the first cuneiform bone?


a- The anterior tibialis, the posterior tibialis, and the fibularis (peroneus)
longus
b- The extensor digitorum, the flexor hallucis, and the fibularis
(peroneus) longus
c- The anterior tibialis, the peroneus brevis, and the fibularis (peroneus)
longus
d- The flexor carpi ulnaris, the flexor pollicis brevis, and the flexor
pollicis longus

The correct Answer is:


A- The anterior tibialis, The posterior Tibialis, and the fibularis (proneus)
longus
264-In which location is the median nerve frequently or mostly
entrapped
a. Forearm
b. Arm
c. Elbow
d. Wrist

The correct Answer is:


D- Wrist

265– pt. has median Nerve compression assist him by


a) Weak of medial 2 fingers & decrease grasping objects
b) Numbness of 3 ½ fingers & weak opponents MS

The correct Answer is:


B- Numbness of 3 ½ fingers & weak opponents MS

266-ulnar nerve give supply to


a- whole hand
b- outer 3 +1/2 fingers
c- inner 1+1/2 fingers

The correct Answer is:


C- Inner 1+ ½ fingers

267-What muscle make lateral rotation of Humeral:


a-Pectoral major
b-Teres minor
c-Teres major
d-Rotator cuff

The correct Answer is:


B- Teres minor as we described above.

268-patient came to you with pain in the right buttocks and local
tenderness in the tibial tuberosity with pain in resistive knee flexion
what is your diagnosis?
A) Hamstring strain
B) hip arthritis
C) sports hernia
D) joint inflammation

The correct Answer is:


A- Hamstring strain Pain in the back of the thigh and lower buttock
when walking, straightening the leg, or bending over Tenderness ,
Bruising

269- Physiotherapist asks pt. to flex the wrist and abduct it


which muscle acts in this action
a- flexor Carpi ulnaris
b- flexor Carpi radialis
c-flexor digitorum profundus
d- biceps brachii

The correct Answer is:


A-flexor carpi radialis action: is flexion and abduction of the wrist

270-physiotherapist asks pt to flex the wrist and adduct it ;

a- flexor Carpi ulnaris


b- flexor Carpi radialis
c-flexor digitorum profundus
d- biceps brachii

The correct Answer is:


A- flexor carpi ulnaris action is flexion with adduction

271-Shortness of whitch muscle cause limitation in shoulder abduction


and latral rotation
a-pectoralis major
b-serratus ant
c-sub scapularis
d-teres major

The correct Answer is:


A-pectoralis \major action is adduction with internal rotation

272-All of the following are adductors of hip joint except


A. Gracilis
B. Pectineus
C. Sartorius
The correct Answer is:
C- Sartorius abductors muscle as we described above

273-winging of scapula due to weakness of


a- pectorals major
b-serratus ant
c-sub scapularis
d-teres major

The correct Answer is:


B- Serratus Anterior

274-injury to long thoracic nerve leading to weakness of


a-pectorals major
b-serratus ant
c-sub scapularis
d-teres major

The correct Answer is:


B- serratus Anterior supplied by Long Thoracic nerve
Pectorals major supplied by lateral and medial pectoral nerve
Sub scapularis and teres major supplied by upper & lower subscapular

275-with affection of phrenic nerve we stretch


a- sternocleidomastoid
b-trapezius
c-diaphragm
d-masseter

The correct Answer is:


A- Sternocleidomastoid above phrenic nerve by 1 inch and we can
manipulate it by stretching SCM muscle

276-which muscle used in swimming


a- pectorals minor
b- serratus anterior
c- teres minor
d-latissmus dorsi
The correct Answer is:
D- Latissmus dorsi as we make extension with adduction with

278-All of the following muscles affected in tennis elbow except


a- Ext carpi radials brevis
b– Ext carpi ulnaris
c– Ext digitorum
d- flex carpi ulnaris

The correct Answer is:


D- flexors carpi ulnaris

279-what’s the name of this muscle:

a- Biceps brachii
b- Brachioradialis

The correct Answer is:


A- Bicecps brachii

280-Patient suffering from pain insertion of this muscle the diagnosis:


a-rotator cuff tear
b-infra spinatus tendindinits
c-surpaspinatous tendinitis

The correct Answer is:


C- supraspinatus tendinitis

281- which of the following doesn`t contribute to stabilizers of


a-sternoclavicular J
b-interarticular disc
c-ant. and post. sc ligaments
d-acromiclavicular ligament

The correct Answer is:


D- Acromioclavicular ligament doesn’t contribute to stabilizers
Stabilizers are : Sternoclavicular joint and it`s ligaments , interarticular
disc

282-which part not included in posterior neck triangle


a-clavicle
b-trapezuis
c-strenoclidomastoid
d-strenum
The correct Answer is:
D- Sternum

282-which is not related to the shoulder triangle


a)Clavicle
b)Sternum
c) Humurars
The correct Answer is:
B- Sternum

283-Protraction and retraction of shoulder occurs at


a-glenohumeral joint
b-scapulothoracic interfance
c-acromioclavicular joint

The correct Answer is:


B- Scapulothoracic interface

277-What is the action of this muscleshape ?

A- hip adduction
B- hip extenstion
C-back extension

The correct Answer is:


B- Hip Extension

284-The physical therapist is reading the physician’s interpretation of an


x-ray that was taken of the left humerus of a 7-year-old patient. The
physician notes in the report the presence of an incomplete fracture on
the convex side of the humerus. Which type of fracture is the physician
describing?
a- Comminuted
b- Avulsion
c- Greenstick
d- Segmental

The correct Answer is:


C- Greenstick

285- Bennett’s fracture is


a. Little finger
b. Thumb
c. Middle finger
d. Distal end of radius

The correct Answer is:


B- Thumb fracture called Bennett`s fracture

286-A patient presents to an outpatient clinic with an order to evaluate


and treat the right forearm and wrist secondary to nerve compression.
The patient has the following signs and symptoms: pain with manual
muscle testing of pronation, decreased strength of the flexor pollicis
longus and pronator quadratus, and pain with palpation of the pronator
teres. What nerve is most likely compromised? What is the most likely
area of compression?
a-Median nerve – carpal tunnel
b-Ulnar nerve – Guyon’s canal
c-Ulnar nerve – pronator quadratus
d-Median nerve – pronator teres

The correct Answer is:


D-Median nerve – pronator teres

287-Thumb limited movement which of the following muscle not


affected:
a- Flex. Policies bevies
b- Abd. Policies bevies
c- oppons. Pollicis

The correct Answer is:


B- Abduction policies bevies

288-Which of the following is not part of the triangular fibrocartilage


complex of the wrist?
a- Dorsal radio-ulnar ligament
b- Ulnar collateral ligament
c- Radial collateral ligament
d- Ulnar articular cartilage

The correct Answer is:


C-Redial Collateral ligament is triangular fibrocartilage complex of the
wrist which is consist of ulnar collateral ligament, ulnar articular
cartilage & dorsal radio-ulnar ligament.
289-Which tendon is most commonly involved with lateral epicondylitis?
a-Extensor carpi radialis longus
b-Extensor carpi radialis brevis
c-Brachioradialis
d-Extensor digitorum

The correct Answer is:


B- Extensor carpi radialis bevies.

290-There are post contusion of the patient thumb, the least movement
allow :
a- radial and ulner deviation
b- thumb adduction and flexion
c- extension and flexion of the wrist
d- supination and pronation

The correct Answer is:


B- Thumb adduction and flexion

291-You are performing sensory tests on a patient diagnosed with C6


nerve root impingement. Testing should concentrate on the:
a-3rd, 4th and 5th fingers
b-ulnar border of the hand
c-thumb and index fingers
d-medial forearm
The correct Answer is:
C- Thumb and Index fingers is a dermatome for C6 nerve root

292-The number of joint and bones in the foot


a-23 joint and 25 bones
b-33 joint and 26 bones
c-23 joints and 17 bones

The correct Answer is:


B-33 joints and 26 bones

293-elbow joint
a- uniaxial
b-biaxial
c- three axial
d-multi-axial

The correct Answer is:


B-Biaxial joint

294-all of the followoing are synovial joints except


a- Elbow joint
b- Knee joint
c- Hip joint
d-Symphysis pubis

The correct Answer is:


D- symphysis pubis isn`t synovial joint

295-which is synovial joint


a- TMJ joint
b- Symphisis pubis

The correct Answer is:


A- TMJ joints is a synovial joint

296– During test of splaneous capitis which action not needed :


a) Extension
b) Flexion
c) Rotation
The correct Answer is:
B-Flexion isn`t needed for splaneous capitis action which is extension ,
rotation , side bending

297-Splenius capitis response of :


a- neck flexion
b- neck extension
c- neck rotation
d- neck side bend

The correct Answer is:


B-neck extension as we described above

298-which of following not needed in testing the trapezius muscle :


a-head flexion
b-head extension
c-head rotation
d-head lateral flexion

The correct Answer is:


A-head flexion isn`t needed in testing for trapezius because it`s action is
rotation, retraction, elevation, and depression of scapula

299-which of following not needed in testing the sternocleidomastoid


muscle:
a-head flexion
b-head extension
c-head rotation
d-head lateral flexion

The correct Answer is:


A- head extension , sternocleidomastoid action Unilaterally; cervical
rotation to opposite side, cervical lateral flexion to same side
Bilaterally; cervical flexion, raises the sternum and assists in forced
inhalation

300-A physical therapist is beginning an evaluation of a 5-year-old boy.


The mother indicates that she pulled the child from a seated position by
grasping the wrists. The child then experienced immediate pain at the
right elbow. The physician’s orders are for right elbow range of motion
and strengthening. Which of the following is the most likely diagnosis?
a- Radial head fracture
b- Nursemaid’s elbow
c- Erb’s palsy
d-Ulnar coronoid process fracture

The correct Answer is:


B- nursemaid`s elbow , babysitter's elbow, or pulled elbow is a
dislocation of the elbow joint caused by a sudden pull on the extended
pronated forearm, such as by an adult tugging on an uncooperative child
or by swinging the child by the arms during play. The technical term for
the injury is radial head subluxation
301-patient has hand injury without open wound and making tight band
only after few 3 week referral to you and have contracture of hand and
wrist causes of these are :
a- Dupuytren’s contracture Tendon hand affected
b-Volkaman contracture Tendons hand&wrist

The correct Answer is:


B- Volkaman contracture tendons hand & wrist.
Dupuytren’s contracture Tendons hand only affected

302-Rheumatoid arthritis
a- cold limb
b- radial deviation
c-pain ,wet ,swelling

The correct Answer is:


C- Pain, wet & swilling

303- The first thing affected in Rheumatoid


arthritis :
a- capsule
b- articular cartilage

The correct Answer is:


A- Capsule

304. Involvement of PIP joint, DIP joint and the carpometacarpal joint of
base of thumb with sparing the wrist is seen in:
a- Rheumatoid arthritis
c- Osteoarthritis
d- Psoriatic arthritis
e- Pseudogout

The correct Answer is:


A- rheumatoid arthritis

305-O.A in old age due to:


a- traumatic
b- tesr and wear process

The correct Answer is:


B- Tesr and wear process
306-Sever old osteoarthritis patient. You recommended for
a- hot backs + exercise + walking
b- walkers
c-cold backs + exercise + walking
d-cane

The correct Answer is:


B- Walkers

307-Pt complains of pain in the hip region from 3years increasing With
time &WB its
A- O.A of hip joint
B- Tight hip flexor
C- Trochanteric bursitis

The correct Answer is:


A- O.A of hip joint

308-Pt cannot raise hand in abduction or maintain abduction position


due to:
a) Adhesive capsulitis
b) Supra spinatous tendenitis
c) Rotator cuff tear

The correct Answer is:


C- Rotator cuff tear

308-patient with shoulder pain during evaluation there is no passive or


active ROM in abd. And rot. That may cause by:
a- frozen shoulder
b- supraspinatus tendonitis
c- rotator cuff tear
d- acromioclavicular dysfunction

The correct Answer is:


A- Frozen shoulder

309-In taping an athlete’s ankle prophylactically before a football game,


in what position should the ankle is slightly positioned before taping to
provide the most protection against an ankle sprain?
a-Inversion, dorsiflexion, abduction
b-Eversion, plantarflexion, adduction
cEversion, dorsiflexion, abduction

The correct Answer is:


C- Eversion, dorsiflexion, abduction

310-Acute adhesive capsulitis:


a- acute inflammation capsule with fibrous
b- chronic inflammation capsule with fibrous
c- capsule inflammation
d- fibrous capsule

The correct Answer is:


C- Capsule inflammation

311-During examination of shoulder, you found increased motion (laxity)


in anterior, posterior and inferior directions, the condition is
a- Global shoulder instability
b- Rotator cuff tear

The correct Answer is:


A- Global shoulder instability
312-A patient with a diagnosis of a rotator cuff tear has just begun
active range of motion. The therapist is strengthening the rotator cuff
muscles to increase joint stability and oppose the superior shear of the
deltoid. Which of the rotator cuff muscles participate least in opposing
the superior shear force of the deltoid?
a- Infraspinatus
b- Subscapularis
c-Teres minor
d-Supraspinatus

The correct Answer is:


D- supraspinatus

313-A therapist is attempting to gain external rotation range of motion


in a patient’s right shoulder. The therapist decides to use contract-relax-
contract antagonist. In what order should the following rotator cuff
muscles contract to perform this movement successfully?
a-Infraspinatus – teres minor
b-Subscapularis – supraspinatus
c-Teres minor – infraspinatus
d-Supraspinatus – subscapularis

The correct Answer is:


D- supraspinatus – subscapularis

314--in condromalacia consider in the ttt?


a- patellar tapping and strenthening of the quadriceps
b- strenthening vastus medialis only
c- strenthening vastus lateralis only
d- strenthening of hamstring

The correct Answer is:


A- patellar Tapping and strentheninh of the quadriceps
315-myositis ossificans most affected
a- wrist Joint
b- Elbow Joint
c- Shoulder Joint
d- Knee Joint
The correct Answer is:
B- Elbow joint
316-Spyndolythesis
a-vertbera above vertebra dislocat posterior
b-vertbera below vertebra dislocat posterior
c -vertbera above vertebra dislocat anterior
d-vertbera belowe vertebra dislocat anterior

The correct Answer is:


C- Vertebra above vertebra dislocate anterior

317- Osteomyelitis is:


a- infective inflammation of the bone due infective bacteria entered into
bone.
b- Decreased bone density and increased porosity
d- due to repetitive trauma
c- Vitamin D deficiency

The correct Answer is:


A- Infective inflammation of the bone due infective bacteria entered into
bone.

318-A patient presents to physical therapy with complaints of pain in the


right hip due to osteoarthritis. Which of the following is not true about
this type of arthritis?
a- Causes pain usually symmetrically because it is a systemic condition.
b- Not usually more painful in the morning.
c- This type of arthritis commonly involves the distal interphalangeal
joint.
d- Mainly involves weight-bearing joints.

The correct Answer is:


A- Causes pain usually symmetrically because it`s a systemic condition ,
this type of arthritis commonly involves the distal interphalangeal joint
are complains of Osteo myelitis not osteoarthritis
319-A 65 years old patient with well treated collas fracture, few
monthes later came with severe pain in hand with coldness color
changes . x ray show big callus around radial head not remember This
may be due to :
A- osteoarthritis of wrist
B -prearticular ossification
C- Sudeck's atrophy
The correct Answer is:
A- Sudeck`s atrophy /reflex sympathetic dystrophy
(RSD)/algodystrophy/causalgia The term complex, regional pain
syndrome is now being used to describe these pathological states. This
is a complication where the patient complains of severe
pain on movement, or at rest, out of proportion to the initial injury. The
limb is swollen. The skin appears shiny and discoloured and feels cold;
in extreme cases this may lead to the limb becoming exquisitely tender
and discoloured. Osteoporosis and permanent contractures

320-Sudeck's atrophy which is not true


a- sever burning pain
b- sympathetic hyperactivity
c- trophic changes occur
d- more perspiration

The correct Answer is:


B- sympathetic hyperactivity

321-which is not true about sudeck's atrophy


a- osteoporosis of bones of hand
b- fully curable by taking sympathetic nerve blocker

The correct Answer is:


B- fully curable by taking sympathetic nerve blocker

322- what is the cause of hip dysplasia?


A)mother malnutrition
B)idiopathic

The correct Answer is:


B-Idiopathic
323-Following the reduction of developmental dysplasia of the hip in an
infant, the physical therapist should instruct the parents to maintain
that hip in a stable position of
a- flexion and adduction
b- Flexion and abduction
c- Extension and adduction
d- Extension and abduction

The correct Answer is:


B- flexion and abduction

324--All is true about med. meniscus except


a- More injured than lateral meniscus
b- Absorb of about 90% of load on knee joint
c- Injury from hyper extension injury

The correct Answer is:


C- injury from hyper extension injury isn`t correct about med. Meniscus
injury because it`s injured by excessive valgus stress with external
rotation.

325-History of football players has injured by twisted knee and take out
of game and after little time it swelling and warm. after few days it
locks during climbing stairs and painful and cannot put full weight
in walking :
a- patella fracture
b- tibia fracture
c- MCL rupture
d- Rupture of something of fibers

The correct Answer is:


C- MCl rupture due to it`s injured by twisted knee , and there is
ascending pain with swelling , warm and locks in some degree of flexion
326- years old foot player come to the physiotherapist with pain at the
left knee joint while the physiotherapist taking history patient said that
the injury occurred when he rotates to the right on weight bearing to the
left knee . it was flexed 60 degree. that means that the injury is at
a- medial collateral ligament
b- semilunar cartilage at the knee
c- femoral condyles
d- upper shaft of the tibia
The correct Answer is:
B- semilunar cartilage at the knee = meniscus tears can happen when a
person changes direction suddenly while running, or standing used one
leg weight bearing and often occur at the same time

327-A patient is an outpatient facility because of an injury sustained to


the right knee joint. Only the structures within the synovial cavity were
compromised during the injury. Knowing this information only, the
therapist is not concerned with injury to which of the following
structure?
a- Patellofemoral joint
b- Anterior cruciate ligament
c- Medial meniscus
d- Femoral condyles

The correct Answer is:


B- Anterior Cruciate ligament

328-What portion of the adult knee meniscus is vascularized?


a- Outer edges
b- Inner edges
c- The entire meniscus is vascular
d- The entire meniscus is avascular

The correct Answer is:


A- outer Edge is more vascularized than whole meniscus

329- Delayed ulnar nerve due to fracture of


a- olecranon fossa
b- Medial epicondyle
c- Lateral epicondyle
The correct Answer is:
B- medial ebicondyle
330-Hyaline cartilage innervated by
A -blood vessels
b- synovial fluid
c- epiphyseal growth

The correct Answer is:


B-synovial fluid

331-Acceleration injury (whiplish injury) may cause


a- pain in posterior aspect of neck
b- pain at post aspect of back
c- dislocation of vertebral colum
d- tearing of post para spinal ligament

The correct Answer is:


A- Pain in posterior aspect of neck . Whiplash is a nonmedical term used
to describe neck pain following an injury to the soft tissues of your neck
(specifically ligaments, tendons, and muscles). It is caused by an
abnormal motion or force applied to your neck that causes movement
beyond the neck's normal range of motion.
Symptoms of whiplash injury are
•Neck pain •Neck swelling •Tenderness along the back of your neck
•Muscle spasms (in the side or back of your neck) •Difficulty moving your
neck around •Headache •Pain shooting from your neck into either
shoulder or arm

332-The most common site of fracture in old patient due to failing is ;


a- head of femur
b- trochanteric
c- shift of femur
d- neck of femur

The correct Answer is:


D- neck of femur

333-All of the following is clinical features in fracture neck of femur


except
a-pain in hip region
b- abduction of leg
c-shortness of the limb
d- lateral rotation of the leg
The correct Answer is:
B- abduction of the leg is incorrect , the clinical feature of Neck femur
fracture is adduction with lateral rotation in the leg .

334-The therapist receives a referral to evaluate a patient with a


boutonniere deformity. With this injury, the involved finger usually
presents in the position of
a- Flexion of the proximal interphalangeal (PIP) joint and flexion of the
distal interphalangeal (DIP) joint.
b- Extension of the PIP joint and flexion of the DIP joint.
c- Flexion of the PIP joint and extension of the DIP joint.
d- Extension of the PIP joint and extension of the DIP

The correct Answer is:


C- flexion of the pip joint extension of the dip joint

335-A boutonniere deformity of the finger would not consist of


a- hyperextension of the proximal interphalangeal joint
b-overstretch of the extensor digitorum communis tendon
c- volar slippage of the lateral bands
d- hyperextension of the distal interphalangeal joint

The correct Answer is:


A- hyperextension of the proximal interphalangeal joint

336-a patient referred to physical therapy with a diagnosis of


Dupuytren’s contracture? Which of the following is used to treat
A- Knee continuous passive motion (CPM)
b- Work simulator set for squatting activities.
c- Hand splint.
d- A two-pound dumbbell

The correct Answer is:


C- Hand splint

337-avascular necrosis can be a possible Squeal of FRACTURE of all of


the following bones, except:
a-Femur neck
b-Scaphoid
c-Talus.
d-Calcaneum
The correct Answer is:
D- Calcaneum

338-player football takes trauma in anterior aspect of tibia with positive


posterior drawer sign
a- anterior curicate ligement
b-posterior curicate ligemnet
c-medial collteral ligement
d- lateral colatteral ligment

The correct Answer is:


A- ACL

339-Avascular necrosis would least likely be a major complicating factor


in the rehabilitation of a patient with:
a-Legg-Calvé Perthes’ disease
b-a tibial fracture
c- degenerative joint disease of the cervical spine
d- a carpal scaphoid fracture

The correct Answer is:


C- Degenerative joint disease of the cervical spine is the least
complicating with avascular necrosis
(LCPD) Legg-Calve Perthes disease: resulting avascular necrosis of the
proximal femoral head is from compromise of the tenuous blood supply
to this area. LCPD usually occurs in children aged 4-10 years. The
disease has an insidious onset and may occur after an injury to the hip.
In the vast majority of instances, the disorder is unilateral. Both hips
are involved in less than 10% of cases, and the joints are involved
successively, not simultaneously

340-To prevent maximal compressive forces being placed on the patella,


a therapist should minimize placing the patient:
A. prone and flexing the knee to 30 degrees
B. in a sitting position with the knee flexed to 90 degrees
C. supine and flexing both the hip and knee to 110 degrees
D. prone and flexing the knee to 110 with the hip extended

The correct Answer is:


D- Prone and flexing the knee to 110 with hip extended
BLOOD SUPPLY TO MUSCLE DURING EX

The cause of cubital valgus fracture of lateral condyle Fracture


The cause of cubital varus fracture of supracondylar Fracture
The position of mechanical traction for lumber disc herniation prone
with no pillow
The sheath of muscle fiber its name sarcolemma
paraesthesia means Destruction in sensory system cause abnormal
sensation
Femoral nerve stretch test The patient lies prone, the knee is
passively flexed to the thigh and the hip is passively extended; the test
is positive if the patient experiences anterior thigh pain. This test is
usually strongly positive in patients with protrusions at L2-L3 and L3-L4,
slightly positive or negative in L4-L5 disk protrusions and negative in
cases with a lumbosacral protrusion.

341-Knee medial rotation (internal Rotation) due to


A. weak vastus medialis
B. weak semitendinosis
C. Weak quadriceps
d- weak bicepis femoris

The correct Answer is:


D- Weak biceps femoris because it`s action is flexing knee joint, laterally
rotating knee joint (when knee is flexed), extending hip joint (long head
only)

342-Full adduction during evaluation of a patient, the therapist observes


significant posterior trunk lean at initial contact (heel strike). Which of
the following is the most likely muscle that the therapist needs to focus
on during the exercise session in order to minimize this gait deviation?

a-Gluteus medius
b-Gluteus maximus
c-Quadriceps
d-Hamstrings,
The correct Answer is:
B- Gluteus Maximus is a responsible for full internal rotation and full
extension
343-Decrease osteoblast cause
a-osteoporosis
b-ricket
c-osteomalacia

The correct Answer is:


A- Osteoporosis

344-When the ankle is forcibly inverted and plantar flexed, the ligament
that is most frequently disrupted is the:
a- deltoid
b- anterior talofibular
c- posterior talofibular
d- calcaneofibular
The correct Answer is:
A- Anterior Talo-fibular ligament

345- Increase glenoid cavity for proximal humerus in shoulder j.


A. capsule
B. labrum
C. ligament and tendon

The correct Answer is:


B- Labrum

346-Inversion injury at the ankle can cause all of the following except
a- FRACTURE tip of lateral malleolus
b- FRACTURE base of the 5th metatarsal
c- Sprain of extensor digitorum bevies
d-FRACTURE of sustentaculam tails

The correct Answer is:


D- Fracture of sustentaculam

347-A 45 years male presented with an expansile lesion in the center of


femoral metaphysis. The lesion shows endosteal scalloping &
punctuates calcifications. Most likely diagnosis is
a- Osteosarcoma
b- Chondrosarcoma
c- Simple bone cyst
d- Fibrous dysplasia
the correct Answer is:
B- Chondrosarcoma

348-most common site of estrogenic sarcoma is:


a-Femur, upper end
b-Femur, lower end
c-Tibia, upper end
d-Tibia, lower end

The correct Answer is:


A- Femur, Upper end.

349-Deformity associated with colles fracture (fracture lower end of


radius)
a- Dinner fork deformity
b - Pes planus
c- Talipus equinovarus

The correct Answer is:


A- Dinner fork deformity

350- Colles fracture may possibly cause injury to which of the following
a- flexor policies
b- abductor policies
c- adductor policies
d- extensor policies longus

The correct Answer is:


D- Extensor policies longus.

351-What the other name of fracture radius:


a-no name
b-pontine fracture
c-bennt fracture
d-colles fracture

The correct Answer is:


D- Colles fracture

352-which is incorrect about plump line of body in standing


A- pass posterior To ear lobe
B- pass through shoulder
C- pass Posterior to greater trochanter
D- pass anterior to ankle

The correct Answer is:


B- Pass through shoulder isn`t correct about plump line.
LOG
æÈíÚÏí ÃãÇã ٢
2ÇáßÊÝ æÇáßÚÈíä
Ýíåã ÍÑÝ ßÇÝ
ÇáÝÎÐ æÇáÇÐäíä ٢ÈíÚÏí æÑÇ
ÇáÑßȪ æ ÇáÝÞÑÉ ÇáÊÇäíÉ Ýí ٢æÈíÚÏí ÌæÇ
ÇááãÈÑÑ

353-One of the possible complications following a fracture is


Volkmann's ischemic contracture. This condition:
a- Is caused by an interference with the venous return
b- Is caused by an interference of the nerve supply
c- May occur if the fracture is sustained in the upper extremity
d-None of the above

The correct Answer is:


D- None of the above

354-Which is correct about Volkmann’s ischemic contracture


a-affect flexor of forearm
b-affect palmar fascia
C-lead to ulnar neuritis

The correct Answer is:


A- Flexor of forearm

355-volkmans ischemic contracture occurs due to:


a-fracture of medial epcondyle of humerus
b-fracture of lat epicondyle of humerus
c-fracture lower third of humerus
d- surgical neck

The correct Answer is:


C- Fracture Lower third of humerus.
356-which is not correct about Gallilze #
A-fracture of radius and subluxation of ulna
b- fracture of ulna and subluxation of radius
c-its occur by falling

The correct Answer is:


B- Fracture of Ulna and subluxation of raduis isn`t correct it`s Monteggia
fracture
Lateral condyles # lead to valgus deformity
Supracondylar # leads to varus deformity

Fracture at surgical neck of humerus injury of Axillary nerve & muscle


affected Deltoid and occurs loss of abduction of arm

Fracture at mid shaft (spiral groove) of humerus injury of Radial nerve &
occur wrist drop & decreased extension of elbow

Fracture at medial epicondyle of humerus injury of ulnar nerve & occur


ulnar claw hand

Fracture of distal radius (resulting from fall on outstretched hand


(extended wrists) produces "dinner fork" appearance
Where distal fragment is posterior)

Colles' fracture of distal radius (resulting from fall on back of hand


(flexed wrists) less common)

Smith's fracture most commonly FRACTURED carpal bone (avascular


necrosis of proximal part when deprived of its arterial supply)

Scaphoid most commonly DISLOCATED carpal bone (occurs in young


adults) Lunate

Galezzi fracture radius & dislocation ulna


Montagia fracture ulna & dislocation radius

357- Monteggia fracture means which of the following


a- fracture of the radius and subluxation of the lower end of ulna
b- fracture of the ulna with radial head subluxation
c- green stalk fracture with minor angulation fracture of the radio-ulnar
joint
The correct Answer is:
B- Fracture of the ulna with radial head subluxation

358-All of the following are associated with supracondylar of humerus,


except:
a- It is uncommon after 15 years of age
b- Extension type FRACTURE is more common than the flexion type
c- Cubits varus deformity commonly results following malunion not sure
d- Ulnar nerve is most commonly involved

The correct Answer is:


D- Ulnar nerve is most commonly involved

359- Valgus deformity results from.


a- lateral condylar-
b- medical condylar-
c- supra condylar.

The correct Answer is:


A- Lateral condylar.

360- Supracondylar ridge of humerus injury to


a- Ulnar n nerve
b- brachial nerve
c- median nerve

The correct Answer is:


C- Median nerve

361-Person fall on lat aspect of shoulder last thing occur is


a- dislocation SC joint
b- dislocate acromio clavicular j=joint
c- # medial clavicle
d- #middle third of clavicle

The correct Answer is:


A- Dislocation in Sterno-clavicle joint is the last thing occurs.

362- Skeletal traction used as ttt of


a) # shaft of femur
b) # tibia
The correct Answer is:
A- Shaft of femur

363-when the fracture accurse in neck of humerus what nerve affected


A- Redial N
B- Median N
C- Ulnar N
D- Subcutaneous nerve

The correct Answer is:


D- Subcutaneous nerve.

364- this angle in the picture means

a- genovalgum
b- coxavalgum
c- coxavara
d- genurecrvatum

The correct Answer is:


C- Coxavara《120
Coxavalgum 》135
Coxanorma =120 ,
Genovalgum Bow leg

365-The angle between Neck of femur and Shaft of femur :


A- 90 Degree
B- 120 Degree
c-150 Degree
d-170 Degree
The correct Answer is:
B- 120 Degree as we described above

366-The angle between longitudinal axis passing throw femoral


neck and longitudinal axis of femur is decreased in the case of:
A -genu valgum
B -genu varum
C -genu recurvatum
D -coxa Vera

The correct Answer is:


D- Coxa Vera as we described

367-Observing a patient in a standing position, the therapist notes that


an angulation deformity of the right knee causes it to be located
medially in relation to the left hip and foot. This condition is commonly
referred to as:
a-genu varum
b-genu valgum
c-pes cavus
d-none of the above

The correct Answer is:


B- genu Valgum

368-the normal angle between the fumer and the neck of fumer
is 126 when the angle increase the deformity is:
a- coxa valga
b- genu Valgus
C- genu recurvatum

The correct Answer is:


A- Coxa Valga as we described above.

369-Coxa Vera present as abnormality of


a-proximal femur
b-distal femur
c-proximal tibia
d-distal tibia

The correct Answer is:


A- proximal Femur
370-Most common compression of radial n in
1-shaft of humerus
2-neck of humerus
3-distal end

The correct Answer is:


A- shaft of humerus spiral groove as we described above.

371- Congenital dislocation occur in


a- Hip joint
b- knee joint
c- shoulder joint
d- elbow joint

The correct Answer is:


A- Hip joint

372-which statement not correct about fracture neck of femur


a-its fatal
b-need arthoplasty
c-lead to avascular necrosis
d-occur mostly in children

The correct Answer is:


D- Occur mostly in children isn`t correct

373-About Tennis elbow all the following is true except


A- also called lateral epichondylitis
B- precipated by ulnar neuritis
C- usually occur in dominant side
The correct Answer is:
B- precipated by ulnar neuritis.

374-paralysis of all intrinsic muscles of handexcept abd poll brevis


A -median n
B -ulnar n
C -Radial n

The correct Answer is:


A- median nerve as we described above
375-most compression site of superfacial peroneal nerve is:
a) Latral condyle of femure
b) Latral head of fibula

The correct Answer is:


B- Lateral head of fibula.

376-Pt with lordosis from standing and +ve Thomas test due to:
a- strong lumbar extensors
b- fixed flex deformity
c- hamstring strain

The correct Answer is:


B- fixed flexion deformity due to it`s positive Thomas test.

377-What is the most likely cause of anterior pelvic tilt during initial
contact (heel strike)?
a-Weak abdominals
b-Tight hamstrings
c-Weak abductors
d-Back pain

The correct Answer is:


A- weak abdominals

378- If the line of gravity is posterior to the hip joint in standing, on what
does the body first rely to keep the trunk from moving into excessive
lumbar extension?
a- Illiopsoas muscle activity
b- Abdominal muscle activity
c- Anterior pelvic ligaments and the hip joint capsule.
d- Posterior pelvic ligaments and the hip joint capsule.

The correct Answer is:


C- Anterior pelvic ligaments and the hip joint capsule
379- sciatic nerve injury due to
a) Post dislocate hip
b) Ant dislocate hip
The correct Answer is:
A- Posterior dislocate hip

380-In contracture of lower lumbar back muscles, the pelvis is affected


by
a. Anterior pelvic tilting with upper thoracic kyphosis
b. Anterior pelvic tilting with lordosis
c. Posterior pelvic tilting with lordosis
d. No effect on the pelvis

The correct Answer is:


B- Anterior pelvic tilting with lordosis

381-Patient with abd scapula, round shoulder .,kyphosis ,ant rotation of


pelvic
A. Lordosis
B. Sway back
C. Flat back
D. Kypholordosis

The correct Answer is:


D- Kypholordosis due to abduction scapula with round shoulder “
Kyphosis “ with Anterior rotation pelvic.

382– Patient has post pelvic tiled, flat lower thoracic & increase upper
back kyphosis
A) Flat back
B) Sway back

The correct Answer is:


A- Flat back because it`s not recognized that the patient has excessive
hip extension with knee extension and flat lower thoracic

383- The therapist performed Trendelenburg test for patient with right
weakness of gluteus medius. The patient stands on right lower limb. The
pelvis drop would be observed as
a. Pelvis does not drop as the patient leans by the trunk to the right
b. Pelvis will drop at the left side
c. Pelvis will drop at the right side
d. Pelvis does not drop as the patient leans by the trunk to the left

The correct Answer is:


B- pelvis will drop at the left side , Patient stands on 1 foot and tries to
maintain his position.
-- The unsupported pelvis should rise for negative.
-- If pelvis stays level or descends for positive.

384-The therapist receives a referral to evaluate a patient with a


boutonniere deformity. With this injury, the involved finger usually
presents in the position of
a- Flexion of the proximal interphalangeal (PIP) joint and flexion of the
distal interphalangeal (DIP) joint.
b- Extension of the PIP joint and flexion of the DIP joint.
c- Flexion of the PIP joint and extension of the DIP joint.
d- Extension of the PIP joint and extension of the DIP joint
The correct Answer is:
C-felxion of the PIP joint and Extension of the DIP joint

385-If there is weakness of right gluteus medius dropping pelvis of


occurs at
a-right pelvic drop at stance phase
b-left pelvic drop at stance phase
c-right pelvic drop at swing phase
d-left pelvic drop at swing phase

The correct Answer is:


D- left pelvic drop at swing phase as we described above.

386- A therapist is evaluating a patient with poor motor coordination.


The therapist observes that when the patient is standing erect and still,
she does not respond appropriately when correcting a backward sway of
the body. With the body in a fully erect position, a slight backward sway
should be corrected by the body firing specific muscles in a specific
order. Which list is the correct firing order?

A- Bilateral abdominals, bilateral quadriceps, bilateral tibialis anterior.


B- Bilateral abdominals, bilateral tibialis anterior, bilateral quadriceps
C- Bilateral tibialis anterior, bilateral abdominals, bilateral quadriceps
D- Bilateral tibialis anterior, bilateral quadriceps, bilateral abdominals.

The correct Answer is:


D- Bilateral tibialis anterior, bilateral quadriceps, bilateral abdominals.

387-Which of the following is observed by the therapist if a patient is


correctly performing an anterior pelvic tilt in standing position?
a- Hip extension and lumbar flexion.
b- Hip flexion and lumbar extension.
c- Hip flexion and lumbar flexion.
d- Hip extension and lumbar extension

The correct Answer is:


B- hip flexion & lumber extension.

388- Which of the following is a possible cause of anterior pelvic tilting


a. Weak lateral trunk muscles
b. Tightened tensor fascia lata
c. Tight hamstring
d. Severe weakness of abdominal muscle

The correct Answer is:


D- Sever weakness of abdominal muscle

389- If the line of gravity is posterior to the hip joint in standing, on what
does the body first rely to keep the trunk from moving into excessive
lumbar extension?
a- Iliopsoas muscle activity
b- Abdominal muscle activity
c- Anterior pelvic ligaments and the hip joint capsule.
d- Posterior pelvic ligaments and the hip joint capsule.

The correct Answer is:


C- Anterior pelvic ligaments and the hip joint capsule

390- Right convex Scoliosis we find


A- Lateral trunk flexion & concave side to RT
B- Lateral trunk flexion &convex side to RT
C- Lateral trunk flexion &convex side to LT
D- Lateral trunk flexion & shorten RT musculature
The correct Answer is:
B- Lateral trunk flexion & convex side to RT

391-A 14-year-old girl with right thoracic scoliosis is referred to physical


therapy. The therapist should expect which of the following findings?
a-Left shoulder high, left scapula prominent, and right hip high
b-Left shoulder low, right scapula prominent, and left hip high
c-Right shoulder high, right scapula prominent, and right hip high
d-Right shoulder low, right scapula prominent, and left hip high

The correct Answer is:


B- left shoulder lo

392-The most common deformity in geriatric is


a- Kyphosis
b- Scoliosis
c- Kyphoscoliosis
d- Lordosis

The correct Answer is:


A- Kyphosis

393- Severe kypho-scoliosis will lead to


a- Left rather than right ventricular failure
b- Right rather than left ventricular failure
c- Frequent respiratory infections
d- No cardiac abnormalities

The correct Answer is:


C- frequent respiratory infections

394-You are evaluating an athlete who is complaining of pain in the left


shoulder region. Your assessment of the shoulder elicits pain in the last
30 degrees of shoulder abduction range of motion. This finding is most
congruent with
a- calcific supraspinatus tendinitis.
b- sub acromial bursitis.
c- acromio-clavicular sprain.
d- thoracic outlet syndrome.

The correct Answer is:


C- Acromio-Clavicular sprain due it`s range of motion
395-Which of the following is the most vulnerable position for
dislocation of the hip?
a- 30o hip extension, 30o hip adduction, and minimal internal rotation
b- 30o hip flexion, 30o hip adduction, and minimal external rotation
c- 30o hip flexion, 30o hip abduction, and minimal external rotation.
d- 30o hip extension, 30o hip abduction, and minimal external rotation.

The correct Answer is:


C- 30o hip flexion, 30o hip abduction, and minimal external rotation.

396-A patient with adhesive capsulitis of the gleno-humeral joint should


demonstrate the greatest limitation of motion when performing shoulder
a-flexion.
b-abduction.
c-medial rotation.
d-lateral rotation.

The correct Answer is:


D- lateral rotation

397-A therapist performs a test for a patient and was positive with the
thigh of the patient raised some inches above the examination table.
What is the test name and which muscle is shortened?
a. Ober test – Iliotibial band
b. Straight leg raising test – hamstring
c. Thomas test – Iliopsoas
d. Thomson – gastrocenmies

The correct Answer is:


C-Thomas test – Iliopsoas

398-PT Lachaman to asses


A- A.C.L
B- M.C.L
C- P.C.L

The correct Answer is:


A – ACL

399-Pt has tear with rotation movement Which test not need:
a- drawar
b- lachmen
C- trendburg

The correct Answer is:


C-Trendulberg Test because it`s for Gluteus Medius , It`s not Rotator
muscle

400-A patient is placed in supine position with the knee in 90o of flexion.
The foot is stabilized by the therapist’s body on the examination table.
The therapist then wraps his fingers around the proximal tibia so that
the thumbs are resting along the anteromedial and the anterolateral
margins. The therapist then applies a force to pull the tibia forward.
What special test is being performed?
a- Pivot shift
b- Lachman’s test
c- Anterior drawer
d- Posterior drawer

The correct Answer is:


C-Anterior Drawer Test

401-A therapist is examining a 3-years-old child who is positioned as


follows: supine, hips flexed to 90 o, hips fully adducted, and knees
flexed. The therapist passively abducts and raises the thigh, applying an
anterior shear force to the hip joint. A click at 30 o of abduction is noted
by the therapist. What orthopedic test is the therapist performing, and
what is its significance?
a- Ortolani’s test – hip dislocation
b- Apley’s compression/distraction test – cartilage damage
c- McMurray test – cartilage damage
d- Piston test – hip dislocation.

The correct Answer is:


A- Ortolani`s test – hip dislocation,
Apley`s compression distraction test & Macmurray Test are knee tests

402- Patient has shoulder dislocation and reduction for this dislocation
and referral to you for early mobilization to prevent:
a- Stiffness
b- Recurrent dislocation
C- Osteoarthritis
The correct Answer is:
B- Recurrent dislocation

403-Fracture due to repeated minor injury:


a- pathological
b- fatigue
c- repeated stress
d- compression

The correct Answer is:


C-Repeated strees

403- Patient has cruciate ligament injury


a) Strength Quad
b) Hamstring
c) Both

The correct Answer is:


C-Both

404-posterior shear test for


a- sacroiliac joint
b- hip joint
c- lumbosacral join

The correct Answer is:


A- Sacroiliac joint

405-The main muscle responsible for climbing stairs and coming from
reclined position,
bringing knee to chest
a- rectusfemoris
b- iliopsoas
c- quadratus lumborum
d- pectineus

The correct Answer is:


B- Iliopsoas muscle
406-All the following is true about ligaments except ,
A- high collagen content
B- laxity lead to hyper mobility
C- connect on bone to another
D- usually heals fast and need not support

The correct Answer is:


D- Usually heals fast and not need support

407- Thompson test is for


a. Shortening of iliotibial band
b. Integrity of Achilles tendon
c. Shortening of iliopsoas
d. Integrity of quadriceps tendon
The correct Answer is:
B- Integrity of Achilles tendon test

408- Talipes equinovarus:


a. Ankle dorsi flexed, everted, adducted
b. Ankle plantar flexed, inverted, abducted
c. Ankle dorsi flexed, everted, abducted
d. Ankle plantar flexed, inverted, adducted

The correct Answer is:


D- Ankle planter flexed, inverted, adducted.

409- patient has shoulder dislocation and reduction for this dislocation
and referral to you for early mobilization to prevent:
a- Stiffness
b- Recurent dislocation
C- Osteoarthritis

The correct Answer is:


B- Recurrent dislocation

410-Fracture due to repeated minor injury:


a- pathological
b- fatigue
c- reapeated stress
d- compression
The correct Answer is:
C- RS

411-A therapist is scheduled to treat a patient with cerebral palsy who


has been classified as a spastic quadriplegic. What type of orthopedic
deformity should the therapist expect to see in a patient’s feet?
A-Talipes equinovalgus
B-Talipes equinovarus
C-Clubfeet
D-B & C are correct

The correct Answer is:


D- B & C are Corrected

412- Valgus position of foot


a- planterflexion
b- dorsiflexion
c- eversion
d- inversion

The correct Answer is:


C- eversion

413-Maximum stability of hip joint depends on


a- bone, ligament, muscles
b- muscles, ligament, bone
c- bone only

The correct Answer is:


A- bone, ligament, bone

414-Ulnar nerve pass


a- anterior to medial epicondyle
b- anterior to lateral epicondyle
c-posterior to medial epicondyle
d- posterior to lateral epicondyle

The correct Answer is:


C- Post. To medial epicondyle

415-The shoulder work with 3 joint except


b- acromio clavicular
c- scapula thoracic
c- sterno clavicular

The correct Answer is:


C-sterno-clavicular

416-which of the following you cannot palpate while examining shoulder


A- 2nd rib
B- Sterno clavicular joint
C-1st rib

The correct Answer is:


C- 1st rib

417-patient came to clinic with morning pain and stiffness this disease
mean
a- systemic degenerative
b- muscle spasm
c- joint infection

The correct Answer is:


A- Systemic degenerative

418- A 13-year-old girl has fractured the left patella during a volleyball
game. The physician determines that the superior pole is the location of
the fracture. Which of the following should be avoided in early
rehabilitation?
a- Full knee extension
b- 45 o of knee flexion
c- 90 o of knee flexion
d- 15 o of knee flexion

The correct Answer is:


C- 90 degree of knee flexion

419-A patient is referred to the therapist with a diagnosis of arthritis.


What type of arthritis would the therapist expect if the patient present
with the following signs and symptoms? (1) Bilateral wrists and knees
are involved, (2) pain at rest and with motion, (3) prolonged morning
stiffness, and (4) Crepitus.
a- Osteoarthritis
b- Rheumatoid arthritis
c- Degenerative joint disease
d- It is not possible to determine with the given information

The correct Answer is:


C- Rheumatoid arthritis

420- The difference between RA and OA


a- OA's degenerative disease
b- RA's is systemic disease & and joint have deformity
c- RA's remission and relapse
d- OA's curable disease

The correct Answer is:


B- RA's is systemic disease & and joint have deformity Because hydro
static pressure on chest wall and on abdomen wall which acts on
diaphragm and there is indirect effect on the shift of blood to the thorax
:resulting in decrease vital capacity

421-patient suffering from morning stiffness this is a sign of


a- swelling of the body tissues
b- prolonged stiffness with inflammation.
c- muscle inflammation.
d- bonny projection at the joint

The correct Answer is:


B- prolonged stiffness with inflammation.

422-In degenerative joint disease which is not occure?


a- increased with weight bearing on the joint
b- gradual onset
c- stiffness at morning
d- increaser after prolonged period of un activity

The correct Answer is:


B- Gradual onset isn`t a Characteristics of degenerative joint disease
Signs and
symptoms which may include: • pain that increases on weight bearing
activities (standing and walking, walking downstairs particularly) •
insidious onset of symptoms followed by progressive periods of relapses
and remissions • pain and stiffness in the morning • stiffness following
periods of inactivity • pain and stiffness that arise after unaccustomed
periods of activity • bony deformity (e.g. characteristic varus deformity
may follow from collapse of the medial compartmental joint space) •
reduction of the
joint space observed on X-ray, with bony outgrowths or osteophytes

ankylosing spondylitis mean : known as Bechterew's disease (or


syndrome) and Marie- Strümpell disease, is a chronic inflammatory
disease of the axial skeleton with variable involvement of peripheral
joints and non-articular structures Also Ankylosing is a term meaning
stiff or rigid and spondylitis means inflammation of the spine

Joints Types
Hip Joint Ball-and-Socket
Knee joint Bi-condylar joint
Ankle Joint hinge joint
Sterno-clavicular Joint Saddle Joint
Acromio-clavicular Joint Plane Joint
Shoulder/Gleno-humeral Joint Ball and Socket Joint
Elbow Joint Hinge Joint
Wrist/Radio-carpal Joint Condyloid Joint Inercarpal and Midcarpal joint
Plane joints
Carpometacarpal Joint of the thumb Saddle
Carpometacarpal joint of the fingers Plane joints

Planes and Axes

423-Abduction to Adduction in which plan occurs


a- From sagittal plan to coronal plan
d- From coronal plan to sagittal plan
d- From longitudinal to transverse

The correct Answer is:


A- from sagittal plan to coronal plan
424- Internal rot & ext. rot on which plane can performed
a- Longitudinal plane around horizontal axis
b- Axial plane around vertical axis

The correct Answer is:


B- Axial plane around vertical axis as we described above.

425- In horizontal axis which divided into caudal and cranial


A) sagittal
B)coronal
C)axial
D)transverse

The correct Answer is:


D- Transverse

426- When you examine shoulder joint by asking patient to abduct


shoulder to 90 degrees then lower slowly, this is:
a- Codman’s Test
b- Infraspinatus test

The correct Answer is:


A- Codman’s Test (for rotator cuff tear) also called (drop arm test)
427-Patient stay at hospital treated by corticosteroid then go to
physiotherapist, this patient will have problem it is
a- myopathy
b- osteoporosis
c- depression
d- decrease appetite

The correct Answer is:


B- Osteoporosis

428-patient has amputation and use artificial limb during walking he


takes abduction gait: this may due to
a- limb is high
b- stress on adductor longus muscle
c- medial aspect is short
d-tight of gluteus medius

The correct Answer is:


D- tight of gluteus medius

429- when examine patient for carpal tunnel syndrome, which nerve do
you examine
a- Radial n.
b- Ulnar n.
c-Median n

The correct Answer is:


C- Median nerve

430- Disease in children and early childhood due to malfunction of


calcium phosphorus metabolism
a-rickets
b-osteomyelitis

The correct Answer is:


A- Rickets

431-Most compression site of ulnar nerve in


a-shoulder joint
b-spinal groove of humerus
c-elbow
d-wrist

The correct Answer is:


C- Elbow

432-In carpal tunnel syndrome the entrapped nerve is the?


a-radial
b-median
c-ulnar
d-axillary.

The correct Answer is:


B- median nerve

433-which muscle doesnt affected in carpal tunnel syndrome


a-flexor pollicis brevis
b-adductor pollicis longus
b-abductor pollicis bervis

The correct Answer is:


B- Adductor policies longus The muscles that might be less affected by
CTS, the "LOAF" muscles...lumbricals, opponens (opposes thumb to
other fingers), abductor pollicis brevis (thumb abduction), and flexor
pollicis brevis (flexes thumb).
**5th finger abduction and adduction of the thumb are mediated by ulnar
nerve muscles

434-which movement not accompany with hyper extension of cervical


region:
a-poking chin
b-decrease flexion of lower cervical region
c-increase activity of SCM AND levator scapulae

The correct Answer is:


B-decrease flexion of lower cervical region

435-what is the best position for artificial hip ankylosis


A-Flex 20 , abd 10, ext rotation 5
B-No flex , abd 10, ext rotation 5
C -Abd 5 , ext rotation 10

The correct Answer is:


A- Flexion 20, abd 10, External Rotation 5, Surgical Considerations:
Arthrodesis, also known as artificial ankylosis
- position of hip fusion:
- neutral abduction, external rotation of 0-30 deg &, 20-25 -deg of
flexion;
- avoid abduction and internal rotation;
- this position is design to minimize excessive lumbar spine motion

436- Position of hip arthroplasty :


a- abd. 30 with flex. Maintained at 70_90
b- Abd. 20 with flex. Maintained at 70_90
c- abd.10 with flex maintained at 70_90
d- abd.30 with flex. Maintained at 20_90

The correct Answer is:


C- Abduction 10 with flexion maintained at 70 to 90 degree.

437- Patient with depression of metatarsal pad and claw toe the clinical
picture:
a- Hyperext of metatarsal bone and ext of I.P.JT
B- FLEXION of metatarsal bone and ext of I.P JT
C- ABDUCTION OF metatarsal bone AND FLEXION I.PJT
D- hyper ext of metatarsal bone and flexion of I.P JT
The correct Answer is:
D- Hyper extension of metatarsal bone and flexion of IP joint • clawing
(hyperextension of the metatarsophalangeal joints and flexion of the
other phalanges)
• Mallet toe (flexion of the distal interphalangealjoints)
• hammer toe (hyperextension of the metatarso-phalangeal and flexion of
the proximal interphalangeal joint)
• Hallux valgus (lateral deviation of the first interphalangea
• Hallux rigidus (stiffness of the first interphalangeal

438-WHICH STATEMENT NOT CORRECT ABOUT TALIPES EQUINO


VARUS
A-club foot
b- maybe congenital
c- P,F and inversion
d. D.F and eversion

The correct Answer is:


D- Dorsi flexion and eversion isn`t Talipes equino varus

439- pt. is referred to physiotherapist with diagnosis of flexion deformity


of Rt knee with examination we will found
A flexion knee +planter flexion +shortening of qudricepes
B flexion knee +dorsi flexion +shortening of hamstring
C flexion knee +no ankle changes +shortening of quadriceps
D flexion knee +planter flexion +lenghing of soles
The correct Answer is:
B- flexion knee + Dorsi flexion + shortening of hamstring

440-patient with winged scapula and excessive internal rotation needs to


a- stretch middle and lower trapezius
b- Stretch pectorals major and strength middle trapezius
c- strength pectorals major and Stretch upper trapezius

The correct Answer is:


B- Stretch pectorals major and strength middle trapezius

441-patient has scoliosis c on right thoracolumbar we found


a-lat flexion of thoracolumbar on right convex
b-lat flexion and left convex
c-lat flex and concave on left side

The correct Answer is:


A& C - lateral flexion of thoracolumbar on right convex with concave on
left side

442-anterior taleofibular ligament to be assessed resist


a- planter flex + inversion
b- dorsi + inv
c-dorsi + eversion
d- plant + ev

The correct Answer is:


A- Planter flexion + inversion it`s Anterior drawer test Patient lies supine
with their foot relaxed and the examiner stabilizes the tibia/fibula with
the foot in 20 degrees of
plantar flexion. The examiner grabs the ankle and draws the talus
forward in the mortise. Inversion gives an anterolateral stress with
increased stress on the
ATF.

443-What ligament is most involved in sustaining the longitudinal arch of


the foot?
A-Plantar calcaneonavicular ligament
b- Long plantar ligament
c- Plantar calcaneocuboid ligament
d- Anterior talofibular ligament

The correct Answer is:


A- Planter calcaneo-navicular ligament

444-Which of the following is not an example of a synarthrodial joint in


the body?
a- Coronal suture
b- The fibrous joint between the shaft of the tibia and fibula
c- Symphysis pubis
d- Metacarpophalangeal

The correct Answer is:


D- Metacarpophalangeal joint is a condyloid joint , but the synarthodial
joint is a fibrous joint which mean A union of two bones by fibrous tissue
such that there is no joint cavity and almost no motion possible; the
types of fibrous joints are sutures, syndesmoses, and gomphoses.
Synonym(s): immovable joint,

:The symptoms are variable and will present themselves differently from
patient to patient. The main symptoms begin with a generalised burning
pain; this is usually
followed by changes in the condition of the skin, which may become
shiny.
In severe cases, the affected body part may swell and, due to
sympathetic nervous system dysfunction, the body part may perspire
more than usual. Because of the pain the patient may not want to move
the injured part. This leads to muscle wastage and a viscious cycle
where stiffness and pain become worse. If the condition persists there
may be adverse changes to the condition of the underlying bone.
Ç
445-Ice massaging is contraindicated with:
a- Inflammation
b- Raynaud’s disease
c- Muscle spasm
d- Acute burn

The correct Answer is:


B- Raynaud`s disease

446-A patient with Osgood-Schlatter disease will experience the most


difficulty and pain while:
a- playing basketball or volleyball
b- sitting for prolonged periods of time
c- walking down stairs.
d- lifting heavy weights

The correct Answer is:


A- playing basketball or volley ball Osgood –Schlatter disease or
syndrome
(also known as Apophysitis of the tibial tubercle, or knobby knees) is an
irritation
of the patellar ligament at the tibial tuberosity.
It is characterized by painful lumps just below the knee and is most often
seen in young adolescents. Risk factors may include overzealous
conditioning (running and jumping), but adolescent bone growth is at the
root of it

447-A teenager comes to an outpatient facility with complaints of pain at


the tibial tubercle when playing basketball. The therapist notices that
the tubercles are abnormally pronounced on bilateral knees. What
condition does the patient most likely have?
a- Jumper’s knees.
b- Anterior cruciate ligament sprain.
c- Osgood-Schlatter disease
The correct Answer is:
C- Osgood- Schlatter disease

448-A tennis player receives a surgical repair of the annular ligament.


Where should the therapist expect to note the most edema?
a- Radial ulnar joint
b-Olecranon bursa
c-Ulno-humeral joint
d-Lateral triangle

The correct Answer is:


D- Lateral triangle.

450- A patient is referred to physical therapy with a ruptured Achilles


tendon. The patient reports that he was moving the lawn and was going
down a steep incline when he felt a sharp, sudden pain in the left heel
region. Which of the following would be proper location for the Achilles
tendon insertion?
a- Talus
b- Calcaneus
c- Cuboid
d- Navicular

The correct Answer is:


B- Calcaneus

449-foot deformity

hallux valgus-

451-What is true about this muscle?

A- This is biceps femoris ms. &supplied by tibial nerve


B- adductor brevis muscle
C- sartorius .
The correct Answer is:
A- This is biceps femoris ms. &supplied by tibial nerve

The talonavicular (blue arrow),


subtalar (red arrow) and
calcaneocuboid (green arrow) joints

453- Patient has chondromalacia contraindicated to use?


a- mobalization
b- taping , bandage
c- strengh hamstring

The correct Answer is:


A- mobilization.

454-Which of the following articulate with the second cuneiform?


a-Navicular
b-Talus
c-First metatarsal
d-Cuboid

The correct Answer is:


A- Navicular

455-patient come with sever elbow pain we can use


a- Heat application
b- Cold application
c- Faradic stimulation

The correct Answer is:


B- Cold application if acute trauma use cold & if chronic use heat & if
acute inflammation so use cold

456-Closed pack position of knee is


a- ext. knee
b- flex knee
c- ext knee with lat tibial rotation

The correct Answer is:


C- Extension knee with lateral tibial rotation
457-What is the closed-packed position of the shoulder?
a. Internal rotation and abduction
b. External rotation and abduction
c. Internal rotation and adduction
d. External rotation and adduction

The correct Answer is:


A- internal rotation and abduction

Neurology & pediatric

458-when you evaluate a CVA patient what is the first thing to notice
A) imbalance and incoordination
B) visual fields
C) can`t express himself

The correct Answer is:


C- can`t express him self

459-When instructing the family of a 9 year-old boy with Duchene


muscular dystrophy,
the main emphasis in the lower extremities should be placed on:
a- strengthening the knee extensors and plantar flexors
b- strengthening the plantar flexors and stretching the hip extensors
c- stretching the hip flexors and plantar flexors
d- strengthening the hip flexors and knee extensors

The correct Answer is:


D- Strengthening the hip flexor and knee extensors

460-Normal action potential


a-30 to – 60 mVolt
b- 60 to – 90 mVolt
c- 30 to – 90 mv
d- 30 to – 80 mv

The correct Answer is:


B- -30 to 90 mv

461- Pt with neurological condition with evaluation found is unable to


perform movement after all repetition, with adequate rest period comes
normal this pt is
A - mythenia graves
B -myopathy
C- hyprertonia

The correct Answer is:


A – Myathenia graves

462-pt with neurological condition with evaluation found is unable to


perform movement without repetitions:
A- myathenia graves
B- myopathy
C- hyprertonia

The correct Answer is:


B- myopathy

463- Polio virus transfer through :


a- Respiratory tract
b- Skin
c- Hepatic failure
d-GIT

The correct Answer is:


B- GIT

464-Post polio clinical features


1 -paralysis
2 -weakness
3 -atrophy
4- no change

The correct Answer is:


3- Atrophy

465-The most functional way to teach an individual with T4 complete


paraplegia to transfer from wheelchair to mat is by using a:
A- stand pivot technique
B- squat pivot technique
C- sliding board
D- back-out technique

The correct Answer is:


B- Squate pivot technique

466-Convalescent stage of polio :


a - 2weeks-2years
b- 2weeks-4years
c- 0-2week
d- 2week to through life

The correct Answer is:


C- 2 weeks – 2 years
Polio The disease may be staged
as:- Stage 1: Acute stage of paralysis: : it begins with fever and
headache,
followed by neck stiffness and meningitis. Muscles are painful and
tender. Paralysis soon follows and reaches its maximum in 2-3 days.
Limbs
are weak and there may be difficulty with breathing and swallowing. If
the
patient does not succumb to respiratory failure, pain and pyrexia
subsides after
7-10 days and the patient enters the convalescent stage. (0 – 2 weeks)

Stage 2: Recovery/convalescent stage:


This stage is prolonged. The return of muscle power is most noticeable
during
the first 6 months, but there may be continuing improvement for up to 2
years. ( 2 weeks – 2 years )

Stage 3: Residual paralysis: Some cases do not progress beyond the


early stage
of meningeal irritation. In others, however recovery is incomplete and
the
patient is left with some degree of asymmetric flaccid paralysis or
muscle
weakness

467-A therapist is evaluating a patient in an intensive care unit. The


therapist notes no eye opening, no verbal response, and no motor
response. On the Glasgow coma scale, what is the patient’s score?
a- 0
b- 3
c- 5
d- 9

The correct Answer is:


A- zero

468-The therapist is evaluating a patient with left-side visual field


deficits in both eyes. A lesion at what location may cause this defect?
a. At the optic chiasm.
b. At the right side optic tract.
c. At the left side optic nerve
d. At the right side optic nerve.

The correct Answer is:


B- at the right side optic tract

469-Which of the following neural fibers are the largest and fastest?
a- C fibers
b-A fibers
c- A and C are equal
e-None of the above
The correct Answer is:
B- A fibers

470-A patient comes to the therapist because she has noted a


pronounced tuft of hair on the center of her spinal column in the lumbar
area. The therapist notes no loss in motor or sensory function. This
patient most likely has what form of spina bifida?
A-Meningocele
b-Meningomyelocele
cSpina bifida occulta
d-Syringomyelocele

The correct Answer is:


C- spina bifida occulta
471-The therapist receives an order to treat a 42-year-old man admitted
to the hospital 3 days ago with a stable wound to the left lower thoracic
spine. The patient is unable to move the left lower extremity and cannot
feel pain or temperature differences in the right lower extremity. What is
the most likely type of lesion?
a- Anterior cord syndrome
b- Brown-Sequad syndrome
C- Central cord syndrome
d- The patient is equally as likely to have anterior cord syndrome as he is
to have

The correct Answer is:


B- Browen-squad syndrome

Brown-Squad syndrome.

472-A therapist is testing key muscles on a patient who recently suffered


a spinal cord injury. The current test assesses the strength of the long
toe extensors. Which nerve segment primarily innervates this key muscle
group?
a-L2
b-L3
c-L4
dL5

The correct Answer is:


D- L5 dorsal root innervate extensors long toes

473- What is the major concern of the physical therapist treating a


patient with an acute deep partial thickness burn covering 27% of the
total body? The patient was admitted to the intensive care burn unit 2
days ago.
a- Range of motion
b- Fluid retention
c- Helping the family copes with the injured patient.
d- Home modification on discharge.
The correct Answer is:
A- Range of motion

474- Persuading a sedentary patient to become more active, the


therapist explains the benefits of exercise. Which of the following is an
inappropriate list of benefits?

a- Increased efficiency of the myocardium to obtain oxygen, decreased


high-density lipoprotein (HDL) and decreased cholesterol
b- Decreased low-density lipoprotein (LDL), decreased triglycerides, and
decreased blood pressure.
c- Increased efficiency of the myocardium to obtain oxygen, decreased
cholesterol, and decreased LDL.
d- Both B and C are inappropriate lists.

The correct Answer is:


A- increase efficiency of the myocardium to obtain oxygen , decrease
HDL & Decrease cholestrol

475-The therapist is evaluating a patient with a diagnosis of cerebral


palsy. The therapist notes that all of the extremities and the trunk are
involved. Further assessment also reveals that the lower extremities are
more involved than the upper extremities and that the right side is more
involved than the left. This patient most likely has which classification of
cerebral palsy?
a-Spastic hemiplegia.
b-Spastic triplegia.
c-Spastic quadriplegia.
d-Spastic diplegia.

The correct Answer is:


D- spastic diaplegia

476-On examination of a cross section of the spinal cord of a cadaver,


the examiner notes plaques. The finding is most characteristic of what
condition?
a- Parkinson’s disease
b- Myasthenia gravis
c- Multiple sclerosis.
d- Dementia.

The correct Answer is:


C- Multiple Sclerosis

477-While ambulating a stroke patient (right side is the involved side),


the therapist notes increased circumduction of the right lower extremity.
Which of the following is an unlikely cause of this deviation?
a- Increased spasticity of the right gastrocnemius.
b- Increased spasticity of the right quadriceps.
c- Weak hip flexors.
d- Weak knee extensors

The correct Answer is:


D- weak knee extensors

478- Which of the following is the most important to assess first during
an evaluation of a patient with a recent stroke?
a- Sensory status
b- Motor control
c- Mental status.
d- Ambulation potential

The correct Answer is:


C- Mental status

479- A posterior lateral herniation of the lumbar disc between vertebrae


L4 and L5 most likely results in damage to which nerve root?
a-L4
b-L5
c-L4 and L5
d-L5 and S1
The correct Answer is:
B- L5

480-A 31-year-old man has loss of vision in one eye, staggering gait,
numbness in bilateral upper extremities, and decreased bowel and
bladder control. The episodes of the above symptoms have occurred
every few weeks for the past 6 months. Each episode has been slightly
worse than the first. What is the most likely condition?

a-Parkinson’s disease
b- Guillain Barre syndrome
c- Multiple sclerosis
d- Amyotrophic lateral sclerosis
The correct Answer is:
C- Multiple Scelrosis

481- The use of compression stockings on the feet and ankles is


contraindicated in which patient population?
a- Chronic venous disease
b- Recent total knee replacement
c- Burn patients
d- Chronic arterial disease

The correct Answer is:


D- Chronic arterial disease

482- A 68-year-old man who suffered a stroke 4 weeks ago (involving the
dominant hemisphere) presents with contralateral hemiparesis and
sensory loss (greater in the lower extremity than in the upper extremity),
mental confusion, and aphasia. What is the most likely location of the
infarction?
a- Middle cerebral artery
b- Internal carotid artery
c- Posterior cerebral artery
d- Anterior cerebral artery

The correct Answer is:


D- Anterior cerebral artery

483- To decrease the risk of hypoglycemia in a patient with type I insulin-


dependent diabetes, which of the following is inappropriate?
a- Eat or drink a snack high in carbohydrates 30 minutes before
exercises.
b- Exercise muscles that have not had an insulin injection recently.
c- A carbohydrate snack for each 30-45 minutes of exercise
d- Exercise at the peak time of insulin effect

The correct Answer is:


D- Exercise at the peak time of insulin effect

484-A patient presents to an outpatient physical therapy clinic with a


140o kyphoscoliosis curve. What is the therapist’s greatest concern?
a-The patient’s complaint of low back pain
b-Gait deviation.
c-Pulmonary status.
d-Poor upright standing posture

The correct Answer is:


C- Pulmonary status

485-A 35-year-old woman suffered brain injury in a motor vehicle


accident and presents with the following symptoms: an intention tremor,
nystagmus, hypotonia, and dysdiadochokinesia. What is the most likely
location of the lesion?
a- Basal ganglia
b- Dorsal columns
c- Frontal lobe
d- Cerebellum

The correct Answer is:


D- Cerebellum – Parkinsonism

486- A 3-month-old infant has a heart condition known as tetralogy of


Fallot. This condition presents with which of the following signs?
A. Atrial septal defect, pulmonary valve stenosis, aorta abnormally
located to the right, and right ventricular hypertrophy.
B. Atrial septal defect, pulmonary valve stenosis, aorta abnormally
located to the left, and right ventricular hypertrophy
C. Ventricular septal defect, pulmonary valve stenosis, aorta abnormally
located to the right, and right ventricular hypertrophy.
D-Ventricular septal defect, pulmonary valve stenosis, aorta abnormally
located to the right, and left ventricular hypertrophy

The correct Answer is:


C- ventricular septal defect , pulmonary valve stenosis , Aorta abnormally
located to the right and right ventricular hypertrophy.

487- A 52-year-old man with sciatica presents to outpatient physical


therapy. The patient indicates that he is experiencing paresthesia
extending to the left ankle and severe lumbar pain. Straight-leg-raise test
is positive with the left lower extremity. Of the following, which if the
most likely source of pain?
a- A lumbar disc with a left posterior herniation or protrusion
b- Piriformis syndrome
c- Sacroiliac joint dysfunction
The correct Answer is:
A- A lumbar disc with a left posterior herniation or protrusion If the
patient experiences sciatic pain when the straight leg is at an angle of
between 30 and 70 degrees, then the test is positive and a herniated disc
is likely to be the cause of the pain.

488-The following is a long-term goal for a patient with spinal cord injury:
independence in performing a manual cough without applying pressure to
the abdomen. This goal is the most challenging and obtainable for a
patient with a complete Lesion at which of the following spinal cord
levels?
a-C5
b-C7
c-T2
d-T10

The correct Answer is:


A- C5

489-The therapist observes a patient with the latter stages of


Parkinson’s disease during ambulation. Which of the following
characteristics is the therapist most likely
observing?
a- Shuffling gait
b- Increased step width
c- Difficulty initiating the first steps
d- A and C

The correct Answer is:


D- A& c shuffling gait & difficulty initiating the first steps

490-An infant with Erb’s palsy presents with the involved upper extremity
in which of the following positions?
a- Hand supinated and wrist extended
b- Hand supinated and wrist flexed
c- Hand pronated and wrist extended
d- Hand pronated and wrist flexed

The correct Answer is:


D- hand pronated and wrist flexed
491-Hand During the evaluation of an infant, the therapist observes that
with passive flexion of the head the infant actively flexes the arms and
actively extends the legs. Which of the following reflexes is being
observed?
a-Protective extension
b-Optical righting
c-Symmetrical tonic neck
d-Labyrinthine head righting

The correct Answer is:


C- Symmetrical tonic neck

492- A therapist is obtaining a subjective history from a new patient


diagnosed with right-side hemiplegia. The therapist notes that the
patient is able to understand spoken language but unable to speak well.
Most of the patient’s words are incomprehensible. The patient also has
difficulty in naming simple objects. What type of aphasia does the patient
most likely have?
a-Anomic aphasia
b-Broca’s aphasia
c-Crossed aphasia
d-Wernicke’s aphasia

The correct Answer is:


B- Brocha`s aphasia

493-A 65-year-old man presents to physical therapy with complaints of


pain due to compression fractures of the C2 and C3 vertebrae. The
patient has an unusually large cranium. He describes his condition by
stating, “Much of my bone tissue is continually decreasing, then
reforming”. The patient also indicates that the condition has caused limb
deformity. Which of the following diseases does he have?
a- Paget’s disease
b- Achondroplastic dwarfism
c- Osteogenesis imperfecta
d- Osteopetrosis

The correct Answer is:


A- Paget`s disease

494- A patient informs his therapist that his problem began 3 months
after a bout of the flu. The patient originally experienced tingling of the
hands and feet. He also reports progressive weakness to the point that
he required a ventilator to breathe. He is now recovering rapidly and is
expected to return to a normal functional level in 3 more months. From
which of the following conditions is the patient most likely
suffering?
a- Parkinson’s disease
b- Guillain - Barrè syndrome
c- Multiple sclerosis
d- Amyotrophic lateral sclerosis

The correct Answer is:


B – Guillain – Barre` syndrome

495-A therapist is evaluating a patient in the intensive care unit. The


therapist notices that the patient is moving his hands and fingers in
slow, writhing motions. Which of the following terms best describes this
type of movement?
a-Lead-pipe rigidity
b-Ballismus
c-Chorea
d-Athetosis

The correct Answer is:


D- Athetosis

496-An infant is being examined by a physical therapist. The therapist is


resisting movement of the right upper extremity and notices involuntary
movement of the left upper extremity. Which of the following is displayed
by the infant?
a- Landau reaction
b- Startle reflex
c- Moro reflex
d- Associated reaction

The correct Answer is:


D- Associated reaction

497-A 32-year-old construction worker fell off a ladder. In his effort to


prevent the fall, the worker reached for a beam with his right arm. This
motion stretched the brachial plexus, resulting in decreased function in
the right arm. Full function returned after 2 ½ weeks. What is the most
likely type of injury?
a-Axonotmesis
b-Neurotmesis
c-Neurapraxia
d-None of the above

The correct Answer is:


C- Neurapraxia

498-Twenty-four hours following a major thermal burn, a patient


experiences syncope while the physical therapist works on positioning
as part of edema management. The syncope is most likely a result of
a- orthostatic hypotension
b- extreme pain
c- intravascular hypovolemia
d- massive infection

The correct Answer is:


C- Intravascular hypovolmia

499-Just after birth, you observe a full-term infant in the neonatal


intensive care unit. In the supine position, the shoulders are abducted
and externally rotated, elbows and fingers are flexed, hips are abducted
and externally rotated and knees are flexed. This posturing would seem
to indicate that the:
a-upper extremity tone is abnormal
b-lower extremity tone is abnormal
c-tone is abnormal in both upper and lower extremities
d-tone is normal in both upper and lower extremities

The correct Answer is:


D- Tone is normal in both upper and lower extremities

500-A child with spastic diplegia is now independent in using a walker.


Additional goals desired by the physical therapist include increasing the
child’s energy-efficiency and velocity while ambulating. In this case, the
selection of walker that is most likely to help improve these factors is:
a-a standard anterior walker with no wheels
b-an anterior rollator walker with two wheels
c-a posterior rollator walker with two wheels
d-a posterior rollator walker with four wheels
The correct Answer is:
C- a posterior rollator walker with two wheels Facilitate more upright
posture
Decrease hip and trunk flexion during stance phase, Decrease double
stance time, Increase walking velocity, Increase stability and interaction
with other children

501-In poliomylitis which is incorrect


A. Hyper reflex
B. Reflex absent
C. Ms weakness

The correct Answer is:


A- hyper reflex

502- Evaluating the gait of a patient with left hemiplegia, you note toe
drag during
mid swing on the left. The least likely cause of this deviation would be:
a- inadequate concentric activity of the ankle dorsiflexors
b- excessive extensor synergy
c- knee and ankle joint pain
d- decreased proprioception

The correct Answer is:


C- knee and ankle joint pain

503-all the following is true about motor neuron disease except


A- Affect anterior horn cell in spinal cord and cause lower motor neuron
lesion
B- Affect corticospinal tract and cause upper motor neuron Lesion
C- Affect cranial nerve and Most common facial nerve affected with
lower motor neuron lesion
d- Affect brain stem and cause upper motor neuron Lesion and cause
lower motor neuron lesion

The correct Answer is:


D- affect brain stem and cause upper motor neuron lesion and cause
lower motor neuron lesion

Ñß Ò Ý í Çá Õ æÑ ÉÚÔÇä ÊÞÏÑ ÊÍá Ãí ÓÄÇá Òí503


504-Hemiplegia is :
a) Loss half field of vision
b) Paralysis halve of the body

The correct Answer is:


B- Paralysis halve of the body

505-lt cva pt has homonymous hemianopia .what can we do to


compensate this problem
A. Help him to sleep on right side
B. Teach him to turn toward right side
C. Set a person to remind him to turn his head to right
D. Put food table ,telephon, ...on right side

The correct Answer is:


D- put food table , telephone , on right side

506- Which of the following is the most important to assess first during
an evaluation of a patient with a recent stroke?
a- Sensory status
b- Motor control
c- Mental status.
d- Ambulation potential

The correct Answer is:


C- Mental status

507-In yellow and red flags which of the following not considered as red
flag in these
situations:
a- history of carcinoma
b- psychosocial factor
c- bowel problems
d- excess weight loss

The correct Answer is:


B- Psychosocial factor is a yellow flags
'Yellow flags' are psychosocial factors including a previous history of
anxiety and depression, impending compensation, absence from work,
sickness benefit, invalidit benefit, passivity and high levels of
dependency and poor coping skills.
'Red flags' are clinical features that should alert the therapist to the
possibility of severe pathology. The include bladder and bowel
malfunction, saddle anaesthesia, bilateral paraesthesia, neurological
signs, unexplained weight loss, a past history of
carcinoma, general debility and fever.

508-Acute Inflammatory Demyelinating Poly radiculo neuropathy is also


referred as
a- Guillain–Barré syndrome
b- Compressive myelopathy
c- Friedreich's ataxia
d- None of the above

The correct Answer is:


A- Guillain Barre syndrome.

509- Facial nerve nuclei innervations:


a- Bilateral pyramidal innervations
b- Unilateral pyramidal innervations
c- Bilateral no pyramidal innervation

The correct Answer is:


A- Bilateral pyramidal innervations.

510-patient suffer from double vision means


a- hemianopia
b- anomia
c- diplopia
d- agnosia

The correct Answer is:


C- Diplopia is meaning double vision

511-Meaning of rigidity is :
a- decreased ROM
b- Muscle stiffness

The correct Answer is:


B- Muscle stiffness

512-When evaluating Parkinson’s pt you observed uniform muscle


resistance; this means
a- cogwheel rigidity
b- lead-pipe rigidity
c- spasticity

The correct Answer is:


B- Lead-pipe rigidity
513- Parkinsonism disease is :
a- cerebral cortex
b- basal ganglion

The correct Answer is:


B- basal ganglion

513-Parkinson’s gait, which is not true


A-Wide base of support with arm swing
B-Propulsion gait
C-Festinating gait
D-Retropulsion

The correct Answer is:


A- wide base of support with arm swing

514-when evaluating Parkinson’s gait ;


a- festinating and shuffling gait
b- short BOS

The correct Answer is:


A- Festinating & shuffling gait

515- A patient with spastic left hemiplegia experiences recurvatum


during stance phase. If the patient is using an ankle-foot orthosis, the
cause of the problem might be attributed to
A- not enough limitation by the posterior stop.
B- not enough limitation by the anterior stop.
C- too much limitation by the posterior stop.
D- too much limitation by the anterior stop.

The correct Answer is:


A- not enough limitation by the posterior stop
516 -In ttt of Parkinson pt :
a- strengthening of elongated muscles and stretching of rigid flexors
b- strengthening of elongated muscles and stretching of rigid extensors
c- strengthening of flexors & extensors
d- none of the above .

The correct Answer is:


A- Strengthening of elongated muscle and stretching of rigid flexors.

517-Pt close his eyes and move his shoulder and then ask him about
degree and position of limb so we test
a-light touch
b-fine touch
c-proprioception

The correct Answer is:


C- Proprioception

518- Prior to performing manual traction in the cervical region, one


should perform a test for vertebral artery insufficiency in order to
ascertain the adequacy of blood supply to the:
a- vertebrae.
b- Upper extremities.
c- Spinal canal.
d- brain stem.

The correct Answer is:


D- brain stem is the most important to guarantee there is adequate blood
supply to it.

clumsy syndrome
*Dyspraxia is the partial loss of the ability to coordinate and perform
skilled, purposeful movements and gestures with normal accuracy
* Apraxia is the term that is used to describe the complete loss of this
ability

519- All of these are non-equilibrium-coordination tests except


a-finger to finger
b-jumping
c-grasp
d-jogging
The correct Answer is:
A- Finger to finger is equilibrium – coordination test

520-A therapist receives an order to evaluate and treat a 76-year-old


woman
who was involved in a motor vehicle accident 2 days ago. The patient’s
vehicle was struck in the rear by another vehicle. The patient has normal
sensation and strength in bilateral lower extremities but paralysis and
loss of sensation in bilateral upper extremities. Bowel and bladder
function are normal. The patient most
likely has what type of spinal cord injury?
a- Anterior cord syndrome.
b- Brown-Sequard syndrome.
c- Central cord syndrome.
d-There is no evidence of an incomplete spinal cord lesion
The correct Answer is:
C- Central cord syndrome.

521-Patient has C V A and middle cerebral artery affected which part will
have the best complete recovery
a-shoulder
b-elbow
c-hand
d-hip

The correct Answer is:


D- hip because the MCA affected the UL more than Lower Limb

522-which is not correct of spina bifida


a- motor paralysis according to level of lesion
b- commonly occur thoracic spine
c- due to decrease folic acid
d- detected by sonography

The correct Answer is:


B- commonly occur thoracic spine ,, it`s commonly occure in Lower back
bone

http://www.spinabifidaassociation.org/.../What_is_Spina... ÏÇ ÇáãÕÏÑ ááì


ÈíÞæá Çä ÇáÝæáíß ÇÓíÏ ãÇáæÔ
ÚáÇÞÉ ÈÇáÓÈíäÇÈíÝíÏÇ
523- The spinal cord...
a- Occupies the lumbar cistern
b- Has twelve (12) cervical segments
c- Contains the cell bodies of postganglionic sympathetic efferent
neurons
d-Ends at the conus medullaris

The correct Answer is:


D – Ends at the conus medullaris

524-Myotome is :
a- muscle supplied by certain nerve
b- skin supplied by certain nerve

The correct Answer is:


A- muscle supplied by certain nerve

525-Dermatome is :
a- muscle supplied by certain nerve
b- skin supplied by certain nerve

The correct Answer is:


B- Skin supplied by certain nerve

526-Electrically insulating material in a nerve fibre is


a- Nodes of Ranvier
b- Axon
c- Dendrites
d-Myelin sheath

The correct Answer is:


D- Myelin Sheath

527-myotome of this biceps muscle is


A- C3-C4
B- C5- C6
C- T1- T2

The correct Answer is:


B- C5 – C6
528-To prepare a patient with a cauda equina lesion for ambulation with
crutches, the upper quadrant muscles that would be most important to
strengthen would be the
A-upper trapezius, rhomboids and levator scapulae.
B-deltoid, coracobrachialis, and brachialis.
C-middle trapezius, serratus anterior, and triceps.
D-lower trapezius, latissimus dorsi and pectoralis major

The correct Answer is:


D- Lower trapezius, Latissimus dorsi and pectoralis major

529- While observing a patient with posttraumatic brain injury (TBI), the
therapist notes an increase in left ankle plantar flexion during loading
response (heel strike to foot flat) of the involved lower extremity. With
this particular patient, the left side is the involved side. Which of the
following is not likely cause of this deviation?
a-Spasticity of the left gastrocnemius
b-Hypotonicity of the left tibialis anterior.
c-Leg length discrepancy.
d-Left quadriceps hypertonicity

The correct Answer is:


C- Leg length discrepancy

530-pt sufrring from weakness of biceps brachials the corresponding


myotome is
A -c4
B -c5
C -c6

The correct Answer is:


B- C5

531– pt has lumber pain radiated to dorsum of foot


a) L1 , L2
b) L4.L5
c) S1.S2

The correct Answer is:


B- L4-L5
532-Superfacial upper abdomen reflex is
a) T7 to t9
b) T11 to t12
c) L1 , l2

The correct Answer is:


A- T7- T9

533-Level of superficial abdominal reflex


a.T10 – T11
b. L1 – L2
c. L2 – L3

The correct Answer is:


A- T10 – T11

534-Patient complaining from back pain, with examination you find


decreased sensation over big toe what level of lesion would you expect
a- L 3 -L4
b- L4 – L5
c- L5 – S1

The correct Answer is:


B- L4- L5

535- while you examine a patient you find decreased tendon jerk at
Achilles tendon,
what is the level of lesion
a- L4 –L5
b- L5 –S1
cs1 – S2

The correct Answer is:


C- S1-S2

536-Pt with sci there is weakness in his lower limb to test him
A. Give resistance to ms at middle ROM
B. Test hip flexors L1-L2
C. Hip ext L3

The correct Answer is:


A- give risistance to MS at middle ROM
537-A physical therapist is attempting to explain the importance of slow
stretching to an athlete training to compete in a marathon. The therapist
explains that quick stretching often causes the muscle to which is a
response initiated by the which are located in the muscle fibers. Fill in
the blanks.
a-Relax – Golgi tendon organs
b-Contract – Golgi tendon organs
c-Relax – muscle spindles
dContract – muscle spindles

The correct Answer is:


D- contract – muscle spindles

538-patient with neck pain and he could not extend his elbow because
it`s painful this pain refered to
A- C5
B- C6
C- C 7
D- C8

The correct Answer is:


B- C6

539- -Dermatome of letter B

a. L3
b. L4
c. L5
d. S1

The correct Answer is:


B- L4
540- for relaxation of parkinson's patient and to facilitate ex .which is
correct
A. Rhithmic initiation
B. Rhithmic stabilization
C. Sit pt on rocking chair
The correct Answer is:
A- Rhythmic initiation
Rhythmic initiation - parkinsonism
Rhythmic stablization - increase stability( Attaxia )

541- for relaxation of parkinson's patient and to facillitat ex which


incorrect :
A. Rhithmic initiation
B. Rhithmic stabilization
C. Sit pt on rocking chair
The correct Answer is:
B- Rhythmic stabilization

542-Ms spindle .which is incorrect?


A. Has capsule for each spindle
B. Detect any tension in ms
C. Parallel with extrafusal ms fiber

The correct Answer is:


B- Detect any tension in muscle is incorrect for muscle spindle it`s
correct about Golgi tendon organs

For remember :
Golgi Tendon organs has a T so we can remember it`s action about
tension of muscle
Muscle spindle has L so we can remember it`s action for Length

543-which statement is incorrect about golgi tendon organ


a- it monitor muscle length
b- its affected by muscle contraction
c- its more scattered in tendon
d- its in the same series of muscle fibers

The correct Answer is:


A- it monitor muscle length is incorrect it`s correct about muscle spindle
544-muscle spindle is stimulated by
a-Passive stretch
b-Passive movement
c-Active exercises

The correct Answer is:


A- passive stretch

545-the name of muscle cell is


a-sarcomer a= sarcolemma
b-actin
c-myosin
d-no name

The correct Answer is:


A- Sarcomera = sarcolemma

546-Anterior horn cell in gray matter not include


a- ventral root
b- alpha motor neuron
c- gamma motor neuron
d- sensory neuron

The correct Answer is:


D- sensory neuron
547-Brain consist of
a- two part
b- Three parts
c- Four part
d- Multipart

The correct Answer is:


A- Two part

548-Motor area
A-Control motor activity of opposite side of the body
B-Control motor activity of same side
C-Receive sensation

The correct Answer is:


B- Control motor activity of same side
549- Choronaxiae definition:
A- Minimum time to depolarize the nerve when current intensity is twice
the rheobase
B- Minimum time to depolarize the nerve

The correct Answer is:


A- Minimum time to depolarize the nerve when current intensity is twice
the rheobase

550-pyramidal tract make decessation at:


a- medulla
b- midbrain
c- pons
d- cerebral cortex

The correct Answer is:


A- medulla

551-Patient with C.V.A damage LT ANTERIOR CEREBRAL ARTERY


AFFECT
A- contralateral leg
b- ipsilateral leg
c- ipsilateralarm

The correct Answer is:


A- Contralateral leg as It`s the most affected leg in ant. Cerebral Artery
Middle cerebral artery
(Supplies most of the convexity of the cerebral hemispheres)Dense
contralateral hemiplegia Contralateral homonymous hemianopia Cortical
type of sensory loss Speech problems in left hemisphere lesions, with
neglect ofcontralateral sideLesions in the right hemisphere result in
parietal damage, visuospatialdisturbances and left-sided neglect
Posterior cerebral artery Visual disturbance Contralateral homonymous
field defect Memory disturbance and contralateral sensory loss

Anterior cerebral artery


Contralateral monoplegia cortical sensory loss sometimes behavioral
abnormalities associated with frontal lobe damage

552- bradykinesia means


A- slowness of movement
b- fast movement
c- impaired limb sensation

The correct Answer is:


A- Slowness of movement

553-Difficult speech is called


A-Dysarthria
B-Dysphagia

The correct Answer is:


A- Dysarthria

554-The term dysapraxia refers to?


a- inability to perform purposfull movements
b- swallowig
c- slow motion
d- involuntary movement

The correct Answer is:


A- inability to perform purposfull movements

555-Bradyphrenia mean
A. slowness of thought
B. slowness of movement
The correct Answer is:
A- Slowness of thought, seen in Parkinsonism

556-The term aphasia refers to?


a- speech problem
b- swallowig
c- slow motion
d- involuntary movement

The correct Answer is:


A – speech problem

557- the marked impairment of production and comprehension of


language :
a- fluent aphasia
b- nonfluent aphasia
c- global aphasia

The correct Answer is:


C- Global aphasia

http://en.wikipedia.org/wiki/Iliofemoral_ligament Ïå ÑÇÈØ ãåã Úä åÐÇ ÇáæÊÑ

558-Which is incorrect about sciatic n


a- to test it by SLR
b - maximum stretch occur 90 flexion
c- to stretch tibial part .pronial part with invesion
d- to stretch tibial part .pronial part with &eversion

The correct Answer is:


D- to stretch tibial part, pronial part with eversion.

559-Burn affecting epidermis and superficial part of derms:


a- superficial partial thickness
b- Deep partial thickness
c- Full thickness

The correct Answer is:


B- deep partical thikeness
Partial thickness
1. In a superficial partial-thickness burn the epidermis is destroyed.
2. In a deep dermal burn both the epidermis and part of the dermis are
destroyed.
There are blisters, patches of white destroyed tissue, and red areas .
Sensation varies according to the depth of dermal damage and the
sensory nerve endings involved.

560-which is incorrect about burn in shoulder and pectoral region:


a- put airplane splint
b- put in ext and abduction
c- put in flex add int rot

The correct Answer is:


C- put in flex , add , int. Rot.
561- Burn at shoulder & arm avoid
a) Airplane splint
b) Abduction
c) Adduction & internal rotation

The correct Answer is:


C- Adduction & internal rotation

562-A 45 year old male presents to the burn unit with partial thickness
burns over the entire right arm, left arm, front of head, and front of chest.
Approximately
What percentage of his body is burned?
a-.31.5%
b-.36%
c-.40.5%
d-.45%

The correct Answer is:


A- 31.5 % due to the rule of 9 :
9 entire R Arm +9 entire L Arm + 4.5 front of the Head +9 front of Chest =
31.5

563-A therapist is asked to estimate the percentage of a patient’s body


that has been burned. The patient is a 32- year-old man of normal size.
Burns are located along the entire anterior surface of the face. The
patient also burned the entire anterior portion of the right upper
extremity in an attempt to guard himself from flames. Using
the rule of nines, what percentage of the patient’s body is burned?
a- 9%
b- 18%
c - 4.5% d- 27%

The correct Answer is:


A- 9 % as we described above.

ÞæÇÚÏ ÇáÍÑæÞ

564-A patient is referred to physical therapy services for care of a burn


wound on the left foot. The majority of the wound is anesthetic. There is
significant eschar formation over the dorsum of the involved foot, and
moderate subcutaneous tissue damage is present. What is the most
likely classification of this burn?
a- Electrical
b- Superficial partial thickness
c- Deep partial thickness
d- Full thickness
The correct Answer is:
D- Full thickness due to there is eschar
Full-thickness
The epidermis, dermis and other underlying tissues are destroyed. The
presenting surface may be black, white or yellow. It is inelastic and
unable to stretch (the
eschar). If the burn is circumferential (e.g. around the forearm, chest or
finger) the damaged skin can perform a tourniquet effect as swelling
develops. In these cases the tension must be released by longitudinal
incisions through the scar along its full length (escharotomy; Figure 5.2).
This procedure will be performed within the first hours of admission to
hospital. The skin is dead, therefore no analgesia is required
Erythema
The skin remains intact, the erythema lasts for a few days, and the
patient does not normally seek medical help unless the problem is
extensive, as can occur with sunburn.

Superficial : The tissue damage results in seepage of fluid in between the


layers of epidermis, causing a blister, which is surrounded by a dark red
erythema. Movement of
the burned areas can be very painful. Blisters will continue to appear
over the first 24 hours after burning
.
Partial thickness
1. In a superficial partial-thickness burn the epidermis is destroyed.
2. In a deep dermal burn both the epidermis and part of the dermis are
destroyed.
There are blisters, patches of white destroyed tissue, and red areas .
Sensation varies according to the depth of dermal damage and the
sensory nerve endings involved.

565-To prevent contracture in a newly admitted 8 year-old with anterior


neck burns, it would be best to position the neck in:
a- hyperflexion
b- slight flexion
c- neutral
d- extension
The correct Answer is:
D- Extension

566-Rolling begin at
a) 4 moths
b) 6 monthes

The correct Answer is:


A- 4 months

567-To test righting - equilibrium in child


A. Sudden push him from side
B. Turn his head
C. Tilt the child to one side

The correct Answer is:


C- tilt the child to one side

568-In normal milestone baby can jump by both legs in swing at


a- 12 months
b- 24 months
c- 18 months
d- 30 months

The correct Answer is:


D- 30 months

569-At any age the child can jumb in one leg holding on
a- 18 month
b- 24 month
c- 30 month

The correct Answer is:


C- 30 months can jump on 2 legs , from 2- 3 years can hopping using one
leg

Milestones devolving ages 100% sure


From supine to prone 3-4 manily to 4momth
From prone to supine 6months
Roll alone 6-8 months
Sit with support 5-7 months
Sit alone 6-8 months
Walk alone 12-15
Climb 18-24 depends holding rail or not
Jump bilateral 30 months
On one 3-4 years

570-child came to u with erb`s palsy c5, c6 diagnosed by erb engram


where is the affection
a- weakness of external rotators
b- Spasticity of external rotators
c- Upnormal pattern of movement + weakness of ant deltoid + biceps
spasm
d- Weakness elbow extensors
The correct Answer is:
C- Upnormal pattern of movement + weakness of anterior deltoid +
biceps spasm
Types of erb's palsy
Erb's child hasn't MORO Reflex
Erb's palsy or Erb–Duchenne palsy is a paralysis of the arm caused by
injury to the
upper group of the arm's main nerves, specifically the severing of the
upper
trunk C5–C6 nerves
Klumpke’s Palsy involves C7 and T1. There is weakness of the wrist
and
finger flexors and of the small muscles of the hand. Unfortunately, there
is no specific
treatment for this type of Erb’s palsy

571-child can kick the ball in


a- 12 month
b- 24 month
c- 30 month

The correct Answer is:


B- 24 month

572-infant with erbs palsy he can full recover with good biceps & deltoid
at:
a- 3 months
b- 6 months
c- 9 months
d-12 months
The correct Answer is:
D- 12 months

573-At any age the child can walk independably


a- 12 month
b- 18 month
c- 24 month
d- 30 month

The correct Answer is:


B- 18 months

574-An infant who can recognize tone of voice, sit independently and
pulls to stand through kneeling is most likely to be:
a- 2-3 Months of age
b- 4-5 Months of age
C-6-7 Months of age
d-8-9 Months of age

The correct Answer is:


D- 8-9 months of age

575-You have evaluated a nine month old who can’t assume or maintain
quadruped position without assistance his parent insist that child has
already begin to walk with assistive you suspect that what parents say is
a- protective ext. downward
b- spontaneous stepping
c- positive supporting reaction
d- negative supporting reaction

The correct Answer is:


B- Spontaneous stepping

576- What the most primitive reflex of human body.


a- Symmetrical tonic neck reflex
b- sucking reflex
c- asymmetrical tonic reflex
d- tonic lybrinthine

The correct Answer is:


B- Sucking reflex
577-baby supine lying abducting both shoulders with 90 elbow flexion
lower limbs extended and adducted at hips and extended at knees and
planter flexed ankles , what is this reflex
a- Tonic labrynthinen reflex
b- Moro reflex
c- STNR
d- ATNR

The correct Answer is:


C- STNR

578- An infant is being examined by a physical therapist. The therapist is


resisting movement of the right upper extremity and notices involuntary
movement of the left upper extremity .which of the following is displayed
by the infant?
A. Landau reaction
B. Startle reflex
C. Moro reflex
D. Associated reaction

The correct Answer is:


D- Associated reaction

579-neuron has no axon, no refractor organ but make synape with


second neuron at the ganglion is called
a- preganglionic neurone
b- postganglionic neurone
c- autonomic neuron.

The correct Answer is:


a- preganglionic neuron

580-The therapist is treating a track athlete who specializes in sprinting


and wants to increase his other speed on the track. To accomplish this
goal, the plan of care should include activities to develop fast-twitch
muscle fibers. Characteristics of this type of fiber include:
a- Fatigues slowly, fiber colors appear red and used more in aerobic
b- Fatigues quickly, fiber colors appear white and used in anaerobic
c- Fatigues quickly, fiber colors appear white and used more in aerobic
d- Fatigues slowly, fiber colors appear white, and used more in anaerobic
The correct Answer is:
B- Fatigues quickly, fiber colors appear white and used in anaerobic Fast
twitch muscles are good for rapid movements like jumping to catch a
ball or sprinting for the bus. They contract quickly, but get tired fast, as
they consume lots of energy.

581-first motor action could be seen in pediatric is


a-head control
b-righting reaction
c-crawling
d-rolling

The correct Answer is:


A- head control

582-Rickets is due to deficiency of vitamin


A -C
B -B
C -K
D –D

The correct Answer is:


D- Vitamin D

583-Spastic diplegia means


A-primarily affecting the legs but arms are less involved
B-only legs are involved
C-only arms are involved

The correct Answer is:


B- Only legs are involved

584- In ttt a 5 years with spastic diplegia as part of the program


relaxation might be accomplish
A. Use of rhythmic stabliztion
B. slow rocking
C. inhibation of parasympathitic
D. facilitation of sympathetic

The correct Answer is:


B- Slow rocking
585-Hypotoncity means
a- decrease tone in muscle
b- Increase tone
c- incoordination

The correct Answer is:


A- Decrease tone in muscle

586-All the following is true about spasticity except:


A- Usually occurs cerebral palsy infant
B- Occur in sci
C- associated with lower motor neuron legion
D-can cause hip dislocation.

The correct Answer is:


A- usually occurs cerebral palsy infant

587-patient during walking raise his hip to clear toes from ground due to
a-paralysis of dorsiflexors
b-paralysis of planter flexors
c-paralysis of quadriceps

The correct Answer is:


A- paralysis of dorsiflexors

588-During evaluation of a hemiplegic patient you found that there is


balance deficiency, the cause is
A- weakness of glutei
B- weakness of quadriceps
C- spasticity of planter flexors
D- weakness of dorsi flexors

The correct Answer is:


A- weakness of Gluteal

589-In SCI which of the following in not of complications Complete


transection of the spinal cord at the C1 level produces all of the
following effects except:
a- Hypotension.
b- Limited respiratory effort
c- Anaesthesia below the level of the lesion.
d- Areflexia below the level of the lesion.

The correct Answer is:


D- Areflexia below the level of the lesion
Complications of SCI are: Skin Breakdown, pneumonia, Osteoporosis and
Fractures, Heterotopic Ossification, Spasticity, Urinary Tract Infections,
Autonomic Dysreflexia, Deep Vein Thrombosis, Pulmonary Embolism
,Orthostatic Hypotension, Cardiovascular Disease , Syringomyelia ,
Neuropathic / Spinal Cord Pain , Medication Problems , Hyperthermia
,Hypothermia

590-In a patient with head injury, unexplained hypotension warrants


evaluation of
a-Upper cervical spine.
b-Lower cervical spine.
c-Thoracic spine.
d-Lumbar spine

The correct Answer is:


A- upper cervical spine

591-nerve which give motor &sensory branch supplying facial muscle is


a- V
B-VI
c- VII
d- VIII

The correct Answer is:


C- V11 facial nerve

592-Fracture dislocation L4-5 will result in:


a- Quada Equina
b- Flaccid Paraplegia
c- Spastic Quadriplegia
d- Spastic Paraplegia

The correct Answer is:


B- flaccid paraplegia

593-Patient with C 5 quadriplegia on tilting table raised 50 degree during


transfer we used
a- hydrolic lifiting
b- sliding the patient
c- two persons carry & transfere

The correct Answer is:


C- Two persons carry & transfer.
594-A therapist is teaching a family to care for a family member at home.
The patient is totally bedbound. To prevent pressure ulcers most
effectively, what should be the maximal amount of time between position
changes?
a-One hour
b-Two hours
c-Six hours
d-Eight hours

The correct Answer is:


B- Two hours

595-Which of the following is not an acceptable long-term goal for a


patient with a complete C7spinal cord injury?
a- Independence with dressing
b- Driving an automobile.
c- Balance a wheelchair for 30 seconds using a “wheelie”.
d- Independence with performing a manual cough.

The correct Answer is:


C- Balance a wheelchair for a 30 second

596-A therapist is treating a patient with an injury at the T8 level and


compromised function of the diaphragm. If no abdominal binder is
available, what is the most likely position of comfort to allow him to
breathe most efficiently?
A. Sitting position
B. Semi-fowler position
C. Standing
D. Supine

The correct Answer is:


D- supine

597-Patient with spinal cord injury (T6) level you can expect all of this
from him except
a- Independent transfer by manual wheel chair
b- Independent transfer by sliding board
c- Independent bowel and bladder control

The correct Answer is:


C- independent bowl and bladder control

598-WHICH statement incorrect about BROWN SEQUARD SYND


SYNDROME
a- ipsilateral babinski sign
b- ipsilat loss of pain and temp
c- contralat loss of pain and temp
The correct Answer is:
B- ipsilateral loss of pain and temp

599-When o2 demand in heart decrease what happened ?


a- decrease HR multiplied by systolic pressure
b- decrease diastolic pressure
c- increase coronary arteriesa
d- increase HR multiplied by systolic pressure

The correct Answer is:


B- decrease diastolic pressure

600-Average skeletal muscle blood flow is:


a- 1-4 ml/min per 100g √
b- 5-8 ml/min per 100g
c- 9-12 ml/min per 100g
d- 13-16 ml/min per 100g
The correct Answer is:
A- ( At rest, skeletal muscle blood flows may be 1-4 ml/min per 100g;
maximal blood flows may reach 50-100 ml/min per 100g,)

601-Whats the blood flow at resting?


1- 1 to 2 ml/min/110g
2- 6 to7 ml/min/110g √
3-60 to 80 ml/min/110g

The correct Answer is:


B- 6 to7 ml / min / 110g so with increase size=increase blood flow

602-voltage of heart
a- -90
b- +90
c- +60
d- -30

The correct Answer is:


A- -90

603-Right Ventrical of the heart


a- have bulk of muscle
b- have low oxygen blood
c- have greater oxygen blood
d- neither of all

604- Lt vent more important vs Rt vent at accident why


A- Contains unoxygnated blood
B- B transmit blood less o2
C- Tranmitts blood to arota
D-Transmit unoxygnated blood to arota

The correct Answer is:


C- transmit blood to aorta

20 -A patient with restrictive lung disease secondary to circumferential


thoracic burns demonstrates decreased ability to expand the lower rib
cage and push the abdominal wall anteriorly. In this case, the therapist
should consider the use of facilitation techniques to enhance the
function of the:

A-rectus abdominis
B-anterior scalenes
C-internal intercostals
D-diaphragm

The correct answer is :


D- Diaphragm breathing used for restrictive pulmonary disease because
it`s action is in conjunction with the intercostal muscles, it lowers the
pressure in the thoracic cavity. This enables air to enter the lungs. When
breathing out, the diaphragm relaxes along with the intercostal
muscles, allowing air to leave. Air may be forced out faster by
increasing abdominal pressure using the tranverse abdominis muscle
28-in ttt of anterior neck pain 3rd degree which is contra indicated?
a-stretchig scar tissue so put head in extention
b-ultra sonic to decrease adhesions
c-put the head in flexion to avoid deformity

The correct Answer is:


C- put the head in flexion will increase the deformity not prevent it.

678-after pt do ex at gym. to check the tolerance of patient...we take


heart rate from
a- femoral artery 10 sec *6
b- brachial artery 15 sec*4
c- radial artery 30 sec * 2
d- carotid artery 60 se

The correct Answer is:


C-radial artery 30 sec*2 , it`s the more accurate and easier .

679-patient complains of cystic fibrosis in LT lingula


a- put patient on RT sidelying
b-put patient on LT sidelying
c- half lying
c-prone lying

The correct Answer is:


A- put the patient on the Rt side-laying to ease the secretions by this
anti-gravity postion

680-the patient is placed in a supine position with the head flat, postural
drainage is optimal for the

a-apical segments of the upper lobes


b-posterior segments of the upper lobes
c-anterior segments of the upper lobes
d-lingular segments of the left upper lobes

The correct Answer is:


C-anterior segments of the upper lobes

681-coronaryartery for the heart;


a-one b-two c-three d- four
The correct Answer is:
B- Two , Right coronary Artery & left coronary Artery

682-in thrombosis we do
a- deep breathing exercises to enhance blood to heart .
b- deep breathing exercises to to improve lung capacity
c- no deep breathing at all

The correct Answer is:


C – no deep breathing at all to prevent movement of the thrombus

683- Cerebral embolism begin first from


a- heart
b- lung
c- vessels of lower limb

The correct Answer is:


A- heart

684- Pulmonary embolism begin first from


a- heart
b- lung
c- vessels of lower limb

The correct Answer is:


C- vessels of the lower limb

685- the most fatal complication of D.V.T is :


a- fat embolism
b- pulmonary embolism
c- arterial embolism
d- fracture embolism

The correct Answer is:


B-pulmonary embolism

686- A 40 years old man, was admitted with FRACTURE shaft femur
following a road traffic accident. On 2nd day he became disoriented. He
was found to be tachypnoeic, and had conjunctival petechiae. Most
likely diagnosis is:
a- Pulmonary embolism
b- b- Sepsis syndrome
c- Fat embolism
d- Haemothorax

The correct Answer is:


C- Fat embolism

687-Cerebral embolism begin first from


a.heart
b.lung
c . vessels of lower limb

The correct Answer is:


A- heart

688-patient with fracture right ribs 7-10 perform secretion clearance


techniques feels suddenly pain at chest , dyspnea, unable to breathe
and trachea shifted to left side , the cause is
A-Pulmonary emboli
B-Pneumothorax
C-Copious mucus

The correct Answer is:


B- pneumothorax.

689-The most fatal complication of dvt is :


a)fat embolism
b)pulmonary embolism
c)arterial embolism
d)fracture embolism

The correct Answer is:


B- pulmonary embolism.

690-To avoid skin problems in geriatrics


a-Change patient position
b-Begin early ampulation
c-Active exercises

The correct Answer is:


A- Change patient position to prevent bed ulcers.
691-contractility of the heart muscle by age
a- increase
b- decrease
c- lost
d- not changed

The correct Answer is:


B- decreased by age

692-Heart innervation
a-sympathetic
b-parasympathetic
c-pyramidal tract
d-a+b

The correct Answer is:


D- a + b, sympathetic and parasympathetic

693-conduction pathway of the heart


a-Av node+AV bundle+ SA node+ purkinje fiber
b-Av node+AV bundle+ purkinje fiber+ SA node
c- SA node+ Av node+ purkinje fiber +AV bundle
d-SA node +AV bundle+ Av node+ purkinje fiber

The correct Answer is:


D- SA node+AV bundle +AV node+Purkinjie fiber

694-Bursed lip tech used in


A.Asthma
B.emphysema
C.Segmental breathing
D.normal person

The correct Answer is :

B- Emphysema due to it`s restrictive disease , in segmental breathing


used for after block

695-burger disease affect


a-veins
b-artery
c-nerve
d-vessels

The correct Answer is :

D- the vessels of the Arteries or some veins

696- patient needs to increase ROM you can use hold relax or contract
relax technique .....what different between two techniques??
a- isotonic contraction
b- verbal comments
c- direction of movement
d- none of the above

the correct answer is :


A- Isotonic contraction used in contract – relax followed by isometric
contraction and we use Isometric contraction in hold relax technique

697-patient with sever back pain, after examination you find disc
herniation; which one of the following will be the least to prove the
condition :
a- MRI
b- CT
c- plain X-ray
d- mylogram

The correct Answer is :


C- plain X-Ray is the less approval for disc herniation and the best is MRI

698-pseudo claudication is due to compression of:


a-Femoral artery
b-Femoral nerve.
c-Cauda Equina.
d-Popliteal artery

The correct answer is :


A – femoral artery

699-Following a cerebrovascular accident involving the dominant right


hemisphere, a patient exhibiting unilateral neglect would generally not

a-eat food only from the right side of the plate.


b-bump a one-arm driven wheelchair into things on the left side.
c-ignore or deny the existence of the left-sided limbs.
d-shave or put make-up only on the left side of the face.

The correct Answer :

D- shave or put make up only on the left side of the face

700-Which of the following is inappropriate for a physical therapist to


include in the treatment plan of an infant with a gestational age of 27
weeks and Down’s syndrome?

a-Bottle feeding
b-Encourage sidelying position
c-Tactile stimulation with the entire hand rather than the fingertips of
the examiner
d-Prone positioning

The correct Answer is:


D- Prone position is prevented due to the child is a pre-matured and his
lung was still in mature

701-Which of the following sources of stimulation is least effective in


obtaining functional goals when treating an infant with decreased
muscular tone?
a-Vestibular
b-Weight-bearing
c-Cutaneous (scratch , quick stretch ,painful stimulation )
d-Vibratory

the correct answer is :


D- Vibratory less effect on Muscle Tone

702-Which of the following is the most important goal in treating


pediatric patients with postural reaction deficits?
a-Age-appropriate responses
b-Automatic responses
c-Conscious responses
d-Lower extremity control before upper extremity control
The correct Answer is:
B- Automatic responses is related to the position which is responsible
for: Protective , equilibrium , Righting ; all of them is called a postural
reaction

Age appropriate responses : response related to the age and disappear


after growing than that Age like : ASTR . STR , LTR

Conscious Responses: is the response related to conscious and active


movement

703-A patient who has suffered a recent stroke is being treated by a


physical therapist. The patient exhibits increased extensor tone in the
supine position along with an exaggerated symmetric tonic labyrinthine
reflex (STLR). What is the best position to initiate flexion movements of
the lower extremity?

a-Prone position
b-Sidelying position
c-Supine position
d-A and B

The correct Answer is :


A-Prone Position in STLR increase flexion tone

704-A patient’s peripheral skin color progresses from blue to white to


red. This would be most characteristic of:
a-chronic venous insufficiency
b-acute venous insufficiency
c-acute arterial insufficiency
d-vasomotor disorders

The correct Answer is :

D-Vasomotor disorders ,

Chronic venous insufficiency skin color is normal or cyanotic in


dependency
Acute venous insufficiency skin colour is like chronic
Acute arterial insufficiency skin colour is Pale , shiny , dry
705-You are evaluating a patient with chronic obstructive pulmonary
disease. You would not expect to find:
a-the presence of hypercapnia
b-increased total lung capacity
c-increased vital capacity
d-abnormal ventilation/perfusion ratio

The correct Answer is :


C –Increased Vital capacity couldn`t be found in CoPD , but there is
almost hypercapnia , increased of Respiration rate lead to Increased in
total lung capacity , abnormal ventilation /perfusion ratio .

706-outerpart of the heart


a-epicardium
b-endocardium
c-pricardium
d-myocardium

The correct Answer is :

C- pericardium , is The outer sac which enclosed the heart and protect it
.
Epicardium : is the inner layer of the pericardium
Myocardium : is the whole major muscle of the heart
Endocadium : is inner layer which linning valves and heart

707-Following an exercise session in a Phase 3 cardiac rehabilitation


program, in which some patient may experience angina pectoris, the
therapist employs a gradual and prolonged cool down period primarily to
prevent:
a-exertional dyspnea
b-tachycardia
c-venous pooling
d-hypertension

The correct answer is :


b-Tachycardia , we choose it due to prevent angina pectoris which
result from un-sufficiency blood supply with O2 to the heart , but
Generally we make a gradual cooling down period for prevent venous
pooling which mean pulling the blood down to the lower extremity

Medical knowledge

708-Irrevirsable risk factor in coronary artery disease


A. Smoking
B. High blood pressure
C. a Lipo-protine
D. History of occlusive peripheral artery

The correct Answer is :


D- history of occlusive peripheral artery ,

UN-Modifiable risk factors: age, male sex, race, and family history.

Modifiable risk factors: smoking, exposure to second-hand smoke,


hypertension, hyperlipidemia, high cholesterol (total or LDL-C) levels,
low HDL-C) levels, high triglyceride levels, diabetes, abdominal obesity,
sedentary lifestyle, high homocysteine levels, and high levels of C-
reactive protein (which indicates inflammation).

709- A patient asks the therapist to explain the function of his


medication verapamil (a calcium antagonist). Which of the following
points should be conveyed in the therapist's explanation?

a- Verapamil causes decreased contractility of the heart and


vasodilation of the coronary arteries
b- Verapamil causes decreased contractility of the heart and
vasoconstriction of the coronary arteries
c- Verapamil causes increased contractility of the heart and
vasodilation of the coronary arteries
d- Verapamil causes increased contractility of the heart and
vasoconstriction of the coronary arteries

The correct Answer is :


A-Verpamil causes increased in contractility of the heart and
vasodilaton of the coronary artries .
Calcium-Channel Blockers. Calcium plays a central role in the electrical
stimulation of cardiac cells and in the mechanical contraction of smooth
muscle cells in the walls of arteries. Calcium-channel blockers are
relatively new synthetic drugs that work by blocking the passage of
calcium into the muscle cells that control the size of blood vessels,
thereby preventing the muscles of the arteries from constricting.

Examples include diltiazem (Cardizem), nicardipine (Cardene), nifedipine


(Procardia, Procardia XL), nimodipine (Nimotop), and verapamil (Calan,
Isoptin, Verelan)

Possible Side Effects : Excessively slow heart rate, low blood pressure,
headache, swelling of ankle/feet, constipation, nausea, tiredness,
dizziness, redness of face and neck, palpitations, and rash.
CHAPTER 19. PHARMACOLOGY FOR THE PHYSICAL THERAPIST 1261

710-A physical therapist in an outpatient clinic is called into a room to


assist an infant who is unconscious and not breathing. The therapist
opens the airway of the infant and attempts ventilation. The breaths do
not make the chest rise. After the infant’s head is repositioned, the
breaths still do not cause the chest to move. What should the therapist
do next?

a-Give five back blows


b-Look into the throat for a foreign body
c-Have someone call 911

The correct answer is :


B-look into the thorat for foreign body

711-Perform a blind finger sweep of the throat Which of the following


circumstances would normally decrease body temperature in a healthy
person?
a-Exercising on a treadmill.
b-Pregnancy
c-Normal ovulation
d-Reaching age of 65 years or older

The Correct Answer is:


D-Reaching age of 65 years or Older
712-Acromegaly is due to malfunction of :
a-Pituitary gland
b-Thyroid gland
c-thymus

The correct Answer is :


A- pituitary gland due to increase release of Growth Hermon

713-Increase growth of bone :


a- ant lobe of pituitary gland
b- thyroid

The correct Answer is :


A- Anterior pituitary gland is responsible for release of (GH) ,
Acromegaly is due to malfunction of Growth Hermon a syndrome that
results when the anterior pituitary gland produces excess growth
hormone (GH) after epiphyseal plate closure at puberty

714- A therapist is treating a patient with spinal cord injury. The


therapist is discharging the patient after
completion of all physical therapy goals. One of the completed long-term
goals involved the ability to dress
and bathe independently with assistive devices. This would be a most
challenging but obtainable goal for
which of the following?
a-C5 quadriplegia
b-C7 quadriplegia
c-T1 paraplegia
d-C4 quadriplegia
The correct Answer is :
B-C5 quadriplegia which means that the lesion isn`t affect the pharanic
nerve which supply the diaphragm and it supply musculocutaneous
nerve Tricepses muscle

715-A therapist is evaluating a wound in a patient with the following


signs: the right foot has a toe
that is gangrenous, the skin on the dorsum of the foot is shiny in
appearance, and no calluses are present. The patient has what type of
ulcer?
a-Venous insufficiency ulcer
b-Arterial insufficiency ulcer
c-Decubitus ulcer
d-Trophic ulcer

The correct Answer is : B- Arterial insufficiency ulcer due to present of


gangrenous , shiny dermis and no calluses which may lead to decubitus
or Trophic
ulcers

716-A physician is preparing a patient for an upcoming procedure. The


physician explains that the procedure will provide a detailed image that
appears to be a slice of the brain. This image is obtained with a highly
concentrated x-ray beam. What procedure is the patient scheduled to
undergo?

a-Angiogram
b-Magnetic resonance imaging (MRI)
c-Positron emission tomography (PET)
d-Computed tomography (CT)

The correct Answer is:

D- CT, Computer-assisted tomography, also known as conventional


tomography, uses a fanlike beam of x-rays to provide an almost three-
dimensional, or tomographic, image. The CT scan provides good
visualization of the shape, symmetry, and position of structures by
delineating specific areas.

Angiography: the radiographic study of the vascular system. A water-


soluble radiopaque dye is injected either intra-arterially (arteriogram) or
intravenously (venogram). A rapid series of radiographs is then taken to
follow the course of the contrast medium as it travels through the blood
vessels. Angiography is used to help detect injury to or partial blockage
of blood vessels

Positron Emission Tomography (PET). A technique in


which a radioactively labeled (positron-emitting isotope) molecule is
used. One of these isotopes is incorporated into a biologically relevant
molecule and injected into or inhaled by the subject. It then
concentrates in brain areas according to the molecule's chemistry and
the brain's metabolic and blood flow demands. These areas are detected
when a positron is emitted from the molecule. Because a wide range of
molecules can be used, PET can provide measurements of blood flow,
blood volume, brain metabolism (especially glucose), and neuro-receptor
or neurotransmitter chemistry.

MAGNETIC RESONANCE IMAGING : (MRI) uses the magnetic


characteristics of the body's tissues rather than ionizing radiation to
produce an image so that the resulting image is the result of the
interaction of body tissues with electromagnetic forces.

Ref :CHAPTER 5. FUNDAMENTALS AND CORE CONCEPTS 223

717-A patient reports to therapy stating that his “sugar is too high” for
exercise. What is the minimal blood glucose level that is considered too
high for a diabetic patient to begin exercise?
a -300 mg/dl
b-400 mg/dl
c-300 g/dl
d-400 g/dl

The correct Answer is:


A-300 mg/dl is the minimum level that considered high to perform an
Exe`s.

718- Which don't present in dermis ?


a- tactile
b- blood vessels

The correct Answer is:


A- Tactile tissue isn`t present in dermis

719-Which of the following are tests for peripheral arterial involvement


in a patient with complaints of calf musculature pain?
a-Claudication time
b-Homan’s sign
c-Percussion test
d-None of the above

The correct Answer is :


A – claudication time test is used for peripheral arterial disease ,

Homan`s sign test is a Discomfort in the calf muscles on forced


dorsiflexion of the foot with the knee straight has been a time-honored
sign of DVT. However, this sign is found in more than 50% of patients
without DVT and is present in less than one-third of patients with
confirmed DVT, making it very nonspecific.

Percussion Test : The percussion test is used to determine the


competence of the greater saphenous vein. While the patient is
standing, the clinician palpates one segment of the vein while
percussing the vein at a point approximately 20 cm higher. If a pulse
wave is felt by the lower hand, an assumption can be made that the
intervening valves are incompetent.

720-The therapist is ordered to evaluate a patient in the intensive care


unit. The patient appears to
be in a coma and is totally unresponsive to noxious, visual and auditory
stimuli. What rating on the
Rancho Los Amigos Cognitive Functioning Scale is most appropriate?
a-I
b-III
c-IV
d-VI

The correct Answer is :


A- grade 1

721-At what age does a human have the greatest amount of fluid in the
intervertebral disc?
a-1 year
b-4 years
c-7 years
d-10 years

The correct Answer is :


A- one year

722-The most common type of stroke is and its primary precipitating


factor is
a- Atherothrombotic, atherosclerosis
b- Atherothrombotic, hypertension
c- Hemorrhage, atherosclerosis
d- Hemorrhage, hypertension

The correct Answer is :

A- Atherothrombotic , Atheroscelerosis next to it Atherothrombotic


hypertension ,
723-A patient who has suffered a recent fracture of the right tibia and
fibula has developed foot drop of the right foot during gait. Which nerve
is causing this loss of motor function?

a-Posterior tibial
b-Superficial peroneal
c-Deep peroneal
d-Anterior tibial

the correct Answer is :


B- superfascial peroneal nerve which passes near to fabula and motor
innervated for dorsiflexion

724- It is most likely than when treating a patient with Lyme Disease of
more than one year’s duration, the physical therapy focus will be on
management of arthritic changes primarily affecting the:

a-small joints of the hands and feet


b-large joints of the body; especially the knee
c-axial joints, especially the lumbosacral spine
d-axial joints, especially the cervical and thoracic spine

The correct Answer is :


B-large joints especially the knee
Presenting features of Brown-Séquard's syndrome

There is a total ipsilateral loss of sensation (analgesia and thermo


anaesthesia) with flaccid paralysis at the level of the lesion.

There is contralateral loss of pain and temperature beginning a few


segments below the lesion (because the spinothalamic tracts enter the
cord and travel ipsilaterally for a few segments before decussating).

No plantar response on this side because of loss of pain sensation.

There is ipsilateral spastic paraparesis with loss of vibration and joint-


position sense
(destruction of ipsilateral dorsal column fibres) below the lesion.
Reflexes are brisk with up going plantar reflex. There may be an
ipsilateral Horner's syndrome if the sympathetic fibres are damaged (in
the neck).[9][10]

There are also sphincter disturbances.

Incomplete forms of the syndrome commonly occur, usually caused by


vascular impairment secondary to compression of the cord, with sparing
of the dorsal columns (separate vascular supply); or inflammatory lesions
(for example, multiple sclerosis).

************************************************************************
**************

Autonomic Nervous System originate and radiate from certain vertebras


on the Spinal Cord at the backbone.
At the middle of each neuron there is a ganglion, before which the
neuron is called a
preganglionic, and after which is called a postganglionic.

The Autonomic Nervous System is responsible for regulation of the


involuntary activities in the body as heart beats, movement of the bowel
and vision accommodation. Also it's involved in cases of stress, rest and
digestion.

The ANS neurons is subdivided into sympathetic and parasympathetic.


Cholinergic d 'the preganglionic neurons are short and calle In the
sympathetic system, d neurons'.

The postganglionic ones are long ending and embedded in the receptor
sites an neurons'.

Adrenergic called ' in the parasympathetic system, the preganglionic


neurons are long and the postganglionic neurons are short and both of
them are 'cholinergic neurons'.

N.B: the nomination of cholinergic or adrenergic is due to the type of the


neurotransmitters run through the neuron itself either Acetyl Choline or
Nor-Adrenalin respectively

Reflexes In Pediatric:

THE ROOTING REACTION

Onset : 28 weeks gestation

Integration : 3 months

Testing position : with the infant supine, the head in midline and hands
on chest

Procedure : gently stroke the infant from the lips to the cheek

Normal response : the infant should turn his head toward the stimulated
side with the mouth opening and a trial of sucking the finger.
May not be present if the infant is not hungry.

MORO
Onset : begins at 28 weeks gestation

Integration : 5-6 months

Testing position : child in supine with head in midline, support the child's
head while
pulling the child to a position halfway between supine and upright sitting
Procedure : support the infant’s head and shoulders with one hand. Allow
the neck to drop back to allow the anterior neck muscles to stretch

Normal response : the shoulders abduct, the elbows, wrists and fingers
extend. Subsequently, the shoulders adduct, and the elbows and fingers
flex

ASSYMETRICAL TONIC NECK REFLEX (ATNR)

Onset : 0-2 months

Integration : 4-6 months

Testing position : supine

Procedure : gently turn the infant’s head to one side

Normal response : a UE flexion tone on the side opposite to the head turn
with an increase in UE extensor tone in the side to which the head is
turned

TONIC LABYRINTHINE REFLEX (TLR)

Onset : birth

Integration : 6 months

Testing position : prone or supine

Procedure : * prone : lift the infant up to evaluate for the presence of


flexor tones * supine : lift the child to sitting to observe for extensor tone

Normal response : prone - flexor tone will dominate; supine - extensor


tone

SYMMETRICAL TONIC NECK REFLEX (STNR)

Onset : 4-6 months

Integration : 8 - 12 months
Testing position : child in quadruped position on the floor

Procedure : passively flex the head forward and then extend it


backwards

Normal response : forward head flexion will produce flexion of the upper
extremities and extension of the lower extremities; extension of the
head will produce extension of the upper extremities and flexion of the
lower extremities

MATURE NECK RIGHTING ACTING ON THE BODY (NOB)

Onset : 4-6 months

Integration : 5 years

Testing position : supine

Procedure : flex the child’s head and slowly turn to one side; hold this
position and observe, repeat on the opposite side

Normal response : child will segmentally roll in the direction of the head
rotation

DEFINITIONS

Agnosia: Loss of knowledge or inability to perceive Objects

Anaesthesia: Absence of sensation

Ataxia: Loss of coordination affecting functional movement

Babinski sign : Abnormal response of the plantar reflex (great toe turns
upwards on testing)

Bradykinesia : Slowness of movement

Clonus : Succession of intermittent muscular relaxation and contraction


usually resulting from a sustained stretch

Diplopia : Double vision


Dysarthria : Incoordination of speech

Dysphagia : Difficulty in swallowing

Dysphasia : Disruption of expressive (produce) and or


receptive(understand) speech

Dyspraxia: Inability to execute volitional purposeful movements

Dystonia : Involuntary movement characterised by twisting and repetitive


movement
Flaccidity : Absence of muscle tone

Hemianopea : Loss of visual field in one half of each eye

Hemiplegia : Paralysis of one side of the body

Hypertonicity : Increased muscle tone

Hypotonicity : Decreased muscle tone

Nystagmus : Involuntary rhythmic oscillation of one or both eyes

Paraesthesia: Disruption of sensation causing abnormal sensation

Ptosis: Drooped eyelid

Rigidity: Stiffness of neurological origin, increased resistance to stretch


throughout the range
Tone :The active resistance of muscle to stretch

Tremor: Fine type of involuntary movement (several types seen in


neurological dysfunction)

Galliazi fracture : is fracture of radius with sublaxation of ulna

Montagia fracture : is fracture of ulna with sublaxation of radius

March fracture : is fracture of metatarsal bones due to long standing


( stress Fracture )

pulmonary ventilation : exchange between atmospheric air and lung


air

inspiratory capacity : the volume of gas that can be taken into the
lungs in a full inhalation

total lung capacity : the amount of gas contained in the lung at the end
of a maximal inhalation vital capacity : the maximal volume of gas that
can be exhaled from full inhalation

tidal volume : the volume of gas inhaled and exhaled during one
respiratory cycle

residual volume : the amount of gas remaining in the lung at the end of a
maximal

exhalation stroke volume : the volume of blood ejected from a ventricle


at each beat of the heart, equal to the difference between the end-
diastolic volume and the end systolic volume.

end-diastolic volume(EDV) : the volume of blood in each ventricle at the


end of diastole, usually about 120–130 mL but sometimes reaching 200–
250 mL in the normal heart

end-systolic volume(ESV) : the volume of blood remaining in each


ventricle at the end of systole, usually about 50–60 mL but sometimes as
little as10–30 mL in the normal heart.

idiopathic sclerosis : lateral curvature of thoracic spine

extrafusal ms fiber innervated by : Alfa motor neuron

intrafusal ms fiber innervated by : Gamma motor neuron

Milwaukee brace : used in scoliosis of thoracic spine

Boston brace : used in scoliosis of lumbar spine

Galezzi fracture : fracture radius & dislocation ulna

Montagia fracture : fracture ulna & dislocation radius

Rayunads disease : bilateral symmetrical …… right answer


Burger’s disease: due to smoking

Carpal tunnel : median n compression

Tarsal tunnel : tibial n compression … posterior

Lumbar Vertebrae Information


Patellar L3, 4
Achilles S1, 2

*Segmental innervation of some muscles of the lower limb (Spinal roots


Muscle innervated) :
L 2/3 hip flexors ILIOPSOAS
L 3 hip adductors ADDUCTOR LONGUS
L 3/4 knee extensors VASTUS LATERALIS VASTUS MEDIALIS
L5 ankle dorsiflexion eversion and inversion + hip abductors
S 1 ankle plantar flexion + hip extensors

*Segmental innervation of some muscles of the upper limb (Spinal roots


Muscle innervated):
C5/6 DELTOID BICEPS BRACHIORADIALIS INFRASPINATUS
SUPRASPINATUS
C 6/7 PRONATOR TERES FLEXOR CARPI ULNARIS
C7 TRICEPS LATISSIMUS DORSI
C 7/8 extensors and flexors of the wrist S 1 intrinsic muscles of the hand

Proprioceptive neuromuscular facilitation :

1. PROPRIOCEPTIVE NEUROMUSCULAR FACILITATION includes


• PNF definition • Neurophysiologic basis of PNF • Uses of PNF • 9 basic
principles of PNF• Techniques of PNF • PNF stretching • Patterns of PNF

3. DEFINITION: Proprioceptive: having to do with any of the sensory


receptors that give information concerning movement and position of
the body
• Neuromuscular: involving the nerves and muscles
• Facilitation: making easier

4. • Proprioceptive neuromuscular facilitation is exercise based on the


principles of functional human anatomy and neurophysiology.• It uses –
Proprioceptive – Cutaneous – Auditory input To produce functional
improvement in motor output and can be a vital element in the
rehabilitation process of sports related injuries.

5. NEUROPHYSIOLOGICAL BASIS OF PNF


• Sherrington – Concepts of facilitation and inhibition – Stretch reflex –
Neurophysiological phenomena

6. FACILITATION• Facilitory - an impulse causing the recruitment and


discharge of additional motor neurons in the spinal cord – Results in
increased excitability in the muscles. – Weak muscles would be aided
through facilitation

7. INHIBITION• Inhibitory - any stimulus that causes motor neurons to


drop away from the discharge zone and away from the spinal cord. –
Inhibition results in decreased excitability of motor neurons. – Muscle
spasticity can be decreased

8. STRETCH REFLEX• the stretch reflex involves two types of


receptors Muscle spindles Golgi tendon organs

9. NEUROPHYSIOLOGICAL PHENOMENA : RECIPROCAL INHIBITION ,


AUTOGENIC INHIBITION is defined as inhibition mediated by afferent
fibers from stretched muscle acting on the alpha motor neurons
supplying that muscle, causing it to reflex

10. Reciprocal inhibition• Is the second mechanism which deals with the
relationships of the agonist and antagonist muscles

11. USES OF PNF•:


1. PNF treatment has been used to increase strength, flexibility,
coordination and functional mobility.
2. The main goal of treatment is to facilitate the patient in achieving a
movement or posture.
3. Stretches as well as diagonals and rational exercise patterns are
used to improve ADL’s functional mobility and athletic performance
4. It is mainly used in orthopedic rehabilitation for musculoskeletal
injuries and in neurological rehab.•
5. PNF can be used for any condition , however the patient condition
level may require modifications.

13. BASIC PRINCIPLES OF PNF


1. Resistance
2. Irradiation and reinforcement
3. Manual contact
4. Stretch
5. Verbal commands
6. Traction and approximation
7. Timing
8. Body positioning and body mechanics

14. RESISTANCE opposing force to the patient’s movement is called


resistance. The amount of resistance provided during an activity must
be correct for the patient’s condition and the goal of the activity. This is
called optimal resistance.

15. RESISTANCE is used in the treatment to:


1. Facilitate the ability of the muscle to contract
2. Increase motor control
3. Help the patient gain an awareness of motion and its direction
4. Increase strength

16. IRRADIATION & REINFORCEMENT DEFINITIONS


• Irradiation : the spread of response to stimulation is called irradiation.•
Reinforcement : means “to strengthen by fresh addition, make stronger”

17. IRRADIATION & REINFORCEMENT Effects :


• Maximal resistance may be used to cause irradiation or overflow from
stronger patterns to weaker patterns or from stronger groups of muscles
within a pattern to weaker groups within the same pattern.

18. MANUAL CONTACT Effects:


1. Stimulates the muscle
2. Stimulates the synergistic muscle to reinforce the movement
3. Promotes trunk stabilization and indirectly helps the limb motion
4. Prevents confusion

19. Touch or manual contactContributes to facilitation by stimulating


the exteroceptors and it should be1. Purposeful2. Directional3.
comfortable

20. STRETCH• The stretch stimulus occurs when the muscle is


elongated• The lengthened position of the muscle is the starting position
of each pattern and the stretch is maintained throughout the
movement.• All the components of a pattern must be stretched
simultaneously
21. STRETCH Effects:1. Stimulates the activity of muscle spindle2. Any
contraction of muscle on stretch will result in movement and the brain
knows not of muscles but of movement.

22. Quick Stretch Muscle Spindle + ++ Alpha Motor Neuron

23. TRACTION• Traction is elongation of trunk or an extremity• Traction


force is applied gradually, maintained throughout the movement, and
combined with appropriate resistance.

24. TRACTION• Joint separation stimulates joint receptors• Muscle


stretch stimulates muscle spindle stretch receptor• Facilitates Alpha
Motor Neuron• Facilitates Strength

25. APPROXIMATION• Definition: Approximation is the compression of


the trunk or an extremity.• Compression through a joint stimulate joint
receptors• Facilitate alpha motor neuron• Facilitate stability

26. APPROXIMATION Uses:1. Promote stabilization2. Facilitate weight


bearing and contraction of postural muscles3. Facilitate upright
reactions4. Resist some component of motion. E.g., use approximation
at the end of shoulder flexion to resist scapula elevation (11)

27. VERBAL STIMULATION (COMMANDS) • The volume with which the


command is given affects the strength of resulting muscle contraction. •
Louder command when strong muscle contraction is required. Softer
and calmer tone when the goal is relaxation and relief of pain.

28. VERBAL STIMULATION (COMMANDS)• The command is divided into


three parts:1. Preparation: readies the pt for action. “ready”2. Action:
tells the pt to start the action. “now pull your leg up and in”3.
Correction: tells the pt how to correct and modify the action. “keep
pulling your toes up”

29. Commmads used• HOLD• PULL/PUSH• RELAX

30. TIMINGS• Timing is the sequencing of motions• Normal timing of


most coordinated and efficient motions is from distal to proximal•
Timing for emphasis involves changing the normal sequencing of motion
to emphasis a particular muscle or desired activity

31. BODY POSTION & BODY MECHANICS • The therapist body should be
in line of motion • Shoulder and pelvis face the direction of motion. •
Therapist stands in walk standing position. • The resistance comes from
the therapist’s body, while the hands and arms stay comparatively
relaxed.

32. TECHNIQUES OF PNF • Rhythmic initiationStrengthening • • Repeated


contraction Slow reversal techniques • Slow reversal-hold • Rhythmic
stabilization Stretching • Contract relax • Hold relax techniques

33. RHYTHMIC INITIATION• Progression from( agonist pattern) PASSIVE


ACTIVE ASSISTED ACTIVEUSED IN• Limited ROM due to increase tone•
Who are unable to initiate movement

34. REPEATED CONTRATION• Patient move isotonically against


maximum resistance repeatedly until fatigue is evidenced• When fatigue
is evident then a stretch at that point in the range should facilitate the
weaker muscles and results in coordinated movement.• USED• To
develop strength and endurance.

35. SLOW REVERSAL• Involves isotonic contraction of the agonist


followed immediately by an isotonic contraction of the antagonist.•
USED1. For development of active ROM and2. Normal reciprocal timing
b/w agonist and antagonist

36. SLOW REVERSAL HOLD• Involves isotonic contraction of the agonist


followed immediately by an isometric contraction, with a hold command
given at the end of each active movement.• USED• In developing
strength at a specific point in the range of motion.

37. RHYTHMIC STABILIZATION• Uses an isometric contraction of the


agonist, followed by an isometric contraction of the antagonist.• USED•
To increase strength and endurance

38. STRETCHING TECHNIQUES/PNF STRETCHING• It is often a


combination of passive stretching and isometrics contractions.•
encourage flexibility and coordination throughout the limbs entire range
of motion.• PNF is used to supplement daily stretching and is employed
to make quick gains in range of motion to help athletes improve
performance.• Good range of motion makes better biomechanics,
reduces fatigue and helps prevent overuse injuries.

39. CONTRACT-RELAX• Moves the body part passively into the agonist
pattern.• Patient is instructed to push by contracting the antagonist
isotonically against the resistance.• USED• When ROM is limited by
muscle tightness.

40. HOLD RELAX• Begins with isometric contraction of the antagonist


against resistance, followed by concentric contraction of the agonist
muscle.

41. PNF STRETCHINGThe initial movement is in the direction of the


stretchNext the athlete pushes in a direction against the stretch The
last movement is a repeat of the initial

42. PNF PATTERNS• Each pattern has three dimension –


1. Flexion or extension
2. Abduction or adduction
3. Rotation•
Movement occurs in a straight line, in diagonal direction with a rotatory
component
43. UPPER EXTREMITY F – ABD –ER , F – ADD – ER , E – ABD -IR , E – ADD
- IR

44. PATTERNS D1 Flexion ,


D2 Flexion Shoulder FLEX, ADD,
ER Shoulder FLEX, ABD, ER
Forearm - Sup Forearm – Sup Wrist - Rad. Flexion Wrist - Rad. Flexion
Fingers - flexion Fingers – Extension SHOULDER

D1 Extension D2 Extension
Shoulder EXT, ABD, IRShoulder EXT, ADD, IR Forearm – Pron Forearm -
Pron Wrist - Ulnar. Extension Wrist - Ulnar ext. Fingers – Extension
Fingers - flexion

45. F – ADD –ER / E – ABD – IR/ F – ABD - ER / E –ADD – IR


Diagonal One Diagonal Two

46. LOWER EXTREMITY F – ABD – IR/ F –ADD – ER / E – ABD – IR/ E – ADD


-ER
47. LOWER EXTREMITY
48. LOWER TRUNK
49. UPPER TRUNK
50. PNF IN SPORTS• Here are some other general guidelines when
completing PNF stretching:
• 1. Leave 48 hours between PNF stretching routines.
• 2. Perform only one exercise per muscle group in a session.
• 3. For each muscle group complete 2-5 sets of the chosen exercise.
• 4. Each set should consist of one stretch held for up to 30 seconds
after the contracting phase.
• 5. PNF stretching is not recommended for anyone under the age of 18.
• 6. If PNF stretching is to be performed as a separate exercise session,
a thorough warm up consisting of 5-10 minutes of light aerobic exercise
and some dynamic stretches must precede it.

• JOURNEL OF ATHLETIC TRAINING•


PNF techniques are most frequently applied during rehabilitation of the
knee, shoulder, and hip, ankle rehabilitation has increased.

• The most frequently used techniques were contract- relax and hold-
relax
• The use of PNF techniques in the muscle re- education phase of
rehabilitation
Muscles:

Anda mungkin juga menyukai